Download as pdf or txt
Download as pdf or txt
You are on page 1of 109

R.

Gupta's ®
Popular Master Guide

CUET-UG
Common University Entrance Test for
Under Graduate Courses/Programmes

DOMAIN SPECIFIC SUBJECT

Political Science
By
RPH Editorial Board
Published by
O.P. Gupta for Ramesh Publishing House
Admin. Office
12-H, New Daryaganj Road, Opp. Officers' Mess,
New Delhi-110002  23261567, 23275224, 23275124
E-mail: info@rameshpublishinghouse.com
For Online Shopping: www.rameshpublishinghouse.com

Showroom
 Balaji Market, Nai Sarak, Delhi-6  23253720, 23282525
 4457, Nai Sarak, Delhi-6,  23918938

© Reserved with the Publisher

No Part of this book may be reproduced or transmitted in any form or by any means, electronic or
mechanical including photocopying, recording or by any transformation storage and retrieval system
without written permission from the Publisher.

Indemnification Clause: This book is being sold/distributed subject to the exclusive condition that neither
the author nor the publishers, individually or collectively, shall be responsible to indemnify the buyer/user/
possessor of this book beyond the selling price of this book for any reason under any circumstances.
If you do not agree to it, please do not buy/accept/use/possess this book.

Book Code: R-2717

ISBN: 978-93-5477-345-7

HSN Code: 49011010


EXAMINATION STRUCTURE

SECTION-II: DOMAIN SPECIFIC SUBJECT

No. of Questions Subject Time


40 Questions  Input text can be used for
to be MCQ Based Questions 45 minutes for
attempted out  MCQs based on NCERT Class XII each Domain
of 50 Syllabus only Specific Subjects

 Mode of the Test : Computer Based Test (CBT)


 Test Pattern : Objective type with Multiple Choice Questions
 Medium of Exam : 13 Languages (Tamil, Telugu, Kannada,
Malayalam, Marathi, Gujarati, Odiya, Bengali,
Assamese, Punjabi, English, Hindi and Urdu)

(iii)
CONTENTS

SAMPLE PAPER

CUET (UG) – Political Science ........................................................................................... 1-4

SECTION-II: DOMAIN SPECIFIC SUBJECT

 POLITICAL SCIENCE
(Multiple Choice Questions) ............................................. 1-96

— Political Theory

— Political Institutions in India

— Politics in India Since Independence

— India’s Foreign Policy

— Governance and Public Policy in India

— Comparative World Politics

— International Relations & International Organisation

— Miscellaneous MCQs.

    

(iv)
EXAM BITES

This Pdf Is
Downloaded From
www.exambites.in

Visit www.exambites.in for


More Premium Stuffs,Latest
Books,Test Papers,Lectures etc.
jeeneetadda
jeeneetadda_official
jeeneetadda

VISIT NOW !!
SAMPLE PAPER (SOLVED) Sample Paper 1

CUET-UG

POLITICAL SCIENCE*

SECTION-II : DOMAIN SPECIFIC SUBJECT

1. Karl Marx, the founder of Marxism, formally was the 6. Feminists argue that women’s values are based
citizen of which one of the following countries? primarily on
A. Germany B. France A. Religious laws
C. UK D. None of the above B. Prescribed social roles
C. Biology and prescribed social roles
2. Which of the following writings were associated with
D. Biology
Muhammad Iqbal?
(a) Tarana-e-Hind 7. Who said that political work is the life blood of all
(b) The Reconstruction of Religious Thought in India economic work?
(c) Historical Role of Islam A. John Rawls B. Mao Zedong
(d) The Bang-e-Dara C. Karl Marx D. Frantz Fanon
Select the correct answer from the given options: 8. Given below are two statements, one labelled as
A. (a), (b) and (c) B. (a), (c) and (d) Assertion (A) and the other labelled as Reason (R):
C. (a), (b) and (d) D. (b), (c) and (d) Assertion (A) : According to Gandhi, politics divor-
ced from religion is a deathtrap.
3. Match List-I with List-II:
Reason (R) : The entry of religion into politics is
List-I List-II
necessary because without religion it
(Books) (Thinkers)
may take communal colour.
(a) Marx, Gandhi and (i) Aurobindo
Socialism On the basis of the two statements choose the correct
(b) Gandhi and (ii) Rabindranath answer from the options given below:
Gandhism Tagore A. Both (A) and (R) are true and (R) is the correct
(c) Savitri : A Legend (iii) Ram Manohar explanation of (A)
and a Symbol Lohia B. Both (A) and (R) are true, but (R) is not the
(d) Gitanjali (iv) B.R. Ambedkar correct explanation of (A)
C. (A) is true, but (R) is false
Choose the correct option from those given below: D. (A) is false, but (R) is true
(a) (b) (c) (d)
A. (iii) (iv) (i) (ii) 9. Given below are two statements, one labelled as
B. (ii) (i) (iv) (iii) Assertion (A) and the other labelled as Reason (R):
C. (iv) (iii) (i) (ii) Assertion (A) : In Machiavellian thought end justifies
D. (iii) (i) (ii) (iv) the means.
Reason (R) : The key objective of the Prince was
4. Who said that art, religion and philosophy differ only to bring unity and integrity of the
in form, their purpose is the same? State.
A. Georg Lukacs B. Sartre
C. Hegel D. Erik Olin Wright On the basis of the two statements choose the correct
answer from the options given below:
5. Which one among the following is NOT related to A. Both (A) and (R) are true and (R) is the correct
Neo-Marxism? explanation of (A)
A. Jurgen Habermas B. Erich Fromm B. Both (A) and (R) are true, but (R) is not the
C. Herbert Marcuse D. Rosa Luxemburg correct explanation of (A)

*40 Questions to be attempted out of 50 1 2717 (SP)—1


2 Sample Paper

C. (A) is true, but (R) is false Choose the correct option from those given below:
D. (A) is false, but (R) is true (a) (b) (c) (d)
A. (iii) (iv) (i) (ii)
10. Who stated that placing restrictions on freedom of
B. (ii) (i) (iv) (iii)
expressions of human beings is like robbing off the
C. (iii) (i) (iv) (ii)
present and the future races?
D. (i) (ii) (iii) (iv)
A. J.S. Mill B. John Locke
C. Isaiah Berlin D. F.A. Hayek 18. Which one of the following services is not mentioned
in the Constitution?
11. The Idea of Justice as fairness in Rawlsian theory
A. Indian Administrative Service
flows from
B. Indian Police Service
A. Individual B. Society
C. Indian Foreign Service
C. State D. Nation
D. Indian Forest Service
12. Who wrote the book, Multicultural Citizenship: A
19. The ‘Clash of Civilization’ thesis means:
Liberal Theory of Minority Rights (1995)?
A. Religious clashes would be a core area of study
A. T.H. Marshal B. B.S. Turner
B. a rejection of the cultural and political inheritance
C. Will Kymlica D. Bheekhu Parekh
of the West
13. Which one of the following Commissions are statutory C. widespread disagreement over moral issues
bodies? D. conflict would not be primarily ideological or
(a) National Commission for Scheduled Tribes economic but, rather cultural in character
(b) National Commission for Human Rights
20. Which International Agreement gave for the ‘first
(c) National Commission for Minorities
time’ legally binding targets on reducing Greenhouse
(d) National Commission for Scheduled Castes Emissions to the countries?
Select the correct answer from the given options: A. Copenhagen 2009
A. (a) and (b) B. (b), (c) and (d) B. Kyoto Protocol 1997
C. (a) and (d) D. (b) and (c) C. Durban Conference 2011
D. Rio de Janerio 1992
14. Who among the following summarizes the themes of
New Public Administration under five heads - relevance, 21. Under the ‘Indus Water Treaty’ with Pakistan, India
values, social equity, change and client-focus? has the exclusive rights over the rivers
A. Frank Marini B. D. Waldo A. Chenab, Ravi, Beas
C. F.W. Taylor D. M.P. Follett B. Ravi, Beas, Sutlej
C. Beas, Sutlej, Jhelum
15. The term ‘flying bees’ got associated with which of
D. Ravi, Chenab, Jhelum
the following associations?
A. ASEAN 22. India has signed Comprehensive Economic Partnership
B. SAARC Agreement (CEPA) with which of the following
C. Shanghai Cooperation Organisation countries?
D. BRICS A. USA B. Russia
C. Japan D. Israel
16. Which of the following statements is true with regard
to NPT and CTBT? 23. Who among the following firmly believed that
A. India has signed but not ratified NPT. periodic communitarian correction is required because
B. India has signed CTBT. ‘Liberalism is a self-subverting doctrine’?
C. India has signed NPT. A. John Rawals B. R. Nozick
D. India has neither signed nor ratified either NPT or C. R. Dworkin D. M. Walzer
CTBT. 24. Who among the following said, “Political behaviour
17. Match List-I with List-II: stands for both an intellectual tendency and a concrete
List-I List-II academic movement?
(Systems) (Articles) A. R.A. Dahl B. G.A. Almond
(a) Electoral system (i) 243-243(O) C. Arthur Bentley D. David Easton
(b) Services under the (ii) 124-147 25. Who said, “A Government without a Constitution is
Union and States a power without right”?
(c) Panchayats (iii) 324-329 A. Thomas Paine B. Karl Deutsch
(d) Judicial system at Union level (iv) 308-323 C. David Apter D. Walter Bagehot
Sample Paper 3
26. By which Constitutional Amendment Act, was ‘Forest D. The political ideas that do not embody or
and Protection of Wild Animals’ shifted from State to articulate class or social interests
Concurrent list?
34. Which of the following are true regarding a revolution?
A. 39th Constitutional Amendment
(a) Change of Constitution
B. 40th Constitutional Amendment
(b) One ruling elite is replaced by another
C. 41st Constitutional Amendment
(c) Transformation in Political Institutions
D. 42nd Constitutional Amendment
(d) Fundamental change in the Socio-economic and
27. Which Article of the Constitution enjoins/bounds all political structures
authorities to act in the aid of the Supreme Court of Code:
India? A. (a) and (b) B. (b) and (c)
A. Article 139 B. Article 141 C. (a) and (d) D. (a), (b), (c), (d)
C. Article 144 D. Article 142
35. Match the items of List-I with items of List-II by
28. Which of the following Committees are helped by the selecting the correct code:
CAG in exercising legislative control over the List-I List-II
executive? (Authors) (Books)
(i) Estimates Committee (a) Robert Dahl (i) Legality and Legitimacy
(ii) Public Accounts Committee (b) S.M. Lipset (ii) Polyarchy : Participation
(iii) Business Advisory Committee And Opposition
(iv) Committee on Government Assurances (c) Schmittcarl (iii) Political Man
Choose the correct answer from the code given below: (d) Roger Charlton (iv) Political Realities : Com-
A. Only (i) and (ii) parative Government
B. Only (ii) and (iv) Code:
C. Only (i), (ii) and (iv) (a) (b) (c) (d)
D. (i), (ii) (iii) and (iv) A. (ii) (iii) (i) (iv)
B. (iii) (i) (iv) (ii)
29. Who among the following suggested the abolition of
C. (i) (iv) (ii) (iii)
the office of the Comptroller and Auditor-General of
D. (iv) (ii) (iii) (i)
India?
A. Paul Appleby B. A.D. Gorwala 36. Which article of the Indian Constitution empower the
C. N.N. Vohra D. L.K. Jha Parliament to make provisions for a contingency when
the offices of both the President and the Vice-President
30. In which of the following countries can a person enter
the public service at almost any level and at almost fall vacant by reasons of death, resignation, removal
any age? or otherwise?
A. UK B. USA A. Article 123 B. Article 72
C. India D. France C. Article 70 D. Article 58

31. When did P.V. Narshima Rao unveil the ‘Look East 37. Who have argued that far from an objective
Policy’? reality, international politics is a ‘World of our making’?
A. In Singapore, through his lecture in September A. Realists B. Feminists
1994 C. Idealists D. Constructivists
B. In January 1992, when India became sectoral 38. Democratic peace theory has been largely associated
dialogue partner of ASEAN with the writings of:
C. In 1995, on the occasion of India’s elevation as A. Immanuel Kant and Voltaire
‘full dialogue partner’ of ASEAN B. Michael Doyle and Bruce Russett
D. Through MEA Annual Report for the year 1995-96 C. Thomas Christensen and Thomas Schelling
32. Who among the following had started two weeklies, D. Mahatma Gandhi and Jawaharlal Nehru
the Karmayogin and the Dharma? 39. Morgenthau’s classification of ideologies doesn’t
A. Mahatma Gandhi B. Aurobindo include:
C. Ranade D. Tilak A. Ideology of status quo
33. Which one of the following is not a feature of ideology? B. Ideology of imperialism
A. A political belief system C. Ambiguous ideologies
B. An action-oriented set of political ideas D. Ideology of hegemony
C. The world view of a particular social class or 40. Which one of the following is not part of five global
social group commons?
4 Sample Paper

A. The High Seas Directions (Qs. 46-50): Read the passage and answer the
B. The Climate System questions.
C. The Deep Ocean floor The term “globalisation” primarily refers to economic
D. The Arctic region activities like trade, movement of capital, goods, labour
Directions (Qs. 41-45): Read the passage and answer the and communication system across boundaries facilitating
questions. higher levels of inter-connectedness in the world. These
Post modernist thinkers hold the view that there cannot economic activities have greater impact on socio-political
be objective knowledge of social issues. For the Post sectors nationally and internationally. The increasing strong
Modernist, knowledge is not simply a cognitive factor: presence of non-state actors like MNCs, the NGOs and
knowledge is also normative and political Power and IGOs has questioned the supremacy of state sovereignty.
knowledge are mutually supportive and they directly imply The rich states are benefited due to their superior control
one another. The Post Modernist are critical of classical over the flow of capital, technology and communication
liberals as well as contemporary positivist who believed in system. A profit oriented global market can never think of
objectivity and superiority of science. Through decons- benefit of all people of the world. The North-South divide
truction, the post modernist want to look at every accepted
has not been obliterated.
or settled idea with new lenses.
46. Which one of the following has not limited state
41. Who among the following is not a Post Modernist
thinker? sovereignty?
A. Richard Ashly B. Jenny Adkins A. Multinational Corporations
C. Michel Foucault D. J. Burton B. Inter Governmental Organization
C. Non-Governmental Organisation
42. Which one of the following statement is incorrect D. Anomic Pressure Groups
according to Post Modernist thinkers?
A. Post Moderners challenged the notion that 47. The globalisation has increased the demand for:
knowledge is eternal A. Laissez Faire state B. Welfare state
B. Knowledge is linked to power C. Totalitarian state D. Socialistic state
C. Knowledge leads to progress of world
48. Which one of the following is incorrect?
D. No knowledge is impartial
43. Which of following term is not preferred by Post The state has become weak in shaping the:
Modernist thinkers? A. Flow of capital
A. Post structuralism B. Deconstruction B. International trade and labour
C. Modernism D. Subjectivity C. Finance
D. Collection of revenues
44. According to Post-Modernist thinker, which attribute
of knowledge does not exist? 49. Globalisation has induces greater:
A. Normative B. Cognitive A. Urbanisation B. Tradition
C. Political D. Objectivity C. Rural societies D. Parochial culture
45. Which of the following is Post Modernist thinker? 50. Globalisation has benefited the countries of:
A. Jacques Derrida B. Immanuel Kant A. Europe and America B. Asia
C. H.J. Laski D. Karl Deutsch C. Africa D. Latin America

ANSWERS
1 2 3 4 5 6 7 8 9 10
D C A C D B B A A A
11 12 13 14 15 16 17 18 19 20
A C D A A D A C D B
21 22 23 24 25 26 27 28 29 30
B C D D A D C A A B
31 32 33 34 35 36 37 38 39 40
A B D D A C D B D D
41 42 43 44 45 46 47 48 49 50
D C C D A D A D A A
POLITICAL SCIENCE

1 (2751) Pol. Sci.—1


2 Political Science
Political Science 3

Political Theory

1. Who among the following defines political science as 10. Which of the following views believes that the state
the study of “the act of human and social control” or originated with class division and class struggle in
the “study of control relationship of wills”? society?
A. Robson B. Pollock A. Liberal view B. Marxist view
C. Maitland D. Catlin C. Pluralist view D. Traditional view
2. ______ is the parent science of all the social sciences. 11. “It is not the consciousness of man which determines
A. Sociology B. History the material conditions of life but it is material
C. Philosophy D. None of the above conditions of life which determine their consciousness”
3. Which of the following deals with morality and – this statement is given by:
formulates rules which should influence the behaviour A. Gettell B. Karl Marx
of man while living in society? C. Galbraith D. Catlin
A. Philosophy B. Ethics 12. “Politics is that part of social science which treats of
C. Sociology D. None of the above the foundations of state and the principles of
4. Ethics is concerned with man as a man and as such, government”. Who made this statement?
it is prior to: A. Willoughby B. Seeley
A. Society B. Science C. Paul Janet D. S.L. Wasby
C. Political Science D. History 13. Guild and Palmer strongly pleaded that the subject of
5. Who among the following prominent writers has given politics should be:
psychological explanations of almost all the political A. The political parties
problems? B. To secure obedience
C. Negative
A. Le Bon B. Baldwin
D. Power instead of the state
C. Graham Wallas D. All of the above
6. Political scientists have borrowed such ideas as 14. The Father of modern sociology, A. Comte, portrayed
society as a potentially harmonious and ordered
‘cultural relativism’, ‘social evolution’, ‘cultural
structure in which all social classes
diffusion’ from:
A. Anthropology B. Sociology A. worked for social change
C. Philosophy D. Economics B. worked for the common good
C. worked for the individual
7. Who among the following said that politics has its D. worked for capitalists
roots, psychologically, in the study of mental habits
15. Socialist states are based on marxian ideology and are
and vocational activities of mankind?
known as the:
A. Barker B. Plato
A. Blue world B. Green world
C. Bryce D. Wallas
C. Red world D. Yellow world
8. Ivor Brown points out that economics deals mainly
16. Who among the following considers Marxism to be an
with _____ and politics is concerned with _____ .
ideology rather than a political theory?
A. wealth, government
A. Germino B. David Held
B. business, state
C. R.A. Dahl D. G.H. Sabine
C. commodities, human beings
D. production, administration 17. Arrange the following works of Isaiah Berlin in
chronological order of his publications.
9. Which of the following theories of the origin of the
state was accepted by liberalism? Select the correct answer from the codes given below:
A. Pluralist theory B. Marxist theory (a) Concepts and Categories : Philosophical Essays
C. Historical theory D. Institutional theory (b) Four Essays on Liberty
3
4 Political Science

(c) Vico and Herder : Two Studies in the History of Select the correct answer from the codes given
Ideas below:
(d) Karl Marx : His Life and Environment A. (a) and (b) B. (b) and (c)
C. (c) only D. (d) only
Codes:
A. (a), (b), (c) and (d) B. (c), (b), (a) and (d) 24. For J.S. Mill, which of the following statements is not
C. (d), (b), (c) and (a) D. (b), (d), (a) and (c) true?
(a) Pleasures differ in quality as well as in quantity.
18. Who among the following said that political theory
(b) Mill makes use of the non-utilitarian arguments.
stands for an abstract model of the political order?
(c) The felicific calculus is absurd.
A. T.P. Jenkin B. R.A. Dahl
(d) Pleasures can be objectively measured.
C. W.T. Bluhm D. Weinstein
Select the correct answer from the codes given below:
19. Which one of the following statements is not true?
A. (b) and (c) B. (a) and (b)
A. Classical political theory is qualitative.
C. (a), (b) and (c) D. (d) only
B. Behavioural political theory is quantitative.
C. Post-behavioural political theory is both 25. Match the following:
qualitative and quantitative. List-I List-II
D. Behavioural theory is not specially concerned (a) Modern political thinker (i) Plato
with Anglo-American model. (b) Positivist school (ii) Machiavelli
(c) Normative political theory (iii) August Comte
20. Which one of the following statements is not true?
(d) Scientific/inductive (iv) Aristotle
A. Neo-liberalism refers to significant revival of
method
interest in classical liberalism.
B. Neo-liberal ideas were taken up by the political Codes:
parties of the New Right in Britain. (a) (b) (c) (d)
C. The revival of classical liberalism occurred as a A. (ii) (iii) (i) (iv)
reaction against the onset of a world recession in B. (ii) (iv) (i) (iii)
the 1970s. C. (i) (ii) (iv) (iii)
D. T.H. Green accepted the classical liberal notion of D. (iv) (i) (iii) (ii)
liberty only. 26. Match the items of List-I with items of List-II by
21. Who among the following said that ideology should selecting the correct answer from the codes :
neither be thought of as liberating or oppressive, nor List-I List-II
as true or false. It can be any these things? (Thinkers) (Theories)
A. Andrew Heywood (a) Mao Tse Tung (i) Proletarian Dictatorship
B. John Rawls (b) Lenin (ii) Cultural Revolution
C. Andrew Gamble (c) Marx (iii) One Country Socialism
D. Bill Coxall (d) Stalin (iv) Communist Party as the
Vanguard of the proletariat
22. For Locke, which of the following statements is not
true? Codes:
(a) The state must be a constitutional state. (a) (b) (c) (d)
(b) Government must possess discretionary power. A. (ii) (iv) (i) (iii)
(c) It is a tolerant state. B. (iii) (i) (iv) (ii)
(d) The right to property is not a natural right. C. (iv) (ii) (i) (iii)
D. (ii) (iv) (iii) (i)
Select the correct answer from the codes given below:
A. (a) and (c) B. (a) and (b) 27. Rearrange the following works of Karl Marx in
C. (b) only D. (d) only sequential order in terms of Publication Year.
(a) Critique of Political Economy
23. For Rousseau, which of the following statements is (b) Poverty of Philosophy
not true? (c) Communist Manifesto
(a) Two original instincts i.e., self-love and sympathy (d) The Holy Family
make up man’s nature.
(b) A thinking man is a depraved animal. Codes:
(c) Family is the only natural society. A. (d), (b), (c) and (a) B. (a), (c), (d) and (b)
(d) General will is not a group mind. C. (b), (a), (d) and (c) D. (d), (c), (b) and (a)
Political Science 5
28. Communitarian theory lays emphasis on: 33. Given below are two statements one labelled as
A. Community as an inescapable ideal Assertion (A) and the other labelled as Reason (R).
B. Unlimited freedom of the individual Assertion (A) : For John Locke, Natural Rights are a
C. Limited role of the state product of nature.
D. Partial regulation of market economy Reason (R) : State has to protect and safeguard
29. Given below are the two statements, one labelled as these rights.
Assertion (A) and the other labelled as Reason (R). A. Both (A) and (R) are true and (R) is the right
Select the correct answer from the codes given below. explanation of (A)
Assertion (A) : Behavioural political theory sought B. Both (A) and (R) are true, but (R) is not the right
to eliminate the role of values and explanation of (A)
make Political Science a pure science C. (A) is true, but (R) is false
and quantitative. D. (A) is false, but (R) is true
Reason (R) : Political Science, being a social science, 34. John Locke’s contribution chiefly lies in his:
can never be value-free as it deals with (a) Consent theory.
human beings who are value-bound. (b) Limited Government.
A. Both (A) and (R) are true and (R) is the correct (c) Theory of state as a corporate body.
explanation of (A). (d) Organic conception of state.
B. Both (A) and (R) are true and (R) is not the correct
explanation of (A). Select the correct answer from the codes given below:
C. (A) is true, but (R) is false. A. (a) and (d)
D. (A) is false, but (R) is true. B. (a) and (c)
C. (b) and (c)
30. Which of the following were described by David Easton D. (a) and (b)
as intellectual foundations of the System Analysis?
Give the correct answer from the codes given below: 35. Dialectical materialism of Karl Marx postulates that:
(a) Regularities (b) Verification (a) Matter is in a state of motion.
(c) Techniques (d) Quantification (b) Motion in the matter is due to the pressure of
environment.
Codes: (c) Law of transformation is both quantitative and
A. (a) B. (a), (b) qualitative.
C. (a), (b), (c) D. (a), (b), (c), (d) (d) Conflict in matter is continuous and endless.
31. Who among the following referred to the ‘decline’ of Identify the correct answer:
political theory? A. (a), (b) and (d) B. (a) and (c)
(a) David Easton and Alfred Cobbon C. (b) and (d) D. (c) and (d)
(b) Lasslett and Robert Dahl
(c) Will Durant 36. Which of the following statement is correct regarding
(d) Nathenial Hawthorn J.S. Mill?
A. (b) and (c) B. (c) and (d) (a) State is justified in confirming itself to limited
C. (a) and (c) D. (a) and (b) functions
(b) Welfare policies are more important.
32. Which of the following statements relating to Mao’s
(c) State intervention into ‘self-regarding’ action.
theory of contradictions are true?
(d) State intervention into ‘other regarding action’ is
(a) Contradictions are universal in all times and in
justified.
all societies.
(b) Antagonistic contradictions exist among the Choose the correct answer:
hostile classes in terms of ownership of productive A. (b) and (c) B. (a) and (b)
forces. C. (a) only D. (d) only
(c) Non-antagonistic contradictions could exist 37. The Magna Carta, the Glorious Revolution and the
among people which could be resolved without writings of John Locke all contributed to the
resorting to violence. strengthening of Great Britain:
(d) Contradictions have no place in a socialist system. A. Absolute Monarchy
Choose the correct answer from the below: B. Ethnic Rivalaries
A. (d) and (a) B. (b) and (d) C. Parliamentary Democracy
C. (a), (b) and (c) D. (a), (b), (c) and (d) D. Imperialist Policies
6 Political Science

38. Which one of the following is not a factor, for Fuku- 45. Who of the following said that felicity is “continued
yama, leading to the triumph of liberal democracy? success in obtaining those things which a man from
A. The struggle for ‘recognition’. time to time desires”?
B. The logic of science’s mastery over nature. A. Bentham B. J.S. Mill
C. The absence of major contradictions in liberal C. Hobbes D. Rousseau
democracy. 46. Which of the following is not an idea of Rousseau?
D. The triumph of liberalism is more in terms of A. A thinking man is a depraved animal
economics than ideologies. B. Science is the fruit of idle curiosity
39. Who among the following said, “Post-behaviouralism C. Philosophy is mere intellectual frippery
was a genuine revolution, not a reaction; a becoming, D. General will is the sum total of wills
not a preservation; a reform, not a counter 47. For J.S. Mill which one of the following is not true?
reformation”? A. Pleasures differ in quality
A. James Bryce B. Charles E. Merriam B. The felicific calculus is absurd
C. David Easton D. G.A. Almond C. The principle of utility is the final end of life
40. Who among the following said, ‘We can not shed our D. Liberty consists in doing what one desires
values in the way we remove our coats’? 48. For Karl Marx, who among the following is not a
A. Leo Strauss B. Alfred Cobban Utopian socialist?
C. Jean Blondel D. David Easton A. St. Simon B. Proudhon
41. Who among the following characterised early C. Fourier D. Engels
liberalism as ‘possessive individualism’? 49. Given below are two statements, one labelled as
A. F.A. Hayek B. Robert Nozick Assertion (A) and other labelled as Reason (R). Select
C. C.B. MacPherson D. Rawls the correct answer from the codes given below:
42. Given below are two statements, one labelled as Assertion (A) : For Gandhiji, Satyagraha is not
Assertion (A) and the other labelled as Reason (R). identical to passive resistance.
Identify the correct answer. Reason (R) : Satyagraha denoted a legitimate,
Assertion (A) : F.A. Hayek portrayed state inter- moral and truthful form of political
vention and collectivism, even in their action by the people against the
moderate forms, as inevitably leading Brutal state power.
to an erosion of Liberty. Codes:
Reason (R) : He is a supporter of laissez faire and A. Both (A) and (R) are true and (R) is the correct
opponent of Keynesian economics explanation of (A)
and the welfare state. B. Both (A) and (R) are true, but (R) is not the
correct explanation of (A)
Codes:
C. (A) is true, but (R) is false
A. Both (A) and (R) are true and (R) is the correct
D. (A) is false, but (R) is true
explanation of (A).
B. Both (A) and (R) are true, but (R) is not the 50. Who coined the slogan “No control, no co-operation”?
correct explanation of (A) A. Mahatma Gandhi B. J.L. Nehru
C. (A) is true, but (R) is false C. Aurobindo D. Subhash Chandra Bose
D. (A) is false, but (R) is true 51. Who among the following stated that, “Every State is
43. Which one of the following statements is not true? known by the rights that it maintains”?
A. Marx drew distinction between ‘ideology’ and A. Harold J. Laski B. T.H. Green
‘science’. C. Immanuel Kant D. Tom Paine
B. N apoleon denounced ideology as cloudy 52. The chief exponent of Scientific Socialism is/are?
metaphysics that ignored history and reality. A. Karl Marx
C. de Tracy coined the term ideology. B. Fredrick Engels
D. Daniel Bell talked of ‘ideology without an end’. C. Both A & B
44. For whom, “ideas” and not the “material conditions of D. Neither A & B
production” are the effective causes of revolution? 53. Who is the author of Grammar of Politics?
A. Karl Marx B. Lenin A. Mill B. Rousseau
C. Engels D. Stalin C. Lasswell D. Laski
Political Science 7
54. “State is known by the rights that it maintains” who 65. Who among the following identifies power with
said? domination in conceptual, methodological and
A. Russel B. Woodrow Wilson political terms?
C. Janet D. Laski A. Michael Foucault B. Steven Lukes
55. Who is the author of Proposed Roads to Freedom? C. Edward Banfield D. V. Pareto
A. Russel B. Tolstoy 66. Two conclusions emerge from the power perspective:
C. Bryce D. Leacock the one who exercises the power had the option or the
56. Maine authored the book alternative to act differently; and those on whom the
A. The History of Institutions power had the option or the alternative to act
B. Democracy and Liberty differently, if power was not exercised over them-The
C. Social Contract statement is made by who among the following?
D. Political Science and Government A. Robert Dahl B. Delbert Miller
C. Steven Lukes D. Edward Banfield
57. Who defined “Liberty is the opposite of over
government”? 67. Social Citizenship refers to
A. Seeley B. J.S. Mill A. right to participate in an appropriate standard of
C. Gramsci D. Marx living
B. right to participate in the exercise of political
58. Which is known as the birth place of democracy?
power
A. Athens B. America
C. more than one country and culture
C. India D. Switzerland
D. the police and defence forces are coercive
59. Direct democracy is now practice in structures of the state
A. Cantons of Switzerland
68. Multiculturalism aims at:
B. States of India
A. accommodating diverse identity groups into a
C. States of America
homogeneous society
D. England
B. state has been conflated with nation in their
60. Who among the following analyses liberty as a triadic conceptualization
relationship in the defined manner-X is free from Y to C. autonomy and self-governing rights
do or become (or not to do or become) Z? D. equality before the law
A. Gerald Mac Callum B. Milton Friedman
69. Match List-I of Authors with List-II of Books:
C. Robert Nozick D. Isaiah Berlin
List-I List-II
61. Who among the following discussed two types of (Authors) (Books)
liberty as Opportunity Concept of Freedom (Negative (a) T.H. Marshall (i) Citizenship and Social
Liberty) Exercise Concept of Freedom (Positive Liberty)? Class (1950)
A. Robert Nozick B. Milton Friedman (b) R. Bendix (ii) N ation-Building and
C. Charles Taylor D. Harold Laski Citizenship (1964)
62. The book “The Constitution of Liberty” (1960) is (c) Robert Nozick (iii) A narchy, State, and
written by who among the following? Utopia (1974)
A. Isaiah Berlin B. Steven Lukes (d) Willy Kymlicka (iv) M ulticultural Citizen-
C. Ronald Dworkin D. F.A. Hayek ship: A Liberal Theory of
Minority (1997)
63. Social primary goods according to John Rawls refers to:
A. goods are distributed by the basic structure of a Select the correct code the options given below:
society (a) (b) (c) (d)
B. include rights and liberties, pow ers and A. (iv) (i) (ii) (iii)
opportunities, and income and wealth B. (iii) (iv) (i) (ii)
C. distribution of “social primary goods” to all the C. (ii) (iii) (iv) (i)
members of society in a fair or just manner D. (i) (ii) (iii) (iv)
D. All the above 70. The concept of multiculturalism was prevalent since
64. “Original position” is under a “Veil of ignorance” is A. ancient times
a concept developed by whom among the following? B. medieval times
A. Robert. A. Dahl B. Gramsci C. modern times
C. A.B. Hall D. John Rawls D. post-modern times
8 Political Science

71. Will Kymlicka argues for C. culture is very closely linked to nationalism
A. group differentiated rights D. resurgence of people’s movements against the
B. racism and ethnic cleansing totalitarian
C. collapse of the European colonial system
76. Capitalism is the primary cause of environmental
D. multicultural in a descriptive sense
degradation-who said this?
72. The book “Reflections on the Revolution in France” A. Habermas B. Medha Patkar
was written by whom among the following C. Arundhoti Ray D. Anna Hazare
conservative thinker?
77. The Dialectic of Sex: The Case for Feminist Revolution
A. Edmund Burke (1993)
(1970) is a work of
B. Michael Oakeshot (1962)
A. Shulamith Firestone B. Roxanne Dunbar
C. James Alexander (2014)
C. Naomi Weisstein D. Judith Brown
D. Andrew Heywood (2007)
73. Who among the following has written on “Citizenship 78. Radical feminists movement was started in which of
the following countries?
and Social Class”?
A. Marshall McLuhan B. T.H. Marshall A. United States B. United Kingdom
C. Manuel Castells D. Anthoni Giddens C. Australia D. All A, B & C

74. The word “citizen” was made popular by which of the 79. Nation-Building and Citizenship (1964) is a book
following revolution of the world written by who among the following political
A. French Revolution in 1789 philosopher?
B. Glorious Revolution, 1689 A. R. Bendix B. Bryan S. Turner
C. American War of Independence, 1776 C. Jack Barbalet D. Charles Taylor
D. None of these 80. Burke’s ‘Reflections on the Revolution in France’ has
75. Why “Melting Pot” in United States is famous? been taken as definitive and fortunate of:
A. all the imm igrant cultures are m ixed and A. Traditional Conservatism
amalgamated without state intervention B. Medieval Conservatism
B. not to facilitate immigrants and others to preserve C. Modern Conservatism
their cultures D. None of these

ANSWERS
1 2 3 4 5 6 7 8 9 10
D A B C D A C C C B
11 12 13 14 15 16 17 18 19 20
B C D B C A C C D D
21 22 23 24 25 26 27 28 29 30
A D D D A A A A B D
31 32 33 34 35 36 37 38 39 40
D C A D A D C D C D
41 42 43 44 45 46 47 48 49 50
C A D B C D C D A C
51 52 53 54 55 56 57 58 59 60
A A D D A A A A A A
61 62 63 64 65 66 67 68 69 70
C D D D A D A A D A
71 72 73 74 75 76 77 78 79 80
A A B A A A A D A C
  
Political Science 9

Political Institutions in India

1. A special session of the Constituent Assembly was 9. Protection of Civil Rights Act, 1955 pertains to:
held at midnight on 14-15 August, 1947 in connection A. Human Rights B. Citizenship
with the: C. Untouchability D. Freedom of expression
A. Independence resolution
10. According to the Constituent Assembly Debates
B. Partition of India
Directive principles is merely another name for the:
C. Transfer of Power
A. Bill of Rights
D. None of the above
B. Human Rights Character
2. Which committee was appointed to prepare guidelines C. Instruments of Instructions
for the Constituent Assembly? D. Governmental Accountability
A. Drafting Committee
B. Congress Expert Committee 11. Which Article of the Constitution states that the state
C. Advisory Committee shall organise village panchayats as units of self-
D. Union Powers Committee government?
A. Article 43 B. Article 40
3. Which committee was set up to prepare a draft
C. Article 45 D. Article 42
constitution on August 29, 1947?
A. Staff Committee 12. The case of Gopalan vs. State of Madras was relevant
B. Union Constitution Committee to:
C. Drafting Committee A. Article 21 B. Article 12
D. Union Powers Committee C. Article 16 D. Article 14
4. The elections to the Constituent Assembly were held 13. How can a citizen protect his Fundamental Rights?
in: A. By a writ in the Supreme Court of India
A. June, 1946 B. July, 1946 B. By approaching the President of India
C. August, 1946 D. July, 1947 C. Through police action
5. Which of the following is included in the Committees D. They are already protected
on procedural Affairs? 14. Which of the following is not a Fundamental Right?
A. Training and Staff Committee A. Right against exploitation
B. Credentials Committee B. Equal pay for equal work
C. Hindi Translation Committee C. Equality before law
D. All of the above D. Right to freedom of religion
6. The elections to the Constituent Assembly were:
15. An interpretation of the Indian Constitution is based
A. Indirect B. Held Twice
on the spirit of the:
C. Direct D. None of the above
A. Preamble B. Fundamental Duties
7. Fundamental duties have been introduced in the C. Fundamental Rights D. None of the above
Constitution by:
16. Right to property has been eliminated from the
A. 42nd Amendment Act, 1976
B. 44th Amendment Act, 1978 fundamental rights by:
C. 73rd Amendment Act, 1993 A. 42nd Constitutional Amendment Act of 1976
D. None of the above B. 44th Constitutional Amendment Act of 1978
C. 43rd Constitutional Amendment Act of 1977
8. The Dominion Status of India was established under: D. 24th Constitutional Amendment Act of 1971
A. Government of India Act, 1919
B. Government of India Act, 1935 17. Fundamental duties enjoin upon a person to respect:
C. Both A and B A. Mother Tongue B. National Flag
D. Indian Independence Act, 1947 C. National Anthem D. Both B and C
9
(2751) Pol. Sci.—2
10 Political Science

18. It is a Fundamental Duty of a citizen of India to 25. Which of the following statements are incorrect?
develop: 1. Jnanpith Award is the most prestigious literary
A. Her or his economic status and social standing honour in the country along with Sahitya Akademi
B. The understanding between different communities Award.
C. Scientific temper and spirit of enquiry 2. It is given for a Magnum opus produced by a
D. All of the above writer.
3. Any writer who writes in any of the official
19. Special provision for the protection of children is
languages of India is eligible for this honour.
made in:
A. 1, 2 B. 2, 3
A. Article 23 B. Article 24
C. 1, 3 D. All the above
C. Article 39(b) D. All of these
26. Which of the following statements are true?
20. ‘Equality before the law’, is an expression of: 1. Nai Roshni is a scheme for Leadership Develop-
A. English Common law ment of Minority Women launched in 2012-13.
B. French Administrative law 2. The scheme would provide minority women
C. American Constitutional law support, leadership training, information and
D. None of the above confidence to interact with government system,
21. Which of the following statements are true? banks and intermediaries at all levels.
1. The Home Minister is the defacto chairperson of 3. The scheme is implemented through Panchayati
NDMA. Raj and NGOs.
2. NDMA is an agency of the Ministry of Home A. 1, 2 B. 2, 3
Affairs. C. 1, 3 D. All the above
3. It is a statutory body established by Disaster 27. Which of the following statements are correct in context
Management Act enacted by the Government of with Transgender Bill?
India in December, 2005. 1. The Bill aims at formulation and implementation
A. 1, 2 B. 2, 3 of a comprehensive national policy for the
C. 1, 3 D. All the above upliftment of transgenders.
22. Which of the following statements is/are true? 2. A National Transgender Welfare Commission and
1. The Finance Act gives effect to financial proposals a special transgender court will be set up.
at the beginning of every Financial Year. 3. The bill demands reservation for the community
in education, financial assistance and social
2. The Finance Act for a particular financial year
inclusion.
also includes the amendments that have been made
4. Tamil Nadu was the first State to constitute a
with respect to Direct Taxes.
welfare board for the transgender community in
A. Only 1 B. Only 2
the country.
C. Both 1 and 2 D. None of these
A. 1, 2, 3 B. 2, 3, 4
23. Which of the following statements are true? C. 1, 3, 4 D. All the above
1. The Rajya Sabha may amend money bills.
28. Which of the following statements is/are correct?
2. Money bill must be returned to the Lok Sabha
1. The 73rd Constitutional Amendment Act provides
within 14 days or the bill is deemed to have
recognition to the Urban Local Bodies as the
passed by the both houses in the form it was third tier of the Government in urban areas.
originally passed by the Lok Sabha. 2. State Election Commission has a responsibility to
3. Money bill cannot be returned by the President to conduct elections for urban local bodies.
the Parliament for its reconsideration, he has to A. Only 1 B. Only 2
give his assent to the bill or can withhold his C. Both 1 and 2 D. None of these
assent.
A. 1, 2 B. 2, 3 29. Which of the following statements is/are correct?
C. 1, 3 D. All the above 1. Competition Commission of India is a consti-
tutional body.
24. Who certifies a bill as a money bill? 2. Being a quasi-judicial body so the appointment
A. President of the Chairman is nominated by higher Judicial
B. Prime Minister body or the Chief Justice of India.
C. Speaker of the Lok Sabha A. Only 1 B. Only 2
D. Finance Minister C. Both 1 and 2 D. None of these
Political Science 11
30. Which of the following statements related to NJAC is/ 2. The Commission considers inclusions in and
are correct? exclusions from the lists of communities notified
1. NJAC will replace the collegium system for the as backward.
appointment of judges as mandated in the existing A. Only 1 B. Only 2
preamended constitution by a new system. C. Both 1 and 2 D. None of these
2. NJAC will be a statutory body. 36. Which of the following statements is/are correct?
3. It will give the executive equal role in appoint-
1. UNESCO’s global education report says that India
ments.
has reduced its out of school children by over
A. 1 and 2 B. 1 and 3 90% and has achieved universal prim ary
C. 2 and 3 D. All of the above
education.
31. Which of the following statements are correct? 2. India is the only country in South and West Asia
1. “Tourist Visa on Arrival-Electronic Travel to have an equal ratio of girls to boys in both
Authorization (TVoA-ETA)” scheme is renamed primary and secondary education.
as e-Tourist Visa. A. Only 1 B. Only 2
2. A tourist visiting India had to obtain the e-Visa C. Both 1 and 2 D. None of these
before departing from his country. 37. Which of the following statements are incorrect?
3. The validity of the Visa is for 30 days from the
1. National Disaster Response Force works under
date-of-arrival.
Ministry of Agriculture.
A. 1 and 2 B. 1 and 3 2. States have to keep 25 per cent of the State Disaster
C. 2 and 3 D. All of the above
Response Fund (SDRF) reserved for local disasters
32. Which of the following are done under Swaccha Bharat such as heavy rain.
Abhiyan? 3. Centre links disaster compensation to farmers with
A. Construction of individual sanitary latrines for annual inflation derived from the Consumer Price
households below the poverty line Index (CPI).
B. Conversion of dry latrines into low-cost sanitary A. 1 and 3 B. 1 and 2
latrines C. 2 and 3 D. All of the above
C. Setting up of sanitary marts
38. Which of the following statements are correct?
D. All of the above
1. The National Food Securities Act (NFSA) passed
33. Which of the following statements related to Bibek in 2013 has been implemented in only 11 states
Debroy Committee recommendations are correct? and union territories.
1. Private entry into running both freight and 2. The government extended the deadline by 6
passenger trains. months for remaining states to implement NFSA.
2. To progressively phase out the practice of a 3. The law aims to provide legal entitlements to 5
separate railway budget. kg of subsidized food-grains per person a month
3. To ensure increases in passenger fares. at ` 1-3/kg.
4. Establishment of an independent body to provide A. 1 and 2 B. 1 and 3
a level-playing field to private players. C. 2 and 3 D. All of the above
A. 1, 2 and 3 B. 2, 3 and 4
39. Which of the following statements are correct?
C. 1, 3 and 4 D. All of the above 1. Rajasthan state government recently tabled
34. Which of the following statements is/are correct? Prevention of Witch-Hunting Bill in the Assembly.
1. The Union Home Ministry has notified a new set 2. Bill provides for life imprisonment if Witch-
of rules to regulate official visits of Governors Hunting causes death, and imprisonment up to
outside their States. five years and a fine.
2. Now governors need prior permission from the A. Only 1 B. Only 2
President and putting a cap of 73 days in a year C. Both 1 and 2 D. None of these
as duration of such visits. 40. Which among the following is NOT a Standing
A. Only 1 B. Only 2
Committee?
C. Both 1 and 2 D. None of these
A. Public Accounts Committee
35. Which of the following statements is/are correct? B. Ethics Committee
1. National Commission for Backward Classes is an C. Railway Convention Committee
Indian statutory body established in 1993. D. Business Advisory Committee
12 Political Science

41. Consider the following statements. Which among them 45. Consider the following statements about the Public
is/are NOT true? Undertakings Committee. Which among them is/are
1. A parliamentary committee is appointed or elected not true?
by either Lok Sabha or Rajya Sabha or nominated 1. Public Undertakings Committee was first created
by the Speaker of Lok Sabha or Chairman of in 1964.
Rajya Sabha. 2. It consists of 15 members from Lok Sabha and 7
2. A parliamentary committee works under the members from Rajya Sabha.
direction of the Speaker of Lok Sabha or the 3. Its Chairman is strictly a member of the Lok Sabha.
Chairman of Rajya Sabha. 4. The main function of the Public Undertakings
3. A parliamentary committee is one which presents Committee is to analyze the accounts of Public
its report to either Lok Sabha or Rajya Sabha or Undertakings and examine reports of CAG on
either to the Speaker or Chairman. public undertakings.
4. A parliamentary committee has a secretariat A. Only 1, 2 and 3 B. Only 2 and 4
provided by the Lok Sabha or Rajya Sabha. C. Only 3 and 4 D. All are true
A. Only 1 and 3
B. Only 2 and 4 46. Consider the following statements about the Ethics
C. Only 1, 2 and 3 Committee. Which among the following is/are NOT
D. All are true true about it?
1. Ethics Committee overlooks the discipline and
42. Consider the following statements. Which among them decorum in the Parliament.
is/are NOT true? 2. It was constituted in Rajya Sabha in 1997, and in
1. Standing Com m ittees are the permanent Lok Sabha in 2000.
committees that are constituted on a regular basis. 3. Ethics Committee is an ad-hoc committee.
2. Ad hoc committees are temporary committees that
4. Ethics Committee is involved in making sure that
are dissolved when the task is completed.
everyone conforms with the code of conduct.
3. Committee of Privileges is an ad-hoc committee.
A. Only 1 B. Only 2
4. Joint Committee on Fertilizer pricing is a standing
C. Only 3 D. All are true
committee.
A. Only 3 and 4 B. Only 1 and 2 47. Consider the following statem ents about the
C. Only 1 and 3 D. Only 2 and 4 Committee on Government Assurances. Which among
them is/are not true?
43. What among the following is NOT true about the
1. It was constituted in 1953.
Public Accounts Committee?
1. The Committee was first set up in 1921. 2. Its main function is to examine the assurances
2. Its main function is to audit the annual reports of and undertakings given by the ministers on the
Comptroller and Auditor General of India (CAG). floor of the House and report back on the status
3. It consists of 15 Lok Sabha members and 7 Rajya of these assurances and promises.
Sabha members. 3. For the Lok Sabha, it consists of 15 members.
4. The term of the office of the Public Accounts 4. For the Rajya Sabha, it consists of 10 members.
Committee is one year. A. Only 1 B. Only 2
A. Only 1 B. Only 2 C. Only 3 D. All are true
C. Only 3 D. All are true 48. When was the first Parliamentary Forum on Water
44. Which among the following is NOT true about the Conservation and Management constituted?
Estimates Committee? A. 1950 B. 1970
1. The first Estimates Committee of the post- C. 2005 D. 1985
independence era was first set up in 1950. 49. When was the first Parliamentary Forum on Youth
2. Estimates Committee has a right to question the constituted?
policies approved by the Parliament. A. 2010 B. 2008
3. It consists of members that are both from Lok C. 2006 D. 1985
Sabha and Rajya Sabha.
4. Chairman of the Estimates Committee is always 50. When was the first Parliamentary Forum on Global
from the Opposition party. Warming and Climate Change constituted?
A. Only 4 B. Only 3 A. 2005 B. 2006
C. Only 2 and 4 D. Only 2, 3 and 4 C. 2007 D. 2008
Political Science 13
51. When was the first Parliamentary Forum on Disaster 3. He allocates and reshuffles the portfolios among
Management constituted? the state ministers.
A. 2011 B. 2010 4. His salary and allowances are determined by the
C. 2009 D. 2008 state legislature.
52. When was the first Parliamentary Forum on Population A. Only 1 B. Only 2
and Public Health constituted? C. Only 3 D. All are true
A. 2007 B. 2006 58. Article 17 of the Indian Constitution provides for:
C. 2005 D. 2004 A. Abolition of titles
53. When was the first Parliamentary Forum on Children B. Equality before law
constituted? C. Abolition of untouchability
A. 2006 B. 2007 D. Equality of opportunity in matters of public
C. 2008 D. 2009 employment

54. Which among the following is/are not a part of the 59. Article 370 of the Constitution of India was for:
State Executive? A. Temporary provisions for Jammu and Kashmir
1. Governor B. Special provisions in respect of Nagaland
2. Chief Minister C. Provisions in respect of the Financial Emergency
3. Council of Ministers (state) D. None of these
4. Advocate General 60. Who among the following was the President of the
A. Only 1 B. Only 4 Constituent Assembly of India?
C. Only 1 and 4 D. All the above A. M.A. Jinnah B. Dr. Rajendra Prasad
55. Which among the following statement is/are not true C. Jawaharlal Nehru D. Lal Bahadur Shastri
about the conditions of a Governor’s office? 61. The majority of the provisions of Indian Constitution
1. A Governor can simultaneously function as a can be amended:
member of Lok Sabha or Rajya Sabha or State A. By the Parliament alone
Legislature while discharging his duties as a B. By the State Legislatures acting together
Governor of other state. C. Only on ratification by half of the States
2. The Governor should not hold any office of profit. D. With the joint approval of the Parliament and
3. The same person can be appointed as a Governor State Legislatures
of 2 or more states.
62. A proclamation of emergency, under Article 352, on
4. The Governor has personal immunity from any
account of war or aggression requires approval of the
liabilities regarding his official acts.
Parliament within:
A. Only 1 B. Only 2
A. One month B. Two months
C. Only 3 D. All are true
C. Four months D. Six months
56. Which among the following statements is/are not true
63. Which Constitutional Amendment gave precedence to
about the executive powers of the Governor?
the Directive Principles of State Policy over
1. The Governor appoints the Vice-Chancellor of
Fundamental Rights?
the universities of the state.
A. 42nd B. 44th
2. The Governor acts as the Chancellor of the
C. 52nd D. 56th
universities of the state.
3. The Governor appoints the Chief minister and 64. Which of the following expressions does not figure in
other state ministers. the Preamble to the Indian Constitution?
4. The Governor appoints the advocate general of A. Secular
the state. B. Federal
A. Only 1 B. Only 2 C. Socialist
C. Only 4 D. All are true D. Sovereign Democratic Republic

57. Which among the following statements concerning 65. The basic features of the Indian Constitution which
the Chief Minister is/are not true? are not amendable under Article 368 are:
1. His role is analogous to that of the Prime Minister. A. Judicial review and the federal system
2. He can recommend the dissolution of the state B. Sovereignty, territorial integrity, federal system
legislative assembly to the governor. and judicial review
14 Political Science

C. Sovereignty, territorial integrity and parliamentary 74. Who said that “Oh! Disrespectable democracy! I love
system of Government you”?
D. Sovereignty, territorial integrity, federal system A. G.B. Shaw B. Lord Bryce
and judicial review and parliamentary system of C. Carpenter D. Appa Dorai
Government
75. The final authority to interpret our Constitution is
66. ‘Cabinet System’ and ‘Collective Responsibility’ are the:
the contributions of: A. President B. Parliament
A. India B. Britain C. Prime Minister D. Supreme Court
C. Ireland D. USA
76. A secular State is one which:
67. The Creamy Layer, concept refers to: A. Is irreligious
A. The grouping based on social status B. Is anti-religion
B. The grouping based on castes C. Has no religion of its own
C. The grouping based on economic status D. Takes into consideration the religious sentiments
D. The grouping based on milk consumption of the people
68. In the Constitution of India, the ‘Right to Consti- 77. Which among the following article is associated with
tutional Remedies’ has been provided in Article: the amending of the Constitution of India?
A. 30 B. 31 A. Article 443 B. Article 308
C. 32 D. 35 C. Article 368 D. Article 321
69. Which amendment to the constitution provides for the 78. The Constitution of India describes the country as a:
reservation of 1/3 seats in the Municipal Boards and A. Federation B. Union of States
village panchayats for women? C. Confederation D. Unitary State
A. 73rd & 74th Amendments
79. Indian Constitution has divided the powers and
B. 82nd & 83rd Amendments
function of the state into:
C. 72nd & 73rd Amendments
A. Two lists B. Three lists
D. 74th & 75th Amendments
C. Four lists D. Five lists
70. By which constitutional amendment political
80. The concept of Constitution first originated in:
defections were banned?
A. The 50th Amendment of 1984 A. Japan B. USA
B. The 53rd Amendment of 1986 C. Britain D. Switzerland
C. The 54th Amendment of 1986 81. Evaluate the following statements:
D. The 52nd Amendment of 1985 1. The legal interpretation of equality is chiefly
71. The system of Privy Purses in respect of former rulers influenced by equality before law and equal
of Indian states before independence was abolished protection of law.
by the Constitution through: 2. Equality before law means rule of law.
A. 26th Amendment Act, 1971 A. 1 is correct, but 2 is incorrect
B. 27th Amendment Act, 1971 B. 2 is correct, but 1 is incorrect
C. 38th Amendment Act, 1975 C. Both are correct
D. 42nd Amendment Act, 1976 D. Both are incorrect

72. What is meant when the Constitution declares India a 82. Which Articles in the Constitution give provisions for
“Secular State”? the electoral system in our country?
A. Religious worship is not allowed A. Articles 124-128 B. Articles 324-329
B. Religions are patronized by the state C. Articles 256-259 D. Articles 274-279
C. The state regards religions as private affairs of the 83. What is the maximum allowed duration between the
citizen and does not discriminate on this basis last session of the dissolved Lok Sabha and the
D. None of these recalling of the Lok Sabha?
73. Under which Article of the Constitution is the A. 2 months B. 4 months
President’s Rule introduced in a State due to the failure C. 5 months D. 6 months
of the constitutional machinery? 84. Who designated the first Indian voting machine?
A. 352 B. 356 A. S.K. Thorat B. Narendra Jadhare
C. 360 D. 350 C. M.B. Haneefa D. A.G. Rao
Political Science 15
85. Which was the first Indian state to go for Internet A. Rajya Sabha B. Parliament
voting? C. President D. Lok Sabha
A. Gujarat B. Punjab 96. The two words that were inserted by the 42nd
C. Karnataka D. Andhra Pradesh
Amendment to the Preamble to the Constitution are:
86. Which is the largest Lok Sabha constituency in General A. Secular, Democratic B. Sovereign, Democratic
Elections by area? C. Socialist, Secular D. Secular, Republic
A. Ladakh B. Barmer 97. Which among the following statements is/are false?
C. Arunachal West D. Delhi Sadar
1. For qualifying to the Lok Sabha, a person must be
87. Which is the smallest Lok Sabha constituency in at least 25 years of age.
General Elections by area? 2. A person can be a member of both Lok Sabha and
A. Delhi Sadar B. Mumbai South Rajya Sabha at the same time.
C. Kolkata North West D. Chandni Chowk 3. A person belonging to SC or ST can contest
elections from a general seat also, and not just the
88. Which is the largest Lok Sabha constituency in General
Elections by population? reserved seat.
4. If a member of either of the Houses votes against
A. Malkajgiri B. Sasaram
the directions given by his party, he can be deemed
C. Unnao D. Daman & Diu
to be disqualified.
89. Which is the smallest Lok Sabha constituency in A. Only 1 B. Only 2 and 4
General Elections by population? C. Only 3 D. Only 2
A. Buxar B. Lakshadweep
98. Which among the following statements concerning
C. Ghaziabad D. Bangalore North
the Chief Minister is/are not true?
90. The objectives of Indian Planning are: 1. At his time of appointment, the Chief Minister
A. Increasing national income need not be a member of the state legislature.
B. Elimination of poverty 2. Chief Minister must always prove his majority in
C. Reducing inequalities in income and wealth the legislative assembly before his appointment.
D. All of the above 3. It is the governor who does the final appointment
91. The term “Fourth Estate” is used for: of the Chief Minister.
A. Parliament 4. The Chief M inister can recomm end the
B. Judiciary dissolution of the legislative assembly to the
C. The Executive governor.
D. The Press and Newspaper A. Only 1 B. Only 2 and 4
C. Only 2 D. Only 2 and 3
92. Through which Constitutional Amendment was the
Nagarpalika Bill passed? 99. Which of the following roles/functions/responsibilities
A. 70th B. 72nd are not under the purview of the Financial
C. 73rd D. 74th Commission?
1. Sharing of net proceeds of taxes between the
93. Who can legislate on those residual matters which are Centre and the States
not mentioned in Central/State/Concurrent lists? 2. Recommending principles for governing the
A. Parliament alone grants-in-aid to the states by the centre.
B. State legislatures exclusively 3. Recommendations on matters referred to it by the
C. Parliament or State legislatures as adjudicated by President for the interest of sound finance.
the Supreme Court 4. Preparation and periodical revision of electoral
D. Parliament after State legislatures concur rolls.
94. Which schedule of the Constitution deals with the A. Only 1 B. Only 2
disqualification of elected members on the ground of C. Only 3 and 4 D. Only 4
defection? 100. The 14th Finance Commission of India, whose
A. 8th B. 9th chairman is Y.V. Reddy, has allocated what % of
C. 10th D. 11th central tax revenues that were shared by the Centre
95. According to the Article 75(3) of the Constitution of with the States?
India the Council of Ministers are collectively A. 56% B. 50%
responsible to the: C. 70% D. 42%
16 Political Science

101. Chief Minister of a state is the sole channel of B. Bihar, Maharashtra, Karnataka, Uttar Pradesh, and
communication between: Andhra Pradesh
A. Ministers and the Governor C. Rajasthan, Assam, Madhya Pradesh, Uttar Pradesh,
B. Ministers and the Legislature Bihar, West Bengal
C. Both A and B D. Tamil Nadu, Punjab, Kerala and Bihar
D. Governor and the Legislature 110. Which Article of the Constitution confers upon the
102. Whips are maintained by the parties to: Governor of a state the power to make ordinances?
A. Discuss matters of national interest A. Article 123 B. Article 213
B. Discipline the members of legislature belonging C. Article 93 D. None of the above
to respective parties 111. With reference to regionalism, consider the following
C. Act as a precautionary measure in case of an statements:
imminent threat of defection from the party 1. Regionalism is a disintegrating force unlike
D. Both B and C
nationalism which is a unifying force.
103. Which of the following has not been specified in the 2. Regionalism is not conducive with the idea of
constitution? federalism.
A. Size of the Council of Minister A. Only 1 B. Only 2
B. Salaries of the Council of Minister C. Both 1 and 2 D. Neither 1 nor 2
C. Both A and B
112. The election process in India follows the first past the
D. Powers of the Chief Minister
post system. This system offers many advantages like:
104. Which Article of the constitution makes a mention of 1. Providing a proportionate outcome of seats based
the Advocate-General for the state? on votes.
A. Article 157 B. Article 167 2. Less wastage of votes.
C. Article 177 D. Article 187 3. Establishing a clear link between representatives
105. Which Article of the Constitution provides the and constituencies.
procedure for the abolition of the second chamber of Select the correct answer using the codes given below.
the Legislature (Legislative Council) in a state where A. Only 1 and 2 B. Only 1 and 3
it exists as well as for the creation of such a chamber C. Only 3 D. Only 2 and 3
in a state where there is none at present?
113. The major benefits of federalism is/are:
A. Article 139 B. Article 231
1. It helps in quick decision making.
C. Article 169 D. None of these
2. It gives voice to regional interests.
106. Of the total number of members of the Legislative 3. It creates a network of checks and balances.
Council in State what is the proportion of members Select the correct answer using the codes given below.
nominated by the Governor of the State? A. Only 1 and 2 B. Only 1 and 3
A. One-fourth B. One-fifth C. Only 2 D. Only 2 and 3
C. One-sixth D. One-seventh
114. Match List-I with List-II and select the correct answer
107. All decisions of the Council of ministers in a state using the codes given below:
relating to the administration of the state of affairs and List-I List-II
proposals of legislation are communicated to the (Parties) (Trade Unions)
Governor by: (a) Indian National Congress (i) C.I.T.U.
A. Secretary of the Legislative Assembly (b) Bhartiya Janata Party (ii) A.I.T.U.C.
B. Chief Minister (c) Communist Party of India (iii) B.M.S.
C. Advisor to the Chief Minister (d) Communist Party of India (iv) I.N.T.U.C.
D. None of the above (Marxist)
108. Which Article confers upon the Governor the power to (v) B.K.S.
reserve a Bill pertaining to the state for consideration Codes:
of the President? (a) (b) (c) (d)
A. Article 100 B. Article 201 A. (iii) (iv) (ii) (i)
C. Article 111 D. Article 179 B. (iv) (iii) (i) (v)
109. The States having Bicameral Legislatures are: C. (iii) (iv) (i) (v)
A. Bihar, Rajasthan, Uttar Pradesh and Odisha D. (iv) (iii) (ii) (i)
Political Science 17
115. Which of the following statements is/are correct? 123. Which of the following Constitution Amendment Acts
1. The National Integration Council is a recommen- seeks that the size of the Councils of Ministers at the
datory body. centre and in a state must not exceed 15% of the total
2. The National Integration Council is a consti- no. of members in the Lok Sabha and the total no. of
tutional body. members of the Legislative Assembly of that state
A. Only 1 B. Only 2 respectively?
C. Both 1 and 2 D. Neither 1 nor 2 A. 91st B. 93rd
C. 95th D. 97th
116. Communalism in Indian context means:
A. Serving the community in a befitting manner 124. Which of the following statements is/are correct?
B. Using communal identity for political gains 1. The mode of removal of a judge of a High Court
C. A group of people bound by ethnic feelings in India is same as that of removal of a judge of
D. Creating friendly relations w ith the other the Supreme Court.
communities 2. After retirement from the office, a permanent judge
of a High Court cannot plead or act in any court
117. In which of the following years, was the National
or before any authority in India.
Integration Council constituted first?
A. 1950 B. 1956 A. Only 1 B. Only 2
C. Both 1 and 2 D. Neither 1 nor 2
C. 1961 D. 1963
125. Which of the following statements about the state
118. Which of the following states/union territories have a
Governor is not true?
common High Court?
A. UP and Bihar A. He is a part of the state legislature
B. He can pardon a sentence of death
B. Punjab and Jammu & Kashmir
C. He appoints Judges of the State High Court
C. Punjab, Haryana and Chandigarh
D. Assam and Bengal D. He has no emergency powers

119. Panchayati Raj is organised at the: 126. To be officially recognised by the speaker of the Lok
Sabha as an opposition group, a party or coalition of
A. Block level
parties must have at least:
B. Village and Block level
C. Village, Block and District level A. 55 members
B. 60 members
D. Village, Block, District and State level
C. 80 members
120. Which of the following courts is responsible for the D. 1/3 of total members of the Lok Sabha
enforcement of Fundamental Rights?
A. Supreme Court B. High Court 127. The entire concept of community development was
conceived by the:
C. District Court D. Both A and B
A. State Government B. Local Government
121. Which of the following statements is/are correct? C. Union Government D. None of the above
1. In Part IX of the Constitution of India, there is no
provision of reservation for women in any seat to 128. Which of the following committees was appointed by
the government to review the working of the
be filled by direct election in every Panchayat.
community development programme?
2. The 11th schedule of the Constitution of India
distributes power between the state legislature A. Balwant Rai Mehta committee
B. Ashok Mehta committee
and the panchayat just as the 7th schedule
C. Santhanam committee
distributed powers between the centre and the
states. D. None of the above
A. Only 1 B. Only 2 129. Which of the following is the main source of income
C. Both 1 and 2 D. Neither 1 nor 2 of the panchayats?
A. Grants from Government
122. Which of the following statements is/are correct?
1. The Governor of a state may reserve a bill for B. Taxes on buildings
C. Tax on commercial crops
consideration of the President of India.
D. All of the above
2. The Governor of a state is competent to withdraw
an ordinance issued in the state at any time. 130. The Chief Executive of the Panchayat Samiti is the:
A. Only 1 B. Only 2 A. Block Development Officer
C. Both 1 and 2 D. Neither 1 nor 2 B. Chairman
(2751) Pol. Sci.—3
18 Political Science

C. Sarpanch 139. Institutions of self-government, called by a general


D. None of the above name “municipalities”, are:
131. Consider the following features of the 73rd A. Nagar panchayats, for transitional areas, i.e., an
area which is being transformed from a rural area
Constitutional Amendment Act.
1. Panchayati Raj bodies will receive finance from to an urban area
the state government in the form of grants. B. Municipal councils for smaller urban areas
2. One-third seats of the total seats in the Panchayati C. Municipal corporations for larger urban areas
Raj bodies will be reserved for women. D. All of the above
3. Mostly states will have three-tier Panchayati Raj 140. Which of the following states first resorted to
system. reservation of women in Panchayati Raj Institutions?
4. Panchayati Raj bodies will prepare plans for A. Andhra Pradesh
economic development, social justice and social B. Karnataka
welfare. C. Maharashtra
Which of the above are correct? Choose the correct D. All of the above
answer using the given codes: 141. What is the minimum age that a person must attain to
A. 1, 2, 3 and 4 B. 2 and 4 be eligible to be a member of a panchayat?
C. 1 and 2 D. 1, 2 and 3 A. 18 years B. 21 years
132. L.M. Singhvi suggested that Panchayati Raj should C. 25 years D. 30 years
primarily be viewed as the: 142. Who among the following constitute the National
A. Rural self-government system Development Council?
B. Local self-government system 1. The Prime Minister
C. Urban self-government system 2. The Chairman of Finance Commission
D. Political self-government system 3. Ministers of the Union Cabinet
133. Municipalities have which of the following authorities? 4. Chief Ministers of the States
A. The Council and its committees Select the correct answer using the codes given below:
B. Chairman/President A. 1, 2 and 3 only B. 1, 3 and 4 only
C. Chief Executive Officer/Chief Municipal Officer C. 2 and 4 only D. 1, 2, 3 and 4
D. All of the above
143. Which of the following statements is/are correct?
134. Urban areas where troops are stationed are known as: 1. National Development Council is an organ of the
A. Cantonments B. Tents
Planning Commission.
C. Camps D. Dormitories
2. The economic and social planning is kept in the
135. Seventy-Third Amendment Act is extremely important concurrent list in the Constitution of India.
for political empowerment of: 3. The Constitution of India prescribes that
A. Women B. Scheduled Castes Panchayats should be assigned the task of
C. Scheduled Tribes D. All of these preparation of plans for economic development
136. Which of the following parts of the constitution and social justice.
envisages a three-tier system of panchayats? A. 1 only B. 2 and 3 only
A. Part IX B. Part VII C. 1 and 3 only D. 1, 2 and 3
C. Part XI D. Part VI 144. The Government enacted the Panchayat Extension to
137. Which of the following amendments inserted parts IX Scheduled Areas (PESA) Act in 1996. Which one of
and IX A in the Constitution? the following is not identified as its objective?
A. 72nd and 73rd B. 70th and 71st A. To provide self-governance
C. 73rd and 74th D. 18th and 19th B. To recognize traditional rights
C. To create autonomous region in tribal areas
138. Part IX A which has come into force in 1993 gives a D. To free tribal people from exploitation
Constitutional foundation to the local self-government
units in: 145. Which among the following countries does not have
A. Villages a Central Civil Service Commission charged with the
B. Rural areas responsibility for recruitment to the higher civil service?
C. Urban areas A. Canada B. UK
D. None of the above C. USA D. France
(2751) Pol. Sci.—3-II
Political Science 19
146. Which of the following was not identified by the Reason (R) : The Accredited Social Health Activist
Santhanam Committee as a major cause of corruption (ASHA) is playing a significant role
in India? in providing healthcare services to the
A. Administrative delays needy couples.
B. Scope for personnel discretion in the exercise of A. Both (A) and (R) are true and (R) is the correct
powers explanation of (A)
C. Cumbersome procedures B. Both A and R are true, but R is not the correct
D. Absence of regulatory functions of the government explanation of A
147. Mayor-in-Council form of Municipal Corporation is: C. A is true, but R is false
A. Brought by the 74th Constitutional Amendment D. A is false, but R is true
Act 154. Assertion (A) : Education of girls and women in India
B. Based on the theory of Separation of Powers has reinforced gender role, specially
C. Akin to cabinet form of government motherhood.
D. Based on the American local government pattern Reason (R) : In India, women’s education was
148. Globalisation means: neglected for many years.
A. Financial market system is centred in a single A. Both A and R are true, but R is not the correct
state explanation of A
B. The growth of a single unified world market B. Both (A) and (R) are true and (R) is the correct
C. Geographical location of a firm is of utmost explanation of (A)
importance C. R is true, but A is false
D. Foreign capitalist transactions D. R is false, but A is true
149. Which one of the following is not the function of 155. Assertion (A) : Corruption is the cause of poverty and
NDC? underdevelopment.
A. Review of National Plans Reason (R) : The poor are corrupt and
B. Review of Socio-Economic Policies underdeveloped.
C. Suggestions to achieve Plan Targets A. Both A and R are correct and R is the correct
D. Review of Union-State Relations explanation of A
B. Both A and R are correct, but R is not the correct
150. The programme aimed at enabling illiterate women to
explanation of A
acquire functional skills, better awareness of health,
C. A is true, but R is false
hygiene and child care is:
A. National Female Literacy Mission D. A is false, but R is true
B. Functional Literacy for Adult Women 156. Assertion (A) : Social Welfare Administration
C. Female Farmers Functional Literacy and Training generates awareness about the
D. Total Literacy Programme challenges of a society in transition
151. HDI is entrusted with reference to: where negative use of technologies
1. Life expectancy at birth and practices is impacting on the well
2. Real GDP per capita being of women and children.
3. Infant mortality Reason (R) : Social Welfare Administration is the
4. Morbidity sum m ation of social w elfare
institutions, policies and programmes.
Codes: A. Both A and R are correct and R is the correct
A. 1 and 2 are correct B. 2 and 3 are correct explanation of A
C. 3 and 4 are correct D. All the above B. Both A and R are correct, but R is not the correct
152. Which Article of the Constitution of India provides explanation of A
for the District Planning Committee? C. A is true, but R is false
A. Article 242 ZD B. Article 243 ZA D. A is false, but R is true
C. Article 243 ZD D. Article 244 ZA
157. India that is Bharat shall be a “union of states”. From
153. Assertion (A) : The National Rural Health Mission which Constitution was adopted the words “union of
(NRHM) has been successful in states”?
improving health condition of the A. USA B. France
poor couples living in the villages. C. Switzerland D. Canada
20 Political Science

158. The Objective Resolution in the Constituent Assembly 165. The outstanding phenomena of Democratic
was moved by: Consolidation are mainly on;
A. Dr Rajendra Prasad (a) Judicial activism & RTI
B. B.R. Ambedkar (b) The recent work of the Election Commission of
C. Jawaharalal Nehru India
D. Alladi Krishnaswamy Iyer (c) The new panchayatiraj or the consolidation of the
159. The procedure for amending the Constitution is in: institutions of local governance
A. Article 368 B. Article 360 (d) An inclusive civil service
C. Article 367 D. Article 371 Select the correct code from the options given below:
160. Some of the basic philosophy developed over time A. (a) & (b) B. (c) & (d)
that embody the concept of constitutionalism are: C. (c) & (b) D. (a), (b) & (c)
A. separation of powers 166. The RTI Act (2005) attempts to consolidate democracy
B. judicial control and in India by:
C. accountable government (a) Strengthening the notion of equality between the
D. All of these governing and the governed
161. Separation of powers divides the mechanism of (b) It also works to offset the imperialist culture of
governance into branches i.e., as: governance
(a) Legislature (c) It is created by the provisions of the Indian
(b) Executive Evidence Act (1872)
(c) Judiciary (d) Sharing of power with politico-administrative units
(d) Parliamentary Committee Select the correct code from the options given below:
Select the correct code from the options given below: A. (a) & (b) B. (c) & (d)
A. (a) & (b) B. (c) & (d) C. (c) & (b) D. (a), (b) & (c)
C. (c) & (b) D. (a), (b) & (c) 167. The multifaceted process of deepening democracy in
162. Who among the following is the exponent of Rule of India necessarily involves the kinds of actors as:
Law? (a) the State as well as
A. A.V. Dicey B. Barker (b) non-State actors
C. Seeley D. Montesquieu (c) non electoral means
(d) abolition of the caste system
163. Dicey lays down three essential components of Rule
of Law as: Select the correct code from the options given below:
(a) Nobody is to be punished except for a specific A. (a) & (b) B. (c) & (d)
breach of law that is established in an ordinary C. (c) & (b) D. (a), (b) & (c)
legal manner before ordinary courts of law. 168. Democracy works on the principle of:
(b) No one is above the law. A. Checks and balances
(c) Courts play a vital role in protecting the rights B. Intervention of the state
and freedoms of an individual C. Dictatorial Nature of the state
(d) The separation of the judiciary from the Executive D. None of these
under Article 50
169. Who among the following developed Cultural Lag
Select the correct code from the options given below: with special reference to social change?
A. (a) & (b) B. (c) & (d) A. William Fielding Ogburn
C. (c) & (b) D. (a), (b) & (c)
B. MacIver and Page
164. Constitutionalism can be governed with the help of C. Karl Marx
various provisions of constitution that are: D. Herbert Spencer
(a) Rule of law
170. Which among the following constitutional amendment
(b) Separation of power
provided for the establishment of the National Judicial
(c) Checks and balances
Appointments Commission as ultra-vires?
(d) Comptroller and Auditor General
A. 99th constitutional amendment
Select the correct code from the options given below: B. 82nd constitutional amendment
A. (a) & (b) B. (c) & (d) C. 73rd constitutional amendment
C. (c) & (b) D. (a), (b) & (c) D. 74th constitutional amendment
Political Science 21

ANSWERS
1 2 3 4 5 6 7 8 9 10
C B C B D A A D C C
11 12 13 14 15 16 17 18 19 20
B A A B A B D C B A
21 22 23 24 25 26 27 28 29 30
B C B C B D D B D B
31 32 33 34 35 36 37 38 39 40
D D D C C C D D C C
41 42 43 44 45 46 47 48 49 50
D A D D D C D C C D
51 52 53 54 55 56 57 58 59 60
A B A B A D D C A B
61 62 63 64 65 66 67 68 69 70
A A A B D B A C A D
71 72 73 74 75 76 77 78 79 80
A C B A D C A B B B
81 82 83 84 85 86 87 88 89 90
C B D C A A D A B D
91 92 93 94 95 96 97 98 99 100
D D A C D C D C D D
101 102 103 104 105 106 107 108 109 110
B B C C C C B B B B
111 112 113 114 115 116 117 118 119 120
D C D D A B C C C D
121 122 123 124 125 126 127 128 129 130
D C A C B A C A D A
131 132 133 134 135 136 137 138 139 140
A B D A D A C C D D
141 142 143 144 145 146 147 148 149 150
B B B C B D D B D B
151 152 153 154 155 156 157 158 159 160
D C A A C B D C A D
161 162 163 164 165 166 167 168 169 170
D A D D D D A A A A

  
22 Political Science

Politics in India Since Independence

1. The extra constitutional body who is authorised to 10. Development in socialism doesn’t mean:
make assessment of national wealth is: A. Equal distribution of resources
A. Finance Commission B. Egalitarianism
B. Comptroller and Auditor General C. Competition
C. Planning Commission D. Common ownership of resources
D. National Development Council
11. Capitalist model of development relates to:
2. National Development Council comprises of: A. Co-operative model of development
A. All Governors of states B. Individual development
B. All Chief Ministers of states C. Equality of all
C. All Central ministers D. None of these
D. Central Cabinet ministers only
12. Public Sector denotes:
3. Mixed Economy means: A. Individual enterprise for people
A. Mixed form of production and distribution B. Government controlled firms
B. Co existence of Public sector and Public sector C. Communication facilities
C. Economies of third world D. Industries
D. Emerging economies.
13. Market Mechanism is a feature of:
4. Development Planning is: A. Mixed economy B. Capitalism
A. Change Oriented B. Hierarchical C. Socialism D. Marxism
C. Centralism D. Bureaucratic
14. Development Planning Indian economy before 1990
5. The Great depression was an economic phenomena was known for its style of functioning as a:
took place during: A. Socialist economy B. Planned economy
A. 1940s B. 1950s C. Mixed economy D. Capitalist economy
C. 1930s D. 2010s
15. The pillar of economic development for Mahatma
6. HDI means: Gandhi was:
A. Human Development Item
A. Grama Swaraj B. Trusteeship
B. Human Development Index
C. Decentralisation D. Sarvodaya
C. Housing Development Bank of India
D. None of the above 16. The amendment which made state planning boards
mandatory:
7. Chairman of NITI AAYOG of India is:
A. 73rd constitutional amendment
A. President of India
B. 61st amendment
B. Prime Minister of India
C. 42nd amendment
C. Vice President of India
D. None of these
D. Home Minister
17. The Five Year plans launched in India in:
8. MGNREGA stands for:
A. 1950 B. 1951
A. Mahatma Gandhi National Rural Employment
Guarantee Act C. 1965 D. 1947
B. National Rural Integration Progremme 18. The reasons for Economic reforms introduced in India
C. A project under UN are mainly on:
D. None of these (a) Public sector was given an important role in
9. The Stages of Economic Growth was written by: development policies during 1951–1990
A. W.W Rostow B. G B Shaw (b) The performance of the majority of public
C. Kuznets D. Adam Smith enterprises was disappointing

22
Political Science 23
(c) They were incurring huge losses because of C. Its gross production
inefficient management D. Its per capita income
(d) Policy changes were proposed to go public sector
27. The Gandhian Plan was presented by:
underatkings
A. M.N. Roy B. Nehru
Select the correct code of the options given below: C. Shriman Narayan D. 8 leading economist
A. (a), (b) & (c) B. (a) & (b)
28. Which of the following is not a feature of Indian
C. (c) & (d) D. (a) & (c)
Planning?
19. The most urgent problem which prompted the A. Physical Planning
introduction of New Economic Policy in 1991 was: B. Indicative Economic Planning
A. Poor performance of public sector C. Decentralised Planning
B. High tax rate leading to tax evasion D. None of these above
C. Foreign exchange crisis
29. ‘Twenty point programme’ was launched in the year:
D. All of these
A. 1969 B. 1975
20. Reason which gives importance to privatization: C. 1977 D. 1980
A. Decline in private monopoly
30. Mahalanobis model has been associated with which
B. Favour the objective of social welfare
five year plan?
C. Reduce the cost by minimising wastages
A. 1st five year plan B. 2nd five year plan
D. Increase employment due to exit policy for sick
C. 3rd five year plan D. 4th five year plan
public enterprises
31. The annual growth rate of India’s population according
21. The main changes in fiscal policy are:
to 2011 census is:
A. Quantitative restrictions on import and exports
A. 1.9% B. 1.76%
reduced
C. 2% D. None
B. System of fixed exchange rate converted into
market determined exchange rate 32. Among the Indian state which has the highest density:
C. Use of foreign exchange made more liberal A. Bengal B. Kerala
D. All of these C. Punjab D. Bihar
22. The new economic policy seeks to replace: 33. As per the 2011 census literacy rate in India was:
A. LLP raj B. LPG raj A. 94% B. 74.04%
C. LQG raj D. LQP raj C. 88.8% D. 100%
23. Which of the following is not the one of those steps 34. Globalization of Indian Economy means:
taken in financial sector reforms? A. Increasing External borrowing
A. Lifting of regulations on interest rate of deposit B. Larger FDI
B. Liberalisation of branching regulations for both C. Import Substitution
private and public sector banks D. Minimum possible restrictions on economic
C. Delicensing of industries relation with other countries
D. Reduction of barriers for entry of private banks 35. Which one of the following statements about
24. Liberalisation leads to some limitation: globalization is not correct?
A. Lesser participation of foreign investors A. All economies are closed
B. It neglects the social welfare B. Secular economies
C. No improvement in productivity C. Open economies with trade liberalization
D. No improvement in financial sector D. None of these are correct
25. Poverty in less developed countries is largely due to: 36. What is the full form of Niti Aayog?
A. Voluntary idleness A. National Institute to Transform India
B. Income inequality B. National Institute for Transforming India
C. Lack of cultural activities C. National Institution to Transform India
D. Lack of intelligence of people D. National Institution for Transforming India
26. The best indicator of economic development of any 37. The book “Planning Economy for India” was written
country is: by:
A. Its agriculture A. V.K.R.V. Rao B. M. Visvesvaraya
B. Its transport C. K.N. Rai D. Manmohan Singh
24 Political Science

38. India’s economic planning cannot be said to be: 50. The National Commission for Schedule Caste was
A. Indicative B. Imperative established under........of Indian Constitution.
C. Limited D. Democratic A. Article 338 B. Article 250
C. Article 180 D. Article 142
39. How many tribes are there in India?
A. 645 B. 565 51. Which of the following is not matched correctly?
C. 345 D. None of these A. National Commission for Women: 1992
B. National Minorities Commission: 1993
40. In which state of India resides the maximum number
C. National Backward Classes Commission: 2002
of tribes?
D. National Child Rights Protection Commission:
A. Madhya Pradesh B. Nagaland
2007
C. West Bengal D. None of these
52. Who appoints the Chairm an of the National
41. Which is the largest tribe in India?
Commission for Scheduled Castes?
A. Bhil B. Gond A. President
C. Baiga D. None of these B. Prime Minister
42. In which state Garo, Khasi and Jaintia tribes are C. Lok Sabha Speaker
located? D. None of the following
A. Assam B. Meghalaya 53. Which of the following is not the function of the
C. Madhya Pradesh D. None of these National Commission for Scheduled Castes?
43. Daphla and Lampo tribes are found in which state? A. Providing Constitutional protection to Scheduled
A. Madhya Pradesh B. Arunachal Pradesh Castes
C. Rajsathan D. None of these B. To investigate any case that violates the interests
of Scheduled Castes
44. Which state has the largest population of Santhals? C. To submit the report to the Prime Minister related
A. Jharkhand B. Arunachal Pradesh to the protection of Scheduled Castes
C. Rajsathan D. None of these D. None of the above
45. Which caste is largest in Tamil Nadu by population? 54. Which constitution amendment has recommended the
A. Vanniyar B. Raji establishment of a commission for Scheduled Castes
C. Saharya D. None of these and Scheduled Tribes?
46. Find out the odd one: A. 41st Constitutional Amendment
A. Poumai – Manipur B. Garo – Mizoram B. 65th Constitutional Amendment
C. Dafla – Uttar Pradesh D. None of these C. 82nd Constitutional Amendment
D. 76th Constitutional Amendment
47. In which year the Forest Right Act was passed?
A. 2000 B. 2006 55. Which amongst the following is/are correct?
C. 2005 D. None of these 1. Indian society during nineteenth century was
marred by religious superstitions and social
48. What is the name of the tribe that is battling against obscurantism.
Vedanta for preservation of Niyamgiri hills? 2. Social conditions of women were good but some
A. The Kondha or Kandha vices like sati, child marriage prevailed in the
B. Kandha Gauda society
C. Oriya Munda 3. Raja Ram Mohan said “Sati is murder according
D. Dongria Kondh to every shastra”.
4. People at lower ladder of the society were called
49. Which of the following is/are true?
untouchables, who formed about 20 per cent of
1. Article 244 in Part X of the Constitution envisages
population.
a special system of administration for certain areas
designated as ‘scheduled areas’ and ‘tribal areas’. Select the correct code:
2. The Fifth Schedule of the Constitution deals with A. Only 1 B. Only 2 is incorrect
the administration and control of scheduled areas C. All of the above D. None of the above
and scheduled tribes in any state. 56. Which amongst the following is/are correct?
Select the correct answer A. The Hindu Widow Remarriage Act came in 1856.
A. Only 1 B. Only 2 B. Vishnu Shastri Pandit founded Widow Remarriage
C. All of the above D. None of the above Association in the 1850s.
Political Science 25
C. Indian Women University was set up in 1916 at 65. National Mission for Empowerment of Women was
Bombay by Professor Karve. launched on:
D. All are correct A. 15 August, 2010
B. 15 August, 2011
57. Who among the following started dalit movement in
C. 15 October, 2010
India?
D. 15 October, 2011
A. Dr. B.R. Ambedkar B. Bal Gangadhar Tilak
C. Jyotiba Phule D. None of these 66. Which scheme promotes community participation
through involvement of Students Volunteers for
58. According to Dr. B.R. Ambedkar the dalits and
empowerment of rural women?
marginalised in this country will have struggle against
A. NAND-GHAR YOJANA
two adversaries. Which are they?
B. eSamvad Portal
1. Brahmanism 2. Capitalism
C. Pradhan Mantri Mahila Shakti Kendra scheme
3. Socialism 4. Communalism
D. She-Box Portal
Codes:
A. 1 and 2 B. 1 and 4 67. Arrange the following initiatives with regard to
C. 2 and 3 D. 3 and 4 Women’s Studies in chronological order:
1. Establishment of Indian Association of Women’s
59. Which of the following matches is not correct?
studies.
A. Louis Dumont — Caste based on purity and
2. First National Conference on Women’s Studies.
pollution
3. Publication of ‘Towards Equality Report’
B. B.R. Ambedkar — Caste as graded form of
4. Second National Education Policy
inequality
Code:
C. M.N. Srinivas — Dominant caste
A. (1), (3), (2), (4) B. (1), (2), (3), (4)
D. G.S. Ghurye — Racial origin of caste
C. (3), (2), (1), (4) D. (4), (2), (3), (1)
60. Name the online platform launched by the Ministry of
68. What is the common focus of most feminist therapies
Women and Child Development to support women
today?
entrepreneurs?
A. Improving living conditions for women around
A. Digital India
the world.
B. Mahila E-haat
B. Creating a more gentle, loving, woman centred
C. One Stop Centre Scheme
world.
D. None of the above
C. Showing how society’s view of treatment of non-
61. In which year the Central Social Welfare Board was dominant groups impacts mental health.
established? D. Seeking balance in the un-equal power
A. 1950 B. 1951 relationships between women and men.
C. 1952 D. 1953
69. For which of the following reasons early feminist
62. Which of the following scheme provide education to approaches to development have been criticised?
girls and their welfare? (a) Only looking at women
A. One Stop Centre Scheme (b) Not engaging with development agencies
B. UJJWALA (c) Having unrealistic aims
C. SWADHAR Scheme (d) Ignoring women’s roles in production
D. Beti Bachao Beti Padhao
Select correct code of the options given below
63. Support to Training and Employment Programme for A. (a), (c) & (d) B. (c), (b) & (a)
Women scheme is: C. (b), (a) & (d) D. (a) & (b)
A. SWADHAR B. STEP
70. Assertion (A) : In patriarchal society, all economic
C. Nari Shakti Puraskar D. RMK
development causes the marginali-
64. Priyadarshini scheme provides: zation and devaluation of women’s
A. Women Empowerment labour.
B. Livelihood in Mid-Gangetic plains Reason (R) : In patriarchal society, the nature of
C. Empower to address women legal, political and house work and women’s responsi-
health problems bility for carrying it out are unchange-
D. All the above are correct. able i.e., constant.
(2751) Pol. Sci.—4
26 Political Science

Code: 77. The epicenter of farmers protest is:


A. Both (A) and (R) are wrong A. Punjab & Haryana
B. Both (A) and (R) are correct and (R) is the correct B. Punjab & Madhya Pradesh
explanation for (A) C. Bihar & Uttar Pradesh
C. (A) is correct, but (R) is wrong D. Punjab & Rajasthan
D. Both (A) and (R) are correct, but (R) is not the 78. Consider the following statement regarding Farmers
correct explanation for (A) Fear in new farms Act:
71. As the women’s suffrage movement emerged from the 1. MSP will stop
abolition m ovem ent, the wom en’s liberation 2. APMC will end
movement grew out of the struggle for: 3. Big Companies will exploit farmers in the name
A. civil rights of contract
A. political power Choose correct code from given below:
B. dominance in the family A. 1 and 3 Only B. 2 and 3 Only
C. challenge against livelihoods C. 1 and 2 Only D. 1, 2 and 3
72. Which of the following is correct with regard to 79. Indian farmer was goaded into cultivation of opium
‘identity politics’ in the feminist context in India? by:
A. Minority, caste and dalit issues A. promise of high prices
B. Race and ethnicity B. system of advances
C. Gender, race, caste, class and minorities C. the village headman
D. Gender, race, caste, minorities D. revenue concessions
73. Which among the following book reveals that the 80. Strip cultivation was practiced by:
white hetrosexual, middle class wom en w ere A. peasants in England B. tribals
discontented and unhappy despite living the “Ideal” C. nomadic pastoralists D. shifting cultivators
female life but were unable to explain why is “the 81. For poorer farmers machines brought misery because
problem that has no name”? of:
A. The second sex B. Feminine mystique A. displacement B. banks refused loans
C. The second stage D. The subjection of women C. banks gave loans D. indebtedness
74. Which among the following feminist identified 82. Which of the following was an important factor in
“alienation” as the concept that will provide us with farmers investing in threshing machines?
a theoretical framework to accommodate main insight A. Labour shortage
of Marxist, radical, psychoanalytic and even liberal B. Insolence of labourers
feminist thought? C. High wages demanded by labour
A. Iris young B. Alison Jagger D. To reduce dependence on labour
C. Juliet Mitchell D. Mary Daly 83. In which Farmers’ movement did Mahatma Gandhi
75. Among the following feminists who has analysed participate first of all?
‘Reproduction through the lens of male alienation’ A. Kheda B. Champaran
while criticizing Shulamith Firestone’s views on female C. Bardoli D. Baroda
biology as the cause of women’s oppression: 84. Which among the following Farmer’s revolt are not
A. Mary O’ Brien B. Mary Dally correctly matched?
C. Kate Millet D. Juliet Mitchell A. Deccan Peasant Revolt (1875)
76. Consider the incorrect statement regarding New Farms B. Mopilla Revolt (1921)
Act, 2020: C. Bardoli (1927)
A. The Farmers’ Produce Trade and Commerce D. Kheda (1918)
(Promotion and Facilitation) Act, 2020 85. Farmers’ movement led by rich farmers in:
B. The Farmers (Empowerment and Protection) A. Uttar Pradesh B. Karnataka
Agreement of Price Assurance and Farm Service C. Maharashtra D. All of these
Act, 2020 86. The beginning of the New Farmers’ Movement in
C. The Farmers Protection Act, 2020 general is seen from the decade of:
D. The Essential Commodities (Amendment) Act, A. 1980s B. 1970s
2020 C. 1960s D. 1940s
Political Science 27
87. Some of the most important Farmer’s struggles that 93. Why has the term ‘civil society’ aroused suspicion in
were carried till the beginning of 1980s.Which of the some Islamic states?
following struggles are not correctly matched? A. In those countries the state denies civil rights to
A. Anti-Single Food Zone, 1972 its citizens.
B. Struggle against Power Tariff, 1975 B. It can be seen as part of a project of ‘Westerni-
C. Struggle against increasing water rates, increasing zation’.
commercial tax, 1975 C. The term is difficult to translate into Arabic.
D. Agitation against defective tractors, 1979 D. It failed to recognize that brutal violence is the
88. The Uttar Pradesh movement under Mahendra Singh only way to facilitate change
Tikait organised many rallies, agitations as well as 94. What does ‘infrapolitics’ denote?
struggles. Some of the major well known struggles are A. Relations between sovereign states
as: B. Relationships between key decision-makers
(a) Struggle against Power Tariff, 1986 C. The way in which seemingly powerless people
(b) Meerut Struggle, 1988 subvert the authority of the powerful
(c) Agitation for Payment of Arrears, 2007 D. The disengagement of many people nowadays
(d) Sit-in Dharna at Jantar Mantar, 2008
from the political process
Codes:
95. A positive view of the political role of the media sees
A. Both (a) & (b) B. Both (c) & (d)
it as:
C. Both (b) & (a) D. All of the above
A. a way of keeping people harmlessly amused when
89. Shetkari Sangathana of Maharashtra also carried couple times are hard.
of struggles. Which of the following are correctly B. a way of promoting constructive ‘role models’ for
matched? young people
(a) Nasik Agitation of 1980 C. much more trustworthy than politicians
(b) Nippani Agitation of 1981 D. at its best, a potential check on executive power
(c) Rail and Rasta Roko agitation of 1981, 1986
(d) Pandharpur Rally of 1983 96. What is civil Society?
A. The society considered as a community of citizens
Codes: linked by common interests and collective activity
A. Both (a) & (b) B. Both (c) & (d) is a civil society.
C. Both (b) & (a) D. All of the above B. It is the aggregate of non-governmental organi-
90. The new strategies “Gav Bhandi” is associated with zations and institutions that manifest interests and
the farmer’s movement of: will of citizens.
A. Maharashtra B. Madhya Pradesh C. It is referred to as the third sector of the society
C. Karnataka D. West Bengal distinct from government and business.
91. Civil society can be understood as: D. All of the above
A. the whole population of a state 97. Which one of the following statements is not correct
B. the people within a state who behave in a about civil society organisations?
politically civilised way A. They are sovereign bodies
C. a social organization made up of retired civil B. They are voluntary but nonprofit-making agencies
servants. C. They work in close proximity with people
D. a framework within which people who lack D. They create awareness; educate and sensitise
political authority conduct their lives. people on common issues and act as catalysts of
92. Why did the term ‘civil society’ become popular in social change
the 1980s? 98. Value of the programme in Social action group work
A. At that time there was growing disenchantment lies in?
with the perceived dishonesty of governments,
A. Establishing satisfying affective (love) relationship
especially as recipients of aid
B. Enhancing knowledge of possible alternative
B. The downfall of many totalitarian regimes was
activities.
attributed to movements generated by civil society
C. Giving expression to creative dynamics drives
C. It seemed to offer the possibility of peaceful
change in other states D. Increasing proficiency in the chosen programme
D. All of the above activity.
28 Political Science

99. Social action group work as a method of social work Codes:


aims at: A. (a), (b) and (d) B. (b), (c) and (d)
A. Development of democratic life style C. (b) and (c) D. (a) and (d)
B. Development of leadership qualities
107. Identify two reasons that state that caste alone cannot
C. Development of capability of adjustment
determine elections in India:
D. All the above
(a) No party wins the votes of all the voters of a caste
100. Which of the following is NOT a principle of social or community.
group work? (b) Some political parties are known to favour some
A. Confidentiality castes and are seen as their representatives.
B. Guided group interactions (c) No parliamentary constituency in the country has
C. Progressive programme development a clear majority of one single caste.
D. Evaluation of the progress made by the group (d) Mobilising and securing political support has
brought new consciousness among the lower castes.
101. The book “Social Group Work – A Helping Process”
is authored by? Codes:
A. H.B. Trecker B. G. Konopka A. (a) and (c) B. (a) and (d)
C. G.H. Mead D. Peter Samuel C. (b) and (c) D. (b) and (d)
102. The Mutual Aid Model of group work practice has its 108. When we speak of gender divisions, we usually refer to:
roots in the practice theory proposed: A. Biological difference between men and women
A. H.B. Trecker B. G. Konopka B. Unequal roles assigned by the society to men and
C. William Schwartz D. Naiper women
C. Unequal child sex ratio
103. Regional Disparity means:
D. Absence of voting rights for women in democracies.
A. It refers uneven economic achievement in different
geographical regions 109. In India seats are reserved for women in:
B. It is reflected by the indicators like per capita income (a) Lok Sabha
C. It refers to the proportion of population living (b) State legislative assemblies
below the poverty line (c) Cabinets
D. All of these (d) Panchayati Raj bodies
104. Government Interventions to reduce Regional Codes:
Disparities are: A. (a), (b) and (d) B. (b), (c) and (d)
A. Higher resource transfers from the Centre to the C. (b) and (c) D. (a) and (d)
Backward States 110. Which among the following statements about India’s
B. Planning Commission (before 2014) mainly in
Constitution is wrong? It:
the form of plan transfers
A. prohibits discrimination on grounds of religion.
C. Finance Commission in the form of non-plan
B. gives official status to one religion.
transfers
C. provides to all individuals freedom to profess any
D. All of these
religion.
105. Among the following, which countries have high D. ensures equality of citizens within religious
participation of women in public life? communities.
A. Sweden and India B. Norway and Sri Lanka
111. In India, the representation of women in legislature
C. Nepal and Finland D. Sweden and Africa
has been:
106. Identify the statements which suggest that it is not A. moderate B. high
politics that gets caste-ridden, it is the caste that gets C. low D. very low
politicised.
112. The emergence of political parties has accompanied
(a) When governments are formed, political parties
with:
take care that representatives of different castes
A. Grow of parliament as an institution
find a place in it.
B. Diversification of political systems
(b) Each caste group incorporates neighbouring castes
C. Growth of modern electorate
which were earlier excluded.
D. All of the above
(c) Various caste groups enter into a coalition with
other castes. 113. Political parties are responsible for maintaining a
(d) Political parties and candidates in elections make continuous connection between:
appeals to caste sentiments. A. People and the government
Political Science 29
B. President and the Prime Minister 122. The issue of adequate participation of women in
C. People and the opposition Panchayati Raj institutions was first raised by:
D. Both A and C A. Santhanam Committee Report
114. Rajni Kothari prefers to call the Indian party system as: B. Ashok Mehta Committee Report
A. Congress system C. Balwant Rai Mehta Committee Report
B. One party dominance system D. All of the above
C. Multi-party systems 123. The term ‘Stasiology’ refers to:
D. Both A and B A. Party politics
B. Scientific study of party politics
115. During which of the following years the Congress
C. Study of pressure group politics
party has faced serious electoral reverses?
D. None of the above
A. 1967 B. 1977
C. 1989 D. All of these 124. Who am ong the following divided the non-
competitive party system into one party authoritarian,
116. What is the most appropriate meaning the term
one-party pluralistic and one-party totalitarian?
secularism in the Indian context?
A. Alan Ball
A. Seperation of religion from politics
B. Freedom of religion B. La Palaombara and Myron Weiner
C. There shall not be any religion of the state C. Harry Eckstein
D. Equality of all religions D. James Jupp.

117. Which of the following are the important socio- 125. The Dalit panther movement was launched in:
economic factors of regional imbalance in India? A. West Bengal B. Bihar
A. per capita income C. Odisha D. Maharashtra
B. regional location of industries 126. Political parties are registered as per the provision of:
C. population A. Article 324 of the Constitution
D. below poverty line B. The Election Commission
118. Which of the following can play a vital role in C. The Election Commission in consultation with
removing regional disparity? the govt.
A. Bureaucracy B. State D. Representation of Peoples Act 1951
C. Civil servants D. Judiciary 127. Which of the following is one of the salient features
119. As a political strategy, communalism is opposed to: of the coalition governments?
A. Nationalism B. Regionalism A. Lack of polarization
C. Localism D. Internationalism B. Political opportunism
C. Political defections
120. Identity the correct chronological order in which the D. All of the above
following came into existence.
Use the codes given below: 128. Which of the following is not the type of coalition
1. Swatantra Party categorized on the basis of interaction of Infrastructure
2. Muslim League and Politics?
3. Bharatiya Janata Party A. Few party governmental coalition
4. Communist Party of India B. Secular coalition
Code: C. Post election coalition
A. 4, 3, 1, 2 B. 1, 3, 2, 4 D. Infrastructure dominated coalition
C. 2, 4, 1, 3 D. 3, 1, 4, 2 129. Birsa Munda was associated with tribal struggle of:
121. A political party is recognized as a regional party, if: A. North East Region B. Jharkahand
A. It gets 4 per cent votes in the state either in the C. Nagar Division D. Deccan
Lok Sabha or the Assembly elections 130. In the first decade following independence, which of
B. It gets 5 per cent votes in the state either in the the following legislations transformed the legal status
Lok Sabha or the Assembly elections of women?
C. It gets 6 per cent votes in the state either in the A. The Hindu Marriage Act, 1955
Lok Sabha or the Assembly elections B. The Succession
D. It gets 7 per cent votes in the state either in the C. The Adoption Act, 1956
Lok Sabha or the Assembly elections D. All of the above
30 Political Science

ANSWERS
1 2 3 4 5 6 7 8 9 10
C B B A C B B A A D
11 12 13 14 15 16 17 18 19 20
B B B C A A B A C C
21 22 23 24 25 26 27 28 29 30
A C C B B C A A B B
31 32 33 34 35 36 37 38 39 40
A B B D C D B B A A
41 42 43 44 45 46 47 48 49 50
A B B A A C B D A A
51 52 53 54 55 56 57 58 59 60
C A C B B D A A D B
61 62 63 64 65 66 67 68 69 70
D D B D B C B D A A
71 72 73 74 75 76 77 78 79 80
A C B B A C A D B A
81 82 83 84 85 86 87 88 89 90
D D B C D A D D D A
91 92 93 94 95 96 97 98 99 100
D D B C D D A A D A
101 102 103 104 105 106 107 108 109 110
B B C D D C A B A B
111 112 113 114 115 116 117 118 119 120
D C D D D D D B A C
121 122 123 124 125 126 127 128 129 130
C C B B D D D C B D

  
Political Science 31

India’s Foreign Policy

1. Which among the following is not a philosophical 10. First SAARC summit was held at:
base of the Indian Foreign policy? A. Dhaka B. New Delhi
A. Containment of Communism C. Colombo D. Islamabad
B. Anti-imperialism 11. India is part of:
C. Non Alignment A. SAARC B. NAFTA
D. Anti-colonialism C. CER S. European Union
2. Who among the following is the chief architect of 12. Which among the following is not a South Asian
India’s Foreign policy? Country?
A. Gandhiji B. Jawaharlal Nehru A. Sri Lanka B. Afghanistan
C. Indira Gandhi D. Sardar Vallabhai Patel C. Pakistan D. China
3. Which agreement was signed between India and 13. India conducted its first Nuclear Test in the year:
Pakistan after Indo-Pakistani War of 1965? A. 1974 B. 1978
A. Tashkent Agreement C. 1998 D. 1996
B. Bangkok Agreement
C. Panchasheel Agreementd 14. Chakmas are the ethnic groups in which of the
D. None of the above following country?
A. Sri Lanka B. Afghanistan
4. Simla Agreement was signed in: C. Pakistan D. Bangladesh
A. 1971 B. 1972
C. 1974 D. 1975 15. India declared LTTE to be a terrorist outfit after the
assassination of who among the following Prime
5. Simla Agreement was signed between: Minister of India?
A. Jawaharlal Nehru and Muhammad Ali Jinnah A. Indira Gandhi B. Rajiv Gandhi
B. Benazir Bhuto and Indira Gandhi C. Lal Bahadur Shastri D. None of these
C. Indira Gandhi and Zulfiqar Ali Bhutto
D. Jawaharlal Nehru and Zulfiqar Ali Bhutto 16. Which among the following is associated with the
liberation of Bangladesh?
6. Simla Accord signed in 1974 is related to: A. LTTE
A. Status of Tibet B. Mukti Bahini
B. Liberation of Bangladesh C. NATO
C. Ending the war between India and Pakistan in D. International Court of Justice
1971
17. Bangladesh was liberated from Pakistan in the Year:
D. None of the Above
A. 1971 B. 1972
7. SAARC was established in the year: C. 1980 D. 1985
A. 1975 B. 1980
18. In which year China conducted its first Nuclear
C. 1985 D. 1990
explosion:
8. Which among the following Decade is designated by A. 1960 B. 1962
SAARC as the SAARC Decade of the Rights of the C. 1964 D. 1974
Child?
19. What is India’s position regarding Iran in the
A. 1991-2000 B. 2001-2010
International Atomic Energy Agency in 2005:
C. 2011-2010 D. 2006-2015
A. India favoured
9. SAFTA agreement came in to force on: B. India abstained from voting
A. 5 September, 1990 B. 30 January, 2005 C. India Voted against
C. 10 December, 1985 D. 1 January, 2006 D. None of these

31
32 Political Science

20. Which among the following is not a principle of India’s 30. Where does the European Commission have its
Nuclear Doctrine today? offices?
A. No first use A. Brussels B. London
B. Credible Minimum deterrent C. Washington DC D. New York
C. Civilian Control 31. Which of the following treaty established the European
D. First use Union under its current name in 1993?
21. Which treaty is a bilateral agreement and governance A. Washington B. Maastricht
treaty between India and Pakistan, signed on February C. Utrecht D. None of these
21, 1999? 32. IAEA stands for:
A. Delhi Declaration B. Colombo Declaration A. International Atomic Energy Agency
C. Beijing Declaration D. Lahore Declaration B. International Alliance of Economic Agencies
22. In 1992 India signed which of the following C. Indian Atomic Energy Agency
Convention (CWC), stating that it did not have D. None of these
chemical weapons and the capacity or capability to
33. In which year ASEAN was formed?
manufacture chemical weapons?
A. 1965 B. 1978
A. NPT B. Biological weapon
C. 1979 D. 1967
C. Chemical Weapons D. Disarmament
34. Where is head quarters of ASEAN located at?
23. India’s Strategic Nuclear Command was formally
A. Beijing B. Jakarta
established in the year ……….. .
C. Brunei D. None of these
A. 2006 B. 1998
C. 2003 D. 2010 35. Which among the following statement regarding
Terrorism is right?
24. The National Security Council (NSC) of India is the
A. There is no universally accepted definition for
apex agency looking into the country’s political,
terrorism
economic, energy and strategic security concerns. It
B. There is a universally accepted definition for
was established by who among the following Prime
terrorism
Minister of India?
C. United Nations definition on Terrorism is accepted
A. Man Mohan Singh B. Indira Gandhi
by all the nations
C. Atal Behari Vajpayee D. Rajiv Gandhi
D. None of these
25. Which among the following is a principle of US foreign
36. Which among the following is not a Panchasheel
policy during the time of Cold War?
Principle?
A. Non Alignment
A. Mutual non-aggression
B. Socialism
B. Peaceful co-existence
C. Containment of Communism
C. No first use of nuclear weapons
D. None of these
D. Equality and mutual benefit
26. Who among the following is not associated with NAM?
37. The Millenium Development Goals are supposed to
A. Jawaharlal Nehru
be achieved by the year:
B. Kwame Nkumrah of Ghana
A. 2020 B. 2015
C. Gamel Abdel Nasser
C. 2030 D. 2050
D. Mao- Tse Tung
38. The ‘Blue helmet’ refers to:
27. In which year the Bandung Conference was held?
A. Red Cross
A. 1950 B. 1955
B. UNICEF
C. 1960 D. 1965
C. UN Peace Keepers
28. Which of the following is a military alliance? D. Human Right volunteers
A. SAARC B. EU
39. Dumbarton Oaks conference of 1944 emphasis on:
C. NATO D. NAM
A. Promotion of peace
29. The NAM summit which demanded expansion of the B. International brotherhood
United Nations Security council had met at: C. Equality
A. Tehran B. Cairo D. Promotion of human right and fundamental
C. Akarta D. Harare freedom
Political Science 33
40. Which decade is known as UN Literacy decade? 51. The main policy-making body of UN dealing with
A. 1970’s B. 1993-2003 human rights issues is:
C. 2013-2023 D. 2003-2012 A. Security Council
41. Camp David Peace Treaty is related to: B. The Human Rights Council
A. The developmental practices in the third world C. UNESCO
countries D. Human Rights Commission
B. Korean problem 52. The International Convention on the Elimination of
C. Pakistan problem All Forms of Discrimination against Women was
D. Palestine problem adapted in the year:
42. Prisoner’s dilemma is a phenomenon that fuels: A. 1977 B. 1952
A. arms race B. arms control C. 1997 D. 1981
C. disarmament D. peace 53. The International Covenant on Civil and Political
43. Seventh Chapter of UN charter deals with: Rights was adapted in the year:
A. Amendment of UN charter A. 1966 B. 1952
B. Collective Security C. 1997 D. 2005
C. Trusteeship 54. The United Nations advisory services programme is
D. NIEO related to:
44. Which article of the UN charter permits military action A. UN reforms B. Financial aid
against aggression? C. Human Rights D. Development
A. 42 B. 44 55. The First International tribunal to deal with terrorism
C. 16 D. 15 as a crime was:
45. The Korean issue was in the year: A. Special Court for Sierra Leone
A. 1950 B. 1947 B. International Criminal Tribunal for Rwanda
C. 1991 D. 2001 C. Nuremburg tribunal
D. None of the above
46. Which of the following decisions in G eneral
Assemblies of United Nations requires two-third 56. The Permanent court of International Justice was
majority? established in
A. Peace and security A. 1919 B. 1922
B. Admission of new members C. 1945 D. 1950
C. Budgetary matters 57. The treaty of Westphalia was signed in the year:
D. All the above A. 1648 B. 1957
47. The Resolution 678 (1990) of UN Security Council C. 1888 D. 1216
deals with: 58. The UNO tries to solve the problems of the world
A. Disarmament B. Kashmir Issue by:
C. Korean Issue D. Iraque Issue A. oppression
48. Which one doesn’t belong to the methods of peaceful B. using its military force
settlement of disputes provided by the U.N? C. co-operation
A. Negotiation D. reducing financial aids preventing import and
B. Arbitration export
C. Judicial settlement 59. “Veto” in UN security council is:
D. Force A. the positive vote of all the members
49. The Human Rights Council was established in: B. the negative vote by a permanent member
A. 2012 B. 1948 C. the negative vote for a central country
C. 1999 D. 2006 D. the negative vote for the security council
50. The ‘Third Committee’ is associated with: 60. The United Nations charter contains the:
A. Collective security A. Aims of organization
B. Trusteeship B. Rates of membership fee
C. Human Rights C. Amount of aid to be granted to each country
D. None of the above D. Financial gain for its members
(2751) Pol. Sci.—5
34 Political Science

61. The United Nations Organization came into existence 70. What is Quad?
as a result of: A. Quad is an informal strategic dialogue of like-
A. Two world war minded democracies
B. A desire of nations to have durable peace B. It is converging across the Indian and Pacific
C. Efforts to make world free from wars oceans
D. All the above C. It comprises of the US, Japan, India and Australia.
D. All of these
62. Which are the specialized agencies of UN known as
Breton world sisters? 71. How many union consisting The African Union (AU)
A. UNESCO and UNICEF as a continent?
B. IMF and WHO A. 55 countries B. 60 countries
C. IBRD and UNESCO C. 70 countries D. 80 countries
D. IMF and World Bank 72. In which year the African bloc was founded?
63. The First International tribunal to deal with terrorism A. On 26 May, 2001 in Addis Ababa
as a crime was: B. On 26 May, 2003 in Addis Ababa
A. Special Court for Sierra Leone C. On 26 May, 2008 in Addis Ababa
B. International Criminal Tribunal for Rwanda D. On 26 May, 2009 in Addis Ababa
C. Nuremburg tribunal 73. Which among the following is not correctly matched?
D. None of the above A. The intention of the AU is to replace the
64. Who is currently appointed as Secretary-General of Organisation of African Unity (OAU)
Gulf Cooperation Council? B. It was established on 25 May 1963 in Addis
A. Fahim bin Sultan Al Qasim Ababa by 32 signatory governments
B. Jamil Ibrahim Hejailan C. The AU ’s secretariat, the African U nion
C. Nayef Falah Mubarak Al Hajraf Commission, is based in Addis Ababa
D. Abdullah Bishara D. To achieve greater unity and solidarity in the
middle east.
65. In which year the Charter of the GCC was signed
formally establishing the institution? 74. In which of the following, a person is constantly
A. 25 May, 1981 B. 25 May, 1980 followed/chased by another person or group of several
C. 25 May, 1985 D. 25 May, 1987 peoples?
A. Phishing B. Bulling
66. Who among the following Prime Minister launched
C. Stalking D. Identity theft
Look East policy in 1992, to give a strategic push to
India’s engagement with South-East Asia region? 75. Which of the following usually observe each activity
A. P.V. Narasimha Rao B. Narendra Modi on the internet of the victim, gather all information in
C. Manmohan Singh D. Atal Behari Vajpayee the background, and send it to someone else?
A. Malware
67. Act East Policy is the successor of:
B. Spyware
A. Look West Policy B. Look East Policy
C. Adware
C. Neither A nor B D. Both A & B
D. All of the above
68. Prime Minister of India highlighted which of the 4C’s
76. Which of the following refers to stealing one’s idea or
of Act East Policy:
invention of others and use it for their own benefits?
(a) Culture
A. Piracy
(b) Commerce
B. Plagiarism
(c) Connectivity
C. Intellectual property rights
(d) Capacity building
D. All of the above
Select the correct code the options given below:
77. Which of the following refers to exploring the
A. (a) & (b) B. (c) & (a)
appropriate, ethical behaviors related to the online
C. (b) & (d) D. (a), (b), (c) & (d)
environment and digital media platform?
69. What is an important dimension of India’s Act East A. Cyber law
Policy? B. Cyberethics
A. Connectivity B. Capacity building C. Cybersecurity
C. Security D. None of these D. Cybersafety
(2751) Pol. Sci.—5-II
Political Science 35
78. Which of the following are famous and common cyber- 80. Which of the following statements is correct about the
attacks used by hackers to infiltrate the user’s system? firewall?
A. DDos and Derive-by Downloads A. It is a device installed at the boundary of a
B. Malware & Malvertising company to prevent unauthorized physical access.
C. Phishing and Password attacks B. It is a device installed at the boundary of an
D. All of the above incorporate to protect it against the unauthorized
79. In Wi-Fi Security, which of the following protocol is access.
more used? C. It is a kind of wall built to prevent files from
A. WPA B. WPA2 damaging the corporate.
C. WPS D. Both A and C D. None of the above

ANSWERS
1 2 3 4 5 6 7 8 9 10
A B A B C A C B D A
11 12 13 14 15 16 17 18 19 20
A D A D B B A C C D
21 22 23 24 25 26 27 28 29 30
D C C C C D B C C A
31 32 33 34 35 36 37 38 39 40
B A D B A C B C A D
41 42 43 44 45 46 47 48 49 50
D A B A A A D D D C
51 52 53 54 55 56 57 58 59 60
B D A D A B A C B A
61 62 63 64 65 66 67 68 69 70
D D A C A A B D C D
71 72 73 74 75 76 77 78 79 80
A A D C B D B D B B

  
36 Political Science

Governance and Public Policy in India

1. The rise in the popularity of the term governance is 6. Which among the following five year plans explicitly
closely linked with the: took upon the issue of governance by indicating that
A. redefinition of the role of state governance has in the recent times emerged at the
B. redefinition of the role of civil society forefront of the development agenda?
organizations A. Fifth Five Year Plan B. Sixth Five Year Plan
C. redefinition of the role of public undertakings C. The Tenth Plan D. 12th Five Year Plan
D. None of these 7. The state comprises the various institutions of:
2. In which of the following report redefined the role of 1. Government 2. Bureaucracy
state? 3. Military 4. Police
A. World Development Report, 1994 5. Courts 6. Social security system
B. World Development Report, 1992 Select the correct codes:
C. World Development Report, 1990 A. 1, 2, 3 & 4 B. 1, 2, 3, 4 & 5
D. World Development Report, 1991 C. 2, 3, 5 & 6 D. 1, 2, 3, 4, 5 & 6
3. Governance refers to: 8. Governance was first used by the World Bank in its
A. the complex set of institutions and actors report on
B. the blurring of boundaries and responsibilities for A. Sub-Saharan Africa in 1989
tackling social and economic issues B. Sub-Saharan Africa in 1990
C. the autonomous self, governing networks of actors C. Sub-Saharan Africa in 1998
D. All of these D. Sub-Saharan Africa in 1994

4. World Bank has employed major dimensions to 9. Which of the following is the distinguishing
construct measures of governance. These dimensions characteristic of State, as compared with other
are: associations?
A. Population B. Territory
1. Voice and Accountability—measuring political,
C. Government D. Sovereignty
civil and human rights
2. Instability and violence—m easuring the 10. Which of the following are the four characteristics of
likelihood of violent threats to or changes in state?
government, including terrorism A. Government, territory, population, association
3. G overnm ent effectiveness—m easuring the B. Association, sovereignty, territory, population
competence of the bureaucracy and the quality of C. Army, territory, population and sovereignty
public service delivery D. Population, territory, government and sovereignty
4. Regulatory burden—measuring the incidence of 11. The term state has often been confused with:
market, unfriendly policies A. Sovereignty B. Association
5. Rule of law—measuring the quality of contract C. Nation D. Government
enforcement, the police, and the courts, as well as 12. Which of the following is not true about difference
the likelihood of crime and violence between the State and the Government?
Select the correct codes: A. The will of the State is expressed through
A. 1, 2 & 3 B. 3, 4 & 5 Government
C. 1, 2, 3, & 5 D. 1, 2, 3, 4 & 5 B. The State is concrete while the Government is
abstract
5. In which year the term governance gained currency in C. The State has original whereas Government has
India with the realization of the Indian economy? delegated powers
A. 1991 B. 1992 D. The State is large whereas Government is a small
C. 1995 D. 1998 body
36
Political Science 37
13. The relationship between civil society and state refers 19. Good governance is associated with:
to: A. accountability of political leadership
1. The civil society is an integral part of governance B. enlightened policy-making
2. Civil society also mediates the relationship C. a civil service imbued with a professional ethos
between the state and the market D. All of these
3. The state mediates the relationship between
20. Good governance incorporates features like:
civil society and the market for the welfare of
1. equity 2. participation
people
3. transparency 4. accountability and
4. The state provides an institutional framework
5. rule of law
within which civil society can prosper and flourish
Select the correct codes:
Select the correct codes:
A. 1, 2, 4 & 5 B. 2, 3, 4 & 5
A. 1, 2 & 3 B. 2, 3 & 4
C. 1, 3, 4 & 5 D. 1, 2, 3, 4 & 5
C. 1, 3 & 4 D. 1, 2, 3 & 4
21. Good Governance Day (Susasan Divas) was established
14. Civil society can be understood as:
in India in 2014, to be observed annually on 25-
A. the whole population of a state.
December, the birth anniversary of:
B. the people within a state who behave in a
A. former Prime Minister Shri Atal Bihari Vajpayee
politically civilised way
B. former Prime Minister Shri P.V. Narsimha Rao
C. a social organization made up of retired civil
C. former Prime Minister Shri V.P. Singh
servants.
D. former Prime Minister Dr. Manmohan Singh
D. a framework within which people who lack
political authority conduct their lives. 22. Good Governance as “the manner in which power is
exercised in the management of a country’s economic
15. Why did the term ‘civil society’ become popular in and social resources for development”. This definition
the 1980s? is made by:
A. At that time there was growing disenchantment A. 1992 report entitled “G overnance and
with the perceived dishonesty of governments, Development” of the World Bank
especially as recipients of aid B. 1992 report entitled “G overnance and
B. The downfall of many totalitarian regimes was Development” of the IMF
attributed to movements generated by civil society C. 1992 report entitled “G overnance and
C. It seemed to offer the possibility of peaceful Development” of African Union
change in other states D. None of these
D. All of the above
23. The Principles of Good Governance defined by United
16. Why has the term ‘civil society’ aroused suspicion in Nations, are mainly on:
some Islamic states? 1. Participation, Rule of Law
A. In those countries the state denies civil rights to 2. Consensus Oriented, Equity and Inclusiveness
its citizens 3. Effectiveness and Efficiency
B. It can be seen as part of a project of 4. Accountability, Transparency & Responsiveness
‘Westernization’
C. The term is difficult to translate into Arabic Select the correct codes:
D. It failed to recognize that brutal violence is the A. 1, 2 & 3 B. 2, 3 & 4
only way to facilitate change C. 1, 3 & 4 D. 1, 2, 3 & 4

17. The periphery of participation should enlarge to 24. The Initiatives for Good Governance in India are mainly
include: on:
A. the civil society B. the state 1. Right to Information
C. the sovereignty D. None of these 2. E-Governance
3. Ease of Doing Business
18. Good government should aim at sustainable human 4. Decentralization
development, participation of civil society 5. Police Reforms
organizations and fair and free elections. The statement
is made by: Select the correct codes:
A. ILO B. General Assembly A. 1, 2, 3 & 5 B. 2, 3, 4 & 5
C. UNDP D. IMF C. 1, 3, 4 & 5 D. 1, 2, 3, 4 & 5
38 Political Science

25. The Model Police Act as a part of good governance Select the correct codes:
initiatives was implemented in the year: A. 1, 3 & 5 B. 2, 3, 4 & 5
A. 2015 B. 2017 C. 1, 3, 4 & 5 D. 1, 2, 3, 4 & 5
C. 2018 D. 2019
31. The Chairman of First Administrative Reforms
26. The Aspirational Districts Programme (ADP) was commission:
launched in January 2018 to: A. Venkidachellaya B. Fasal Ali
1. transform the lives of people in the under- C. Hanumanthaya D. None of these
developed areas of the county
32. MGNREGA stands for:
2. Anchored in NITI Aayog, the programme is aimed
A. Mahatma Gandhi National Rural Employment
at transforming 115 most backward district
Guarantee Act.
3. Focused interventions in the field of health and
B. National Rural Integration Progremme.
nutrition,
C. A project under UN
4. Focused interventions in education, agriculture
D. None of these
and water management,
5. Focused interventions in financial inclusion and 33. Sevottam associated with:
skill development A. Ombdusman
B. Lok Ayukta
Select the correct codes:
C. Grievances Redress Cell
A. 1 & 2 B. 4 & 3
D. Supreme Court
C. 3 & 5 D. 1, 2, 3, 4 & 5
34. In India Omudsman is known as:
27. The Good Governance Index was launched on the
A. Sessions Judge B. Municif Judge
occasion of Good Governance Day on:
C. Lok Ayukta D. Lok Pal
A. 25 December, 2019 B. 25 December, 2020
C. 25 December, 2018 D. 25 December, 2017 35. In which one of the following kinds of a public
28. The Good Governance Index is a uniform tool across undertaking ministerial control is all pervasive?
A. Government company
States to assess:
B. Departmental undertakings
1. the Status of Governance and impact of various
C. Government corporation
interventions taken up by the State Government
D. Joint undertaking
2. the Status of Governance and impact of various
interventions taken up by the Union Territories 36. Of all controls over public administration the one
3. to provide quantifiable data to compare the state which is more continuous and self-corrective is:
of governance in all states and Union Territories A. Control by the legislature
4. to improve governance and shift to result oriented B. Control by the executive
approaches and administration. C. Control by the judiciary
Select the correct codes: D. Control by the media
A. 1, 2 & 3 B. 2, 3 & 4 37. What is about Accountability and the Right to
C. 1, 3 & 4 D. 1, 2, 3 & 4 Information Act in governance?
29. Objective of probity in governance: 1. The Right to Information A ct, 2005 has
1. To ensure accountability in governance introduced a huge element of transparency in the
2. To maintain integrity in Public Service decision-making in the government
3. To ensure compliance with the process 2. Digitisation of various services to the citizens has
4. To preserve public confidence in Govt process not only facilitated faster delivery of services
5. To avoid the potential for misconduct, fraud, and 3. It also provided a clear trail of transactions for
corruption any analysis by an oversight agency.

Select the correct codes: Select the correct codes:


A. 1, 2, 3 & 4 B. 2, 3, 4 & 5 A. 1 & 2 B. 2 & 3
C. 1, 3 & 5 D. 1, 2, 3, 4 & 5 C. 1 & 3 D. 1, 2 & 3
30. The ethical concern of governance has been given in 38. Accountability and the Citizen’s Charter are meant
both eastern and western literature like: for:
1. Bhagavad Gita 2. Arthashastra 1. The Citizen’s Charter also clearly spelt out the
3. Confucius 4. Plato responsibilities of various agencies of the
5. Mill, etc government.
Political Science 39
2. The Citizens Charter of the Central Board of Direct Select the correct codes:
Taxes (CBDT) provides for inter-alia service A. 1, 2, 3 & 4 B. 2, 3 & 4
delivery standards C. 1, 2 & 3 D. 1, 3 & 4
3. Includes the specified timelines for issue of refunds
44. The major Legislative Control over executive are
or redressal of grievances
mainly on:
Select the correct codes: 1. Question Hour
A. 1 & 2 B. 2 & 3 2. Adjournment Debates
C. 1 & 3 D. 1, 2 & 3 3. Debates on Enactment of Acts and Amendments
39. The role of Vigilance in Accountability: 4. Budget Discussion
1. The institutional mechanisms existence of strong 5. President’s Speech
and independent accountability institutions is a 6. Audit
necessary condition for good governance. Select the correct codes:
2. These institutions can detect the violations and A. 1, 2, 3, 5 & 6 B. 1, 2, 3, 4, 5 & 6
deviations from prescribed norms as well as the C. 1, 2, 3, 4 & 5 D. 2, 3, 5 & 6
instances of poor administration
3. They can indicate the abuse of power and 45. The Administrative control over the executive are
unconstitutional conduct mainly on:
1. The administration performs the policymaking
Select the correct codes: function concerning finance
A. 1 & 2 B. 2 & 3 2. Government decides the policy of expenditure
C. 1 & 3 D. 1, 2 & 3 3. Acceptance of provision in the budget estimates
40. Which of the following article of the Indian 4. Questions like pay, pension and provident fund
Constitution says, the Union Budget of a year is to the officials are all determ ined by the
referred to as the Annual Financial Statement (AFS)? government
A. Article 112 B. Article 114 Select the correct codes:
C. Article 115 D. Article 116 A. 1, 2 & 4 B. 1, 2, 3 & 4
41. Judicial control over administration refers to: C. 1, 2 & 3 D. 2, 3 & 4
A. The control exercised by the Courts over the 46. The Administrative Reforms Commission in its report
administration is called judicial control on “Finance Accounts and Audit” highlighted the
B. The power of the court to keep the administrative problem and observed that: “The Control of the
acts within the limits of law Finance Minister” over public expenditure is exercised,
C. It also implies the right of an aggrieved citizen to in the main, three stages: What are these?
challenge the wrongful act of administration in 1. Approval of programmes or policies in principle
the court of law 2. Acceptance of provision in the budget estimates,
D. The primary purpose of judicial control over and
administration is the protection of the rights and 3. Prior sanction to incurring of expenditure subject
liberty of citizens to such power as have been delegated to the
42. Which among the following Instruments of Checks & administrative ministries
Balances are in the democratic set up in the country? 4. Administration has to provide the machinery
1. Legislature 2. Executive Select the correct codes:
3. Judiciary 4. Civil society groups A. 1, 2 & 3 B. 1, 2, 3 & 4
Select the correct codes: C. 2, 3 & 4 D. 1, 3 & 4
A. 1 & 2 B. 2 & 3 47. The advantages of Budgetary Control System refers
C. 1, 2 & 3 D. 1, 3 & 4 to:
43. The Legislative control over executive are mainly on: 1. The extent by which actual results have varied
1. Power to assess works of the executive through from defined objective
the question hour and zero hour 2. The centralized control with delegated
2. Impeachment of the President responsibility
3. The executive acting through Ministers cannot 3. Budget under various functional units indicates
raise money by taxation the limits for expenses and results to be achieved
4. The executive borrowing or otherwise without 4. Budgetary system helps managers to learn from
the authority of Parliament past experience
40 Political Science

Select the correct codes: 55. The Right to Information in India is governed by two
A. 1, 2 & 3 B. 1, 2, 3 & 4 major bodies. What are these?
C. 1, 2 & 4 D. 2, 3 & 4 1. Central Information Commission (CIC)
2. State Information Commissions (SIC)
48. Control through Parliamentary Committees refers
3. Consumer Commission (CC)
to:
4. National Human Rights Commission (NHRC)
1. control over revenue
2. expenditure Select the correct codes:
3. borrowing A. 1 & 2 B. 1 & 4
4. accounts C. 2 & 3 D. 3 & 4
Select the correct codes: 56. Which of the following porovisions regarding
A. 1, 2 & 3 B. 1, 2, 3 & 4 “National Water Policy, 2012” is/are correct?
C. 1, 2 & 4 D. 2, 3 & 4 1. Envisages to establish a standardized national
information system with a network of data banks
49. Judicial control over legislature and executive are and data bases
mainly on: 2. Guidelines for the safety of storage dams and
A. Impeachment and the removal of the judges other water-related structures.
B. Unamendability of the constitution under the basic 3. Regulate exploitation of groundwater
structure doctrine 4. Resource planning and recycling for providing
C. The pow er to review executive action to maximum availability.
determine if it violates the Constitution
Select the correct codes:
D. All of these
A. 1, 2, 3 & 4 B. 2, 3 & 4
50. Administrative culture means: C. 1, 2 & 3 D. 1, 3 & 4
1. improving the Performance and Integrity of the
57. State Public Information Officer is directly under the
Public Services
corresponding:
2. the civil servants are to adhere to ethical standards
A. State Governor
3. committed to basic principles of the Constitution
B. State Chief Minister
4. to regulate the relationship between the politicians
C. Cabinet Ministers
and the civil servants
D. President of India
Select the correct codes:
58. Consumer Protection Act aims at:
A. 1, 2 & 3 B. 1, 2, 3 & 4 1. to provide better and all-round protection to
C. 1, 2 & 4 D. 2, 3 & 4 consumers
51. Which of the following measures were taken up by the 2. applies to the whole of India
GOI in administrative reforms is/are correct? 3. applies to all goods and services unless otherwise
A. Secretariat Reorganization Committee, 1947 expressly notified by the Central Government
B. Shri N. Gopalaswamy Ayyangar Report, 1950 4. has no jurisdiction over the State Information
C. A.D. Gorwala Committee, 1951 Commission
D. Committee on Plan Projects, 1958 Select the correct codes:
52. Who was the chairman of the Administrative Reforms A. 1, 2 & 3 B. 1, 2 & 4
Commission (ARC), 1966? C. 2, 3 & 4 D. 3 & 4
A. K Hanumanthaiya B. Veerappa Moily 59. Consumer Protection act:
C. Shri Dharam Vira D. L K Jha 1. provides for a three-tier consumer grievance
53. In which year “The Second Administrative Reforms redressal machinery with the District Forums
Commission” (ARC) was constituted as a Commission 2. provides for a three-tier consumer grievance
of Inquiry, under the Chairmanship of Veerappa Moily? redressal machinery
3. provides for a three-tier consumer grievance
A. 31 August, 2005 B. 30th July, 2007
redressal machinery at the base
C. 30th March, 2007 D. 30th April, 2006
4. provides for a three-tier consumer grievance
54. In which year at the central level the Right to redressal machinery at the middle level
Information Act was passed by the Parliament? 5. provides for a three-tier consumer grievance
A. 2008 B. 2007 redressal machinery at the National Commission
C. 2005 D. 2006 at the apex level
Political Science 41
Select the correct codes: 3. In Sweden, the judiciary is NOT within the purview
A. 1, 2 & 3 B. 1, 2, 3, 4 & 5 of the Ombudsman system
C. 1, 2, 3 & 5 D. 2, 3 & 4
Which of the above statement/s is/are true?
60. In which year “The Consumer Protection act” was A. Only 1 and 3 B. Only 2 and 3
passed? C. Only 2 D. All are true
A. 1986 B. 1987
68. Which was the first Indian state to establish the
C. 1988 D. 1999
institution of Lokayukta?
61. In which among the following act is/are correct? A. Bihar B. Uttar Pradesh
1. The Essential Commodities Act, 1955 C. Andhra Pradesh D. Maharashtra
2. The Prevention of Food Adulteration Act, 1955
69. Which state’s Lokayukta’s office is considered to be
3. MRTP Act, 1970
the strongest in terms of power and scope?
Select the correct codes: A. Bihar B. Karnataka
A. 1 & 2 B. 1, 2 & 3 C. Andhra Pradesh D. Maharashtra
C. 1 & 3 D. 2 & 3
70. When did The Lokpal and Lokayuktas Act, 2013 came
62. In which country The concept of Citizens Charters into force?
originated? A. June 2013 B. May 2013
A. Britain B. France C. December 2013 D. January 2013
C. Poalnd D. Austarlia
71. Who appoints the Lokayukta and Upalokayukta?
63. The Right of Citizens for Time Bound Delivery of A. Governor
Goods and Services and Redressal of their Grievances B. Chief Minister
Bill, 2011 (Citizens Charter): C. Speaker of Lok Sabha
1. Seeks to create a mechanism to ensure timely D. Judge of High Court
delivery of goods and services to citizens
2. It requires every public authority to publish a CC 72. Which of the following committee recommended for
within six months of the commencement of the Act Panchayati Raj System in India?
3. Levies a penalty of up to ` 50,000 for failure to A. Punchhi Samiti
render services B. Balwantrai Mehta Committee
4. Emphasised on citizens as customers C. Singhvi Committee
D. None of the following
Select the correct codes:
A. 1, 2 & 4 B. 1, 2, 3 & 4 73. How many tiers are there in the Panchayati Raj system
C. 1, 2 & 3 D. 2, 3 & 4 of India?
A. One-tier B. Two-tier
64. Which is the oldest known system designed for the
C. Three-tier D. Four-tier
redressal of citizen’s grievance?
A. Ombudsman System B. Lokpal 74. Which of the following system is established on the
C. Lokayukta D. None of the above basis of the direct election?
A. Gram Panchayat B. Block Committee
65. Which was the first Commonwealth country in the
C. Zila Parishad D. Both B and C
world to adopt the Ombudsman system?
A. India B. Sweden 75. Which of the following statements is not correct?
C. New Zealand D. United Kingdom A. Panchayati Raj was established in India by Jawahar
Lal Nehru
66. The ‘Procurator system’ is still running in which of the
B. Madhya Pradesh was the first state which
following countries?
A. India B. Sweden implemented The Panchayati Raj system in India
C. New Zealand D. Russia C. 73rd Constitutional amendment was implemented
in 1992
67. Consider the following three statements: D. Tamil Nadu has adopted the bicameral method
1. The judiciary is within the purview of Lokpal/
Lokayukta in India 76. Which of the following Article is related to Panchayati
2. In New Zealand, the judiciary is NOT within the Raj?
purview of the Parliamentary Commissioner for A. Article 243 B. Article 324
Investigation C. Article 124 D. Article 73
(2751) Pol. Sci.—6
42 Political Science

77. Which schedule was added to the Constitution by the 2. It w as introduced in early 1992’s on the
73rd Constitutional Amendment? recommendation of Brettonwoods commission.
A. 6th B. 7th 3. It w as introduced in early 1980’s on the
C. 9th D. 11th recommendation of Brundtland commission.
4. It means that Present generation should fulfill its
78. How many posts are reserved for women at all levels
needs while considering the needs of the future
in the Panchayati raj system.
generation as well.
A. 1/3 B. 1/2
C. 2/3 D. 1/4 Codes:
A. 1, 2 & 3 B. 1, 2, 3 & 4
79. Who gives the final approval to the five-year plans of
C. 1, 2 & 4 D. 2, 3 & 4
India?
A. National Development Council (NDC) 87. Which among the following component is the
B. Ministry of Finance foundation of participatory development?
C. Planning Commission (now NITI Aayog) A. Good Governance B. Governance
D. President of India C. State D. Judicial review
80. The very first five-year plan of India was based on the 88. The Banking Ombudsman Scheme 1995 was framed
model of: by the Reserve Bank of India under the:
A. Mahalanobis model B. Harrod Domar Model A. Banking Regulations Act, 1949
C. Bombay Plan D. None of these B. Banking Regulations Act, 1959
C. Banking Regulations Act, 1969
81. Which programme gave the slogan of Garibi Hatao?
D. Banking Regulations Act, 1979
A. 7th five-year plan B. 3rd five year plan
C. 6th five year plan D. 5th five-year plan 89. Participatory Development means:
1. It is top-down participation in the sense that the
82. Jawahar Rojgar Yojna was launched in:
management of the project defines where, when
A. 5th five year plan B. 6th five year plan
and how much the people can participate
C. 8th five year plan D. 7th five year plan
2. It is more dominant in terms of resources available
83. Which statement depicts the best definition of 3. It means conventional project practice in a more
sustainable development? participatory and sensitive manner
A. It means optimal utilization of natural resources. 4. It is bottom up participation
B. Sustainable use of natural resources without
Select the correct codes:
considering the need of the future generation.
A. 1 & 2 B. 1, 2, 3 & 4
C. Present generation fulfills its needs while considers C. 1, 2 & 3 D. 3 & 4
the needs of the future generation as well.
D. None of the above 90. Participation in Development means:
1. It is bottom up participation in the sense that the
84. The concept of sustainable developm ent is local people have full control over the processes
recommended by the: and the project provides for necessary flexibility
A. Brundtland Commission 2. It is more prevalent with NGOs then with the
B. Brettonwoods Commission governments
C. World Bank 3. It entails genuine efforts to enage in practices
D. World Economic Forum which openly and radically encourage people’s
85. Which of the following is considered as the three participation
pillars of the sustainability? 4. It is top-down participation
A. Economic development Select the correct codes:
B. Social development A. 1, 2 & 3 B. 1, 2, 3 & 4
C. Environment protection C. 1, 3 & 4 D. 3 & 4
D. All of the above
91. ‘NAMASTE’ is a portal of which Union Ministry of
86. Which of the following statement/s is true about the India?
sustainable development? A. Ministry of Defence
1. It was introduced in early 1980’s in particular B. Ministry of Communications
through the publication of the W orld C. Ministry of AYUSH
Conservation Strategy by IUCN, UNEP and WWF. D. Ministry of External Affairs
Political Science 43
92. The Agriculture trading portal e-NAM (National 2. achieve the objective of ‘Housing for All’ by
Agriculture Market) was launched on April 14, in 2035
which year? 3. achieve the objective of ‘Housing for All’ by
A. 2010 B. 2012 2022
C. 2016 D. 2018 4. achieve the objective of ‘Housing for All’ by
93. Mukhyamantri Yuva Yogayog Yojana’, that was seen 2024
in news, is an initiative of which Indian state/UT? 99. The mission aims to achieve the objective of “Housing
A. Madhya Pradesh B. Uttar Pradesh for All’ by 2022 through its four pillars. What are
C. Haryana D. Tripura these?
94. The Prime Minister recently launched 10000 FPOs all 1. in-situ slum redevelopment
over the country. What does FPO stand for? 2. affordable housing through a credit linked subsidy
A. Farmer Priority Organisations scheme
B. Farmer Produce Organisations 3. affordable housing in partnership between public
C. Farmer Productivity Organisations and private agencies
D. Farmer Primary Organisations 4. subsidy for beneficiary led individual house
construction or enhancement
95. What is the objective of the ‘covid19.odisha.gov.in’
portal launched by Odisha in tackling the crisis faced Codes:
due to Covid-19? A. 1, 2, 3 & 4 B. 1, 2, 3
A. Provide official data on positive cases C. 2 & 4 D. Only 2
B. Registration of migrant workers 100. The ‘Housing for All’ scheme faces the following key
C. Registration for Public Distribution System constraints: Which are correctly matched?
D. Provide tele-medicine facility 1. Lack of access to finance from formal financial
96. Which of the following provision of houses to every institutions.
family is /are correct? 2. Long-drawn out, multi-level approval system in
1. The President’s clarion call to ensure the provision urban areas in a large majority of municipal
of houses to every family jurisdictions.
2. It remains the key objective of the ‘Housing for 3. For several categories of houses or those in
All’ scheme. particular locations, these delays are common even
3. The government has made it clear that one of its where a single window system has reportedly been
key priorities is to ensure safe and affordable introduced
housing for all. 4. Limited private sector participation in affordable
4. This mandate also includes upgradation of slums housing schemes in urban areas.
Codes: Codes:
A. 1 & 2 B. 1, 2, 3 & 4 A. 1, 2, 3 & 4 B. 1, 2, 3
C. 2 & 4 D. Only 2 C. 2 & 4 D. Only 2
97. PMAY-G is converged with Swachh Bharat Mission 101. What was the theme of the economic survey 2021?
(Gramin) and Mahatma Gandhi National Rural A. COVID-19 warriors
Employment Guarantee Scheme (MGNREGS) for the: B. COVID 19 Pandemic
1. construction of sanitary toilets C. Doctors and Health Workers
2. unskilled wage component D. None
3. support provided to homeless families
102. The Swachh Bharat Mission (SBM) was launched on
4. or to those who live in kutcha houses as per the
October 2, 2014, to make India open defecation free
Socio-Economic Caste Census (SECC), 2011 data.
by 2019. It has two sub missions. What are these?
Codes: A. Swachh Bharat Mission (Gramin) for rural areas
A. 1 & 2 B. 1, 2, 3 & 4 under the Ministry of Drinking Water and
C. 2 & 4 D. Only 2 Sanitation (MDWS)
98. Under the Pradhan Mantri Awas Yojana “Urban- B. Swachh Bharat Mission (Urban) for urban areas
PMAY-U), the mission aims to: under the Ministry of Housing and Urban Affairs
1. achieve the objective of ‘Housing for All’ by C. Neither A nor B
2027 D. Both A & B
44 Political Science

103. In which year the government of India launched 107. Which among the following scheme is coming under
“Namami Gange” scheme clean and rejuvenate the Universal Health coverage?
River Ganga to maintain “Aviral” and “Nirmal Dhara” A. PM-JAY under Ayushman Bharat
and ensure its ecological and geological integrity? B. Rashtriya Swasthya Bima Yojana (RSBY)
A. 2014-15 B. 2017-18 C. Pradhan Mantri Bhartiya Janaushadhi Pariyojana
C. 2018-19 D. 2010-11 D. RSBY scheme
104. Under Ayushman Bharat, scale-up a new vision for 108. Which of the following statements regarding Ek Bharat
comprehensive primary health care across the country, Shrestha Bharat programme is/are correct?
built on the platform of health and wellness centres 1. It aims to actively enhance interaction between
(HWCs), to: people of diverse cultures living in different States
1. Provide quality ambulatory services for an and UTs in India.
inclusive package of diagnostic, curative, 2. Ministry of culture is the nodal agency for
rehabilitative and palliative care, close to the coordination of the programme.
people
Select the correct answer using the code given below:
2. Deliver preventive and promotion services, and
A. 1 only B. 2 only
action on the ground to tackle determinants of ill
C. Both 1 and 2 D. Neither 1 nor 2
health locally
3. Create a mass movement for Healthy India (Swasth 109. Recently The State of Food Security and Nutrition in
Bharat Jan Andolan) the World, 2019 was released. In this context consider
4. None of these the following statements: Which of the statements is/
are correct?
Codes:
A. 1, 2, 3 & 4 B. 1, 2 & 3 1. It is a biannual report issued by FAO, IFAD, UNEP,
C. 1, 2 & 4 D. 2, 3 & 4 WEP, and WHO.
2. For the first time, the report comes with estimates
105. The key features of New India’s primary health care of the prevalence of moderate or severe food
system will be: insecurity based on the Food Insecurity
1. Primary health care nucleus comprising five to Experience Scale (FIES).
six upgraded sub-centres 3. The report highlights that hunger is declining.
2. Coupled with a primary health centre, and
population outreach Select the correct code:
3. Planning of health facility distribution in districts A. 1 and 2 only B. 1 and 3 only
4. Digitization of family records and information C. 2 only D. 1 only
from the community to the facility level 110. Which of the following are the features of PM
Codes: Formalization of Micro Food Processing Enterprises
A. 1, 2, 3 & 4 Scheme?
B. 1, 2 & 3 1. Adoption of One District One product Approach
C. 2, 3 & 4 2. Focus on waste to wealth products and minor
D. 1 & 2 forest produce
3. Credit-linked capital subsidy for setting new
106. On the strong platform of Pradhan Mantri – Jan Arogya
micro food processing units
Yojana (PM-JAY), the objectives of the schme are to:
4. Focus on capacity building and research
1. Attain a coverage of at least 75 per cent of the
population with publicly financed health insurance Select the correct answer using the code given below:
(covering most secondary and tertiary care A. 1, 2 and 3 only B. 1, 2 and 4 only
procedures) by 2022-23 C. 1, 3 and 4 only D. 2, 3 and 4 only
2. Reduce by 50 per cent the proportion of 111. Consider the following statements regarding the
households facing catastrophic health expenditure POSHAN Abhiyaan. Which of the statement are
from the current levels correct?
3. Quality of health care delivered to citizens and 1. It is a multi-ministerial convergence mission with
population health outcomes. the vision to ensure attainment of malnutrition
Codes: free India by 2022.
A. 1 & 2 B. 1, 2 & 3 2. The Ministry of Health and Family Welfare
C. 2 & 3 D. 1 & 3 (MoHFW) is implementing POSHAN Abhiyaan.
Political Science 45
3. Its objective is to reduce stunting in identified Select the correct code:
Districts of India with the highest malnutrition A. 1 only B. 2 only
burden by improving utilization of key C. Both 1 and 2 D. Neither 1 nor 2
Anganwadi Services and improving the quality
116. Consider the following statements regarding the
of Anganwadi Services delivery.
Bharatiya Poshan Krishi Kosh (BPKK): Which of the
Select the correct code: statements is/are correct?
A. 1 and 2 only B. 2 and 3 only 1. It was launched by Ministry of Agriculture with
C. 1 and 3 only D. 1, 2 and 3 support from Bill & Melinda Gates Foundation.
112. Consider the following statements regarding Pradhan 2. It aims to reduce malnutrition through a multi-
Mantri Kisan Maan Dhan Yojana (PM-KMY): Which sectoral results-based framework.
of the following statements is/are correct? Select the correct code:
1. It is a Central Sector Scheme administered by the A. 1 only B. 2 only
Department of Finance, Ministry of Finance. C. Both 1 and 2 D. Neither 1 nor 2
2. The Life Insurance Corporation of India (LIC) is
117. With reference to Shala Darpan Portal, consider the
the Pension Fund Manager for the scheme and
following statements: Which of the statements are
responsible for Pension pay out.
correct?
Select the correct code: 1. It provides an integrated platform to meet the
A. 1 only B. 2 only educational and administrative needs of all
C. Both 1 and 2 D. Neither 1 nor 2 stakeholders.
113. Consider the following statements about Jan Soochna 2. It has been launched by Union Ministry of Human
Portal. Which of the statements is/are correct? Resource Development for Kendriya Vidyalaya
1. It is the first ever public information portal Samiti.
launched by the government of Rajasthan. 3. It is a duplex communication system capable of
2. It will not require citizens to submit RTI sending and receiving effective dialogue between
application for seeking information. every stakeholder.
3. The portal will prevent anomalies and corruption Select the correct code:
in government. A. 1 and 2 only B. 2 and 3 only
Select the correct code: C. 1 and 3 only D. 1, 2 and 3
A. 1, 2 and 3 B. 1 only 118. Which of the following statements is/are correct
C. 2 and 3 only D. 2 only regarding Swachh Survekshan?
114. Consider the following statements regarding National 1. Ministry of Drinking Water and Sanitation
Nutrition Survey. Which of the statements is/are correct? released Swachh Survekshan League in New
1. Comprehensive national nutrition survey was Delhi.
conducted by Ministry of women and child 2. In the category of population between one lakh
development (MoWCD) in partnership with and 10 lakhs, Jamshedpur occupied the 1st spot.
UNICEF. Select the correct answer using the code given below:
2. Clinical development services was the monitoring A. 1 only B. 2 only
agency for the survey.
C. Both 1 and 2 D. Neither 1 nor 2
Select the correct code:
119. Consider the following statem ents regarding
A. 1 only B. 2 only
Sustainable Development Goals India Index: Which
C. Both 1 and 2 D. Neither 1 nor 2
of the statements is/are correct?
115. Consider the following statements regarding the Right 1. Ministry of Health and Family Welfare has
to Information Act (RTI) 2005. Which of the statements released the second edition of the Sustainable
is/are correct? Development Goals (SDG) India Index.
1. A non-government organization financed 2. India’s composite score improved from 57 in 2018
indirectly by the funds of Government is a public to 60 in 2019-20
authority under the act.
Select the correct code:
2. While the institution of the Supreme Court falls
A. 1 only B. 2 only
under the ambit of the act, the office of Chief
C. Both 1 and 2 D. Neither 1 nor 2
Justice of India (CJI) has been exempted.
46 Political Science

120. Consider the following statements regarding ‘Swachh 3. ECA incentivises storage infrastructure
Paryatan’ mobile app: Which of the statements are development
correct? Select the correct code:
1. The app facilitates general public to communicate
A. 1 and 2 only B. 1 and 3 only
their complaints about any unclean area/garbage
C. 3 only D. 1, 2 and 3
piles in and around tourist destinations.
2. It has been launched by Ministry of Drinking 125. The Mahatma Gandhi National Rural Employment
Water and Sanitation. Guarantee Act (MGNREGA) is the world’s largest:
3. Only the Adarsh Smarak Monuments have been A. Anti-poverty programme
identified for inclusion in this app. B. Housing for all Program
A. 1 and 2 only B. 2 and 3 only C. Water for all Program
C. 1 and 3 only D. 1, 2 and 3 D. None of these
121. The 3rd version of Bhuvan Panchayat web portal was 126. In the cash benefits transferred in 2019-20 under the
recently launched. Consider the following statements Direct Benefits Transfer Scheme, the largest head under
in this regard: Which of the statements is/are correct? which DBT transfers were made was:
1. It aims to provide geo-spatial services to aid A. Fertilizer Subsidy
gram panchayat development planning process. B. PAHAL – LPG Subsidy
2. It has been launched by Ministry of Panchayati C. MGNREGA
Raj. D. Farmer compensations for drought, calamity, etc.
Select correct code: 127. Which of the following is/are true regarding Pradhan
A. 1 only B. 2 only Mantri Awaas Yojana — Gramin?
C. Both 1 and 2 D. Neither 1 nor 2 1. Cost of unit assistance to be shared between
Central and State Governments in the ratio 75:25
122. Consider the following statements regarding UMANG
in plain areas.
App: Which of the above statements is/are correct?
2. The beneficiary will be entitled to 90 days of
1. It is developed by National e-Governance Division
unskilled labour from MGNREGA.
(NeGD) to fast-track mobile governance in India.
2. It provides major services offered by only Central Select the correct code:
and State Government departments. A. Only 1 B. Only 2
3. Presently, it supports only English language. C. Both 1 and 2 D. Neither 1 nor 2
Select the correct code: 128. National Mission for a Green India is merged with:
A. 1 only B. 1 and 2 only A. Swachha Bharat Abhiyan
C. 2 and 3 only D. 1, 2 and 3 B. National River Conservation scheme
C. National Afforestation Program
123. Consider the following statements regarding Good
D. MGNREGA
Governance Index (GGI): Which of the above
statements is/are correct? 129. Which of the following statements are correct?
1. Ministry of Housing and Urban Affairs has 1. MGNREGS fulfills the constitutional rights in
launched the ‘Good Governance Index’. India.
2. The objectives of GGI are to provide quantifiable 2. MGNREGA is to be implemented mainly by gram
data to compare the state of governance in all panchayats.
states and UTs. 3. The places where the recent unseasonal rain and
hailstorms have affected crops the government
Select the correct code:
will increase the number of work days under the
A. 1 only B. 2 only
C. Both 1 and 2 D. Neither 1 nor 2 Mahatma Gandhi National Rural Employment
Guarantee Scheme to 200 from 100.
124. Essential Commodities Act (ECA), 1955 was in news
recently. Which of the following statement(s) is/are Select the correct code:
incorrect about ECA? A. 1 & 2 B. 1 & 3
1. ECA was enacted to ensure the easy availability C. 2 & 3 D. 1, 2 & 3
of essential commodities to consumers. 130. With reference to National Rural Health Mission
2. Government intervention through ECA can (NRHM), which of the following are the jobs of ‘ASHA’
increase price volatility of essential Commodities a trained community health worker?
Political Science 47
1. Accompanying women to the health facility for 137. Which of the following district topped the rank of
antenatal care checkup Aspirational district for the period of June 1, 2018,
2. Using pregnancy test kits for early detection of and October 31, 2018.
pregnancy A. Virudhunagar (Tamil Nadu)
3. Providing information on nutrition and B. Mewat (Haryana)
immunization C. Pakur (Jharkhand)
4. Conducting the delivery of baby D. Hailakandi (Assam)
Select the correct answer using the codes given below: 138. Which of the following is true about the Aspirational
A. 1, 2 and 3 Only B. 2 and 4 Only district initiative of NITI Aayog?
C. 1 and 3 Only D. 1, 2, 3 and 4 A. Aspirational ranking of districts is based on 10
131. Who is the current CEO of the NITI Aayog? parameters
A. Rajeev Kumar B. Arvind Subramanyam B. The first delta ranking for aspirational districts
C. Amitabh Kant D. Narendra Modi was released in June 2018.
132. Which of the following statement is correct about the C. The aspirational district programme was launched
NITI Aayog? by Prime Minister Narendra Modi on January 5,
A. NITI Aayog was on formed 25 January, 2016 2016.
B. NITI A ayog comes under the M inistry of D. All of the above statements are true
Commerce and Industry 139. Which of the following is not matched correctly about
C. The full form of NITI Aayog is National Institute NITI Aayog?
for Transforming India A. NITI Aayog head office: Delhi
D. The NITI Aayog is a policy think tank of the B. Narendra Modi: Chairman
Government of India
C. Parent Agency: Government of India
133. When was NITI Aayog established? D. NITI Aayog replaced: National Development
A. 1 January, 2015 B. 25 January, 2016 Council
C. 14 May, 2014 D. 1 July, 2015
140. Right to Education Act (RTE) provided free and
134. Which of the following statement is/are correct about compulsory education to children in 2009 and enforced
the NITI Aayog? it as a fundamental right under which article?
1. The aim of NITI Aayog is to achieve Sustainable A. Article 21-A
Development Goals and to enhance cooperative B. Article 22-A
federalism in the country. C. Article 23-A
2. The Prime Minister of India is the ex-officio D. Article 24-A
Chairperson of the NITI Aayog
3. There are 8 full time members in the NITI Aayog 141. In which part Originally, Article 45 and Article 39 (f)
of DPSP, had a provision for state funded as well as
Select the correct code: equitable and accessible education?
A. Only 2 & 3 B. Only 2 A. Part IV of Indian Constitution
C. Only 1 D. Only 1 & 2
B. Part III of Indian Constitution
135. Who of the following is the part of Governing Council C. Part II of Indian Constitution
of NITI Aayog? D. None of these
A. All Chief Ministers of the states
142. The first official document on the Right to Education
B. Chief Ministers of Delhi and Puducherry
was with which of the following committee?
C. Lieutenant Governor of Andaman & Nicobar Island
D. All of the above A. Ramamurti Committee Report in 1990
B. R.S. Sarkaria Commission
136. Which of the following is not the function of the NITI C. Tapas Majumdar Committee
Aayog? D. None of these
A. To evolve a shared vision of national development
priorities sectors and strategies 143. In which of the following Committee (1999) was set
B. To foster cooperative federalism up, which encompassed insertion of Article 21A.
C. To give final approval to the Five Year Plans of A. Ramamurti Committee
India B. Tapas Majumdar Committee
D. To create a know ledge, innovation and C. Both A & B
entrepreneurial support system D. Neither A nor B
48 Political Science

144. The Government has also launched an integrated 145. Which among the following achievements of Right to
scheme, for school education named as Samagra Education Act, 2009 is/are correct?
Shiksha Abhiyan, which subsumes the three schemes 1. The RTE Act has successfully managed to increase
of school education: which of the following is/are enrolment in the upper primary level (Class 6-8)
correct? 2. Stricter infrastructure norms resulted in improved
1. Sarva Shiksha Abhiyan (SSA) school infrastructure, especially in rural areas
2. Rashtriya Madhyamik Shiksha Abhiyan (RMSA) 3. More than 3.3 million students secured admission
3. Centrally Sponsored Schem e on Teacher under 25% quota norm under RTE
Education (CSSTE) 4. It made education inclusive and accessible
nationwide
Select the correct code: Select the correct code:
A. Only 2 & 3 B. 1, 2 & 3 A. Only 2 & 3 B. 1, 2, 3 & 4
C. Only 1 D. Only 1 & 2 C. Only 1 D. Only 1 & 2

ANSWERS
1 2 3 4 5 6 7 8 9 10
A B D D A D D A D D
11 12 13 14 15 16 17 18 19 20
A B D D D B A C D D
21 22 23 24 25 26 27 28 29 30
A A D D A D A D D D
31 32 33 34 35 36 37 38 39 40
C A C C B B D D D A
41 42 43 44 45 46 47 48 49 50
A C A B B A B B D B
51 52 53 54 55 56 57 58 59 60
D A A C A A A A B A
61 62 63 64 65 66 67 68 69 70
A A C A C D C D B D
71 72 73 74 75 76 77 78 79 80
A B C A B A D A A B
81 82 83 84 85 86 87 88 89 90
D D C A D B A A A A
91 92 93 94 95 96 97 98 99 100
C C D B B B B C A A
101 102 103 104 105 106 107 108 109 110
A D A B A B A A C B
111 112 113 114 115 116 117 118 119 120
C B A B A D C B B C
121 122 123 124 125 126 127 128 129 130
A A B C A B B D A A
131 132 133 134 135 136 137 138 139 140
C D A D D C A B D A
141 142 143 144 145
A A B B B
  
Political Science 49

Comparative World Politics

1. According to which theory, participation represents 8. Which of the following personalities are associated
the process by which goals are set and means are with Reformation, the great religious movement of the
chosen in relation to all sorts of social issues? sixteenth century Europe which resulted in the
A. Socialist theory establishment of Protestantism?
B. Marxist theory A. Martin Luther B. Machiavelli
C. Democratic theory C. Louis XIVth D. Garibaldi
D. None of the above
9. The socio-cultural economic force/forces which made
2. Which of the following are the major types of cardinal contribution of the birth and development of
participation, exercised by citizens in the decision- liberalism were:
making process in government and administration? A. Renaissance and Reformation
1. Democratic B. Enlightenment
2. Administrative C. Scientific and Industrial Revolutions
3. Social D. All of the above
4. Community Development
10. The idea of ‘possessive individualism’ is given by:
Select the correct answer using the codes given below: A. Hannah Arendt
Codes: B. C.B. Macpherson
A. 1, 2 and 3 B. 2, 3 and 4 C. Karl Popper
C. 1, 3 and 4 D. 1, 2, 3 and 4 D. None of the above
3. Which book published in 1908, is the first study of 11. A theory of ‘social engineering’ or ‘piecemeal
pressure groups? engineering’ is advanced by:
A. The Process of Government A. Ernest Gellner B. L.T. Hobhouse
B. Comparative Politics C. Charles Taylor D. Karl Popper
C. The Politics of the Developing Areas
12. Who among the following attached utmost importance
D. Political Behaviour
to human dignity?
4. Who among the following made the famous statement: A. Hegel B. Kant
“England does not love coalitions”? C. Marx D. Green
A. K.C. Wheare B. Bertrand Russell
13. Which of the following is not a dimension of the
C. Disraeli D. Churchill
‘felicific calculus’ propounded by Bentham?
5. Coalition governments continue to be: A. Intensity B. Duration
A. Stable B. Undemocratic C. Probability D. Purity
C. Unstable D. None of the above
14. Contributing significantly to the doctrine of
6. Who initiated the systematic study of pressure utilitarianism, he said, “each to count as one, and no
groups? one for more than one”. He is:
A. Powell B. Lenin A. J.S. Mill B. Jeremy Bentham
C. Grazia D. Bentley C. James Mill D. David Hume
7. Which of the following is not the main feature of the 15. Which of the following pairs is not correctly matched?
economic philosophy of sarvodaya as emphasised by A. Democratic theory : C.B. Macpherson
Vinobaji? B. Freedom : Maurice Cranston
A. Elimination of poverty C. The Tyranny of : L.T. Hobhouse
B. Helping world peace Majority
C. Cooperation of all political parties D. Problems of : D.D. Rapheal
D. Revival of the old social order Political Philosophy
49
(2751) Pol. Sci.—7
50 Political Science

16. Which of the following are corollaries of the rule of List-I List-II
law? (Forms of (Description)
1. Ruling elite of society must adhere to the dictates Democracy)
of law and steer clear of any arbitrariness in (a) Social 1. It is basically a Guild socialist
governance. Democracy idea and Cole was its expo-
2. Law of the land should be properly notified so nent. Its aim is the manage-
that the citizens know as to how it will effect ment of industry by workers
them. on self-governing lives
3. Laws should be ‘general’ in form so that they are (b) People’s 2. Its central idea is economic
uniformly applicable to all the citizens; there Democracy equality. Karl Marx used it in
should be no separate laws for any specific place of political democracy
category of the people. and called it the determining
4. Rights and duties of the citizens and ruler alive factor of politics
should be laid down in the form of a legal code. (c) Industrial 3. It is neither capitalist nor
Democracy socialist. It is partly liberal
Select the correct answer using the codes given below:
and partly socialist. It aims at
Codes: social equality and welfare of
A. 1 and 2 B. 2 and 3 the people.
C. 3 and 4 D. 1, 2 and 3 (d) Economic 4. It is com m only used by
17. Match List-I with List-II and select the correct answer Democracy com m unist states. It is
using the codes given below the lists: intended to put emphasis in
List-I List-II the politico-economic, macro-
(a) A.V. Dicey 1. Law and O pinion in micro and suprainfra nature of
England in the Twentieth their democracy.
Century Codes:
(b) Roscoe Pound 2. Law in a Changing (a) (b) (c) (d)
Society A. 2 4 1 3
(c) Morris Ginsberg 3. Introduction to the Study of B. 4 3 1 2
the law of the Constitution C. 4 1 3 2
(d) W. Friedmann 4. The Spirit of the Common D. 3 4 1 2
law
20. There are certain mechanisms to secure direct
Codes: democracy. Which of the following pairs is/are not
(a) (b) (c) (d) correctly matched in this connection?
A. 3 4 1 2 1. Referendum : A special procedure of referring a
B. 4 3 2 1 particular billon constitutional
C. 3 4 2 1 amendment for popular vote. If a
D. 4 3 1 2 majority votes in its favour, the
18. “Whenever any form of Government becomes bill becomes law
destructive to these ends, it is the Right of the People 2. Recall : A devise to enable to a people’s
to alter or abolish it, and to institute new representative to call the popular
Government.”—The A merican Declaration of assembly consisting of all the adult
Independence, 1776. members of the electorate of his or
her constituency in order to listen
Which of the following is implied in the above to their grievances and redress
statement? them
A. Right to recall 3. Initiative : A devise which enables a specified
B. Right to carry out armed insurrection number of people to draft a bill
C. Right to electoral boycott and send it to the legislature for
D. Right to revolution its consideration. It is a positive
19. Match List-I (Forms of Democracy) with List-II power in the hands of the people
(Description) and choose the correct answer from the to initiate a law which they desire
codes given below the lists: to be passed
(2751) Pol. Sci.—7-II
Political Science 51
4. Plebiscite : A mode of securing opinion of the 23. Assertion (A) : The existence of a single political
people’s representatives in an party is antithetical to democracy.
assembly. It is a negative power of Reason (R) : The essence of democracy is its
the representatives to put a check hospitality to all kinds of opinions
on undesirable legislation by the and so one party state is diametrically
government opposite to the democratic system.
A. Both (A) and (R) are true and (R) is the correct
Select the correct answer from below:
explanation of (A).
A. 1 and 2 B. 2 and 3
C. 2 and 4 D. Only 2 B. Both A and R are true, but R is not the correct
explanation of A.
21. Match List-I (Scholars) with List-II (Works) and select C. A is true, but R is false.
the correct answer from the codes given below the D. A is false, but R is true.
lists:
List-I List-II 24. Match List-I (Confederations) with List-II (Duration)
and select the correct answer from the codes given
(Scholars) (Works)
below the lists:
(a) Political Parties: A Sociolo- 1. Maurice
gical Study of the Oligarchical Duverger List-I List-II
(Confederation) (Duration)
Tendencies of Modern
(a) German Confederation 1. 1991 – Continuing
Democracy
(b) Political Parties: A Genuine 2. Giovanni (b) Swiss Confederacy 2. 1815 – 1866
(c) American Confederacy 3. 1781 – 1789
case for Discontent Sartori
(d) Commonwealth of 4. 1815 – 1840
(c) Parties and Party Systems: 3. Robert
A Frame-work for Analysis Michels Independent States
(d) Political Parties: Their 4. Jean Blondel Codes:
Organisation and Activity (a) (b) (c) (d)
in the Modern State A. 1 4 3 2
Codes: B. 1 3 4 2
C. 2 4 3 1
(a) (b) (c) (d)
D. 2 3 4 1
A. 2 3 1 4
B. 2 3 4 1 25. Though the country is divided into administrative
C. 3 2 1 4 units called ‘Departments’ which are divided into
D. 3 4 2 1 cantons, arrondissements and communes, each having
its organs for local administration, but the general
22. The main characteristics of the party implied in Lenin’s
conceptualization are: opinion is that it is almost misleading to talk about
local governments.
1. The party is in possession of a unique type of
knowledge as well as a distinctive method. Its The country referred to above is:
knowledge base is Marxism and its method is A. USA B. Switzerland
dialectic. C. France D. Italy
2. The party is in principle a rigidly disciplined and
26. Consider the following points about the Presidential
carefully selected sm all group. It claim s
system of government of the American model:
intellectual superiority because of the possession 1. The President and his ministers cannot become
of unique knowledge.
the members of the legislature. In case the
3. The party has to be territorialy all pervasive in
President appoints somebody as his minister who
the sense that its branches should spread in all the is also a member of the legislature, he will have
parts and regions of the country.
to resign his legislative membership before
4. The party is, by design, a highly centralised
accepting the ministerial assignment.
organization. Autonomy of local units is thus 2. Ministers do not attend the sessions of the
ruled out. The rank and file would be subject to
legislature and though they may go and also speak
strict discipline and rules of obedience.
in the House, they cannot vote. Again, the
Select the correct answer from the following: legislature cannot censure the conduct of the
A. 1 and 2 B. 1, 2 and 3 ministers, it can only initiate proceedings of
C. 1, 2 and 4 D. 1, 2, 3 and 4 impeachment against the President.
52 Political Science

3. The President may send his messages to the A. Both (A) and (R) are true and (R) is the correct
legislature and the latter may, and may not, act explanation of (A).
according to the wishes of the Head of the State. B. Both A and R are true, but R is not a correct
4. The bills passed by the legislature are subject to explanation of A.
the veto power of the President who may exercise C. A is true, but R is false.
this power in case of bills not pertaining to his D. A is false, but R is true.
choice. The budget prepared by the President is
submitted to the legislature that may pass it, or 30. Which of the following statements is false?
cut it wherever it so chooses. A. Pressure groups are an important channel of
communication for transmission of political ideas
Which of the above are valid? Select the correct answer from the mass of the citizen to the rulers
from the following: B. Pressure groups are influence groups organised to
A. 1 and 2 B. 2 and 4 influence policies. They attempt to exert influence
C. 1, 2 and 3 D. 1, 2, 3 and 4
over the government or parliament or local
27. Assertion (A) : The most im portant merit of a authorities
parliamentary form of government is C. Pressure groups are non-political entities. They
harmony and cooperation between the are involved in the political process but are not
legislature and executive. willing to assist in forming a government.
Reason (R) : Under the parliamentary system the D. None of these.
acts of omission and commission of
31. Anomic Pressure Groups are those groups.
the government can be criticised
A. Which perform the role of interest articulation by
without any loss of time as the
legislature is sitting most of the time publicising protests in order to exert pressure on
and the ministry or the executive also policy-makers for changing particular policies and
sits in the legislature. There is no decisions
difficulty in asking the questions. B. Which perform the role of interest articulation by
A. Both (A) and (R) are true and (R) is the correct associating themselves with certain political
explanation of (A). parties
B. Both A and R are true, but R is not the correct C. Which perform the role of interest articulation
explanation of A. from the political platform.
C. A is true, but R is false. D. None of the above
D. A is false, but R is true.
32. The Weberian model of bureaucracy has been criticised
28. Assertion (A) : A Presidential system is suitable for from a variety of stand points. Consider the following
emergencies. points in this regard:
Reason (R) : All powers are concentrated in the 1. The Weberian model appears to be a ‘machine
hands of the President and he can theory’ and a closed system model taking little
afford to take prompt decisions to account of organisational interactions with the
meet a crisis. environment.
A. Both (A) and (R) are true and (R) is the correct 2. The structural features of bureaucracy as laid down
explanation of (A). by Weber might be suitable for routine and
B. Both A and R are true, but R is not the correct repetitive tasks but these w ould produce
explanation of A. dysfunctional consequences in terms of human
C. A is true, but R is false. behaviour if the jobs would involve innovation
D. A is false, but R is true.
and creativity.
29. Assertion (A) : A federal system is a poor device as 3. Weberian model has not been found to respond
far as harmonization of national unity adequately to the tasks of socio-economic
with local autonomy is concerned. transformation in the developing countries. These
Reason (R) : In the federal system, there is always tasks require quick changes whereas the capacity
the danger of secession of units from of the Weberian model for adaptation to change
the federal government. As the units is rather slow.
enjoy autonomy, there can develop a 4. Weber’s model is confined to the study of formal
spirit of defiance and independence bureaucratic structure and fails to take notice of
and that can ultimately result in a the informal structure of bureauracy such as
revolt against the federal government. informal relations, informal norms and values, an
Political Science 53
informal power hierarchy and informal power 40. According to the Marxist point of view, politics is
struggles. only a part of:
Which of the above points make a valid criticism of A. Structure B. Substructure
the Weberian model? Select the correct answer from C. Superstructure D. Class structure
below: 41. Marx, Engels and Lenin argued that political systems
A. 1 and 3 B. 2 and 3 should be compared and contrasted with reference to
C. 1, 2 and 3 D. 1, 2, 3 and 4 their respective:
33. Match List-I with List-II and select the correct answer A. Substructures B. Class structures
using the codes given below the lists: C. Superstructures D. Caste structures
List-I List-II 42. Who propounded the Elitist theory?
(a) Kaplan 1. Comparative Politics A. Mosca B. Robert Dahl
(b) Freeman 2. Political Science and Government C. Lasswell D. Catlin
(c) Garner 3. Power and Society 43. Besides common law and the statute law, the English
(d) Gilchrist 4. Principles of Political Science courts apply a certain type of rules which is known as:
Codes: A. Rationality B. Veracity
(a) (b) (c) (d) C. Equity D. None of the above
A. 1 2 3 4 44. In which book Dicey developed the idea of Rule of
B. 2 3 1 4 law, published in 1885?
C. 3 1 2 4 A. The British Constitution
D. 3 1 4 2 B. Law and the Constitution
34. Who among the following is one of the advocates of C. An Introduction to the British Constitution
the Power theory in politics? D. None of the above
A. Catline B. Kaplan 45. Who said that “rule, predominance or supremacy of
C. Lasswell D. All of the above the law” is an outstanding feature of the British
35. Political Socialisation is the process of induction into Constitution?
the political culture and the psychological dimension A. Dicey B. Harvey
of the: C. Tocqueville D. Barther
A. Socio-economic system 46. The distinction between Constitutional law and
B. Political system Administrative law is very clear in countries like:
C. Economic system A. USA B. Germany
D. Social system C. India D. Both (A) and (C)
36. Who defines the political system as the ‘institutions, 47. The doctrine of the Rule of law was enunciated by:
processes and interactions through which values are A. Rawls B. Dicey
authoritatively allocated in a society’? C. Bryce D. Barker
A. Almond B. Easton
C. Aptev D. Pye 48. The law made by the King-in-Council or by the King-
in-Parliament is known as the:
37. Who was one of the first among the modern political A. Common law
scientists to challenge the traditional approaches? B. Statutory law
A. Freud B. Lasswell C. Civil law
C. Bernard Crick D. Charles Merriam D. None of the above
38. Traditional study of politics was dominated by: 49. Which among the following cannot be reckoned as or
A. Behavioural approach associated with political participation?
B. Normative approach A. Citizen’s attitudes, perceptions or knowledge or
C. Marxist approach interest in politics
D. Power approach B. Electoral activities such as voting campaigning
39. Who is the chief exponent of Communication Theory individually or collectively
Approach? C. The activities of professionals such as government
A. Almond B. Karl Deutsch officials, party officials, political candidates etc.
C. Marx D. Coleman D. Both (A) and (C)
54 Political Science

50. Which among the following scholars draw a distinction 60. Who stated that “a realistic survey of the British
between ‘participation’ and ‘mobilization’? Constitution today must begin and end with parties
A. Myron Weiner and discuss them at length in the middle”?
B. Norman H Nie A. K.C. Wheare B. W.B. Munro
C. Sidney Verba C. Ivor Jennings D. None of the above
D. All of the above 61. Who has opined that a political party “is held together,
51. Who elaborated the theory of separation of powers primarily by its ideology and organisation”?
which had a great democratic appeal and influenced A. Laski B. Edmund Burke
the making of American constitution? C. Roucek Huszar D. Carl Friedrich
A. Bentham B. Mill 62. Pressure groups were referred to as ‘anonymous empire’
C. Locke D. Montesquieu by:
52. The statement that “Balance in the structure of society A. MacIver B. Almond
is a pre-requisite to the maintenance of democracy,” C. S.E. Finer D. None of the above
is made by: 63. Who says that victory is “the first commandment of a
A. Gaetano Mosca B. Karl Mannheim political party”?
C. C. Wright Mill D. Ernest Barker A. Alan Ball B. Barker
53. The “Iron Law of Oligarchy” was propounded by: C. Finer D. None of the above
A. James Burnham B. Robert Michels 64. Decentralisation may be:
C. Gaetano Mosca D. Graham Wallas A. Political B. Administrative
54. Who doveloped a model of the democratic process C. Moral D. Both (A) and (B)
and described it as ‘polyarchy’? 65. Without decentralisation ______ is impossible.
A. Giovanni Sartori B. S.M. Lipset A. Comparative administration
C. Robert Dahl D. None of the above B. Financial administration
55. Who has described democracy as “government by the C. Grass-roots administration
poorest, the most ignorant, the most-incapable who D. Personnel administration
are necessarily the most-numerous”? 66. The cabinet under a Presidential system is:
A. Talleyrand B. G.D.H. Cole A. Part of the legislature
C. Lecky D. Bryce B. Controls the lower judiciary
56. Who differentiated parties from authoritarian groups C. A tool of the President
on the basis of the criterion that genuine parties rest D. Both (B) and (C)
upon “formally free recruiting”? 67. Who among the following opines that Federalism,
A. Laski B. Max Weber when successful, has generally been a stage towards a
C. Carl Friedrich D. None of these Unitary government?
57. Who pointed out that, “the party system was in A. Wheare B. Laski
particular the mechanism by which the class state was C. Dicey D. All of the above
transformed into the nation state”? 68. Who defines the term bureaucracy as the systematic
A. Burke B. MacIver organization of tasks and individuals into a pattern
C. Laski D. None of these which can most effectively achieve the ends of
58. In his study of political parties Robert Michels focussed collective efforts?
on: A. Gladden B. Pfiffner
A. Organisation B. Cadre C. Max Weber D. Marx
C. Structure D. Leadership 69. Who among the following identified bureaucracy with
59. The demarcation of parties as those of Left and those institutions and large scale organisations in society?
of Right can be traced in its origin to: A. Karl Marx B. Engels
A. Soviet Political Parties C. Harold Laski D. Marshall E. Dimock
B. American Constituent Assembly 70. Which of these statements about how to study the
C. Custom followed in the legislatures of European American presidency would be supported by an
Continental Countries advocate of institutional analysis?
D. None of the above A. Focus on the occupant
EXAM BITES

This Pdf Is
Downloaded From
www.exambites.in

Visit www.exambites.in for


More Premium Stuffs,Latest
Books,Test Papers,Lectures etc.
jeeneetadda
jeeneetadda_official
jeeneetadda

VISIT NOW !!
Political Science 55
B. Focus on public opinion 79. Who said, ‘The empires of the future will be empires
C. Focus on the office of the mind’?
D. None of these A. Winston Churchill B. Karl Marx
C. Gabriel Almond D. Franklin Roosevelt
71. The behavioural approach emphasizes:
A. Behaviour rather than attitudes 80. An opinion poll is:
B. Individuals rather than instiutions A. A measure of what public opinion would be if
C. Contemporary events rather than history everyone were fully informed about an issue.
D. Social structure rather than rational choices. B. A moderated discussion about a particular topic
among a small group.
72. Which method is associated with the behavioural
C. A lobbying campaign aimed at the general public.
approach?
D. A series of questions asked in a standard way of
A. Sample surveys
a systematic sample of the population.
B. Participant observation
C. Comparative history 81. Which of the following is true for social movements?
D. Elite interviews A. They focus on a specific issue.
B. They operate only at national level.
73. Which is a central concern of the structural approach?
C. They seek to become the government.
A. Power within a group
D. They stand for public office.
B. Relationships between group
C. Group leaders 82. In most communist states, political participation:
D. Group culture A. was more extensive and more regimented than in
liberal democracies.
74. Which of the following comes closest to Almond and B. was more extensive and less regimented than in
Verba’s notion of a civic culture? liberal democracies.
A. A culture in which most people are active in civic C. was less extensive and more regimented than in
affairs liberal democracies.
B. A culture which blends an active majority with a D. was less extensive and less regimented than in
passive minority liberal democracies.
C. A culture which blends an active minority with a
passive majority 83. In most liberal democracies, the most common channel
D. A culture which gives priority to civil over through which interest groups seek to influence public
policy is:
personal affairs
A. the legislature B. the courts
75. According to Almond and Verba, the civic culture: C. the bureaucracy D. the political executive
A. Inhibits the stability of liberal democracies
84. Over the last few decades, the significance of
B. Is unconnected to the stability of liberal
promotional groups in most liberal democracies has:
democracies.
A. increased B. stabilized
C. Contributes to the stability of liberal democracies.
C. declined D. disappeared
D. Is incompatible with the stability of liberal
democracies. 85. Which type of group exerts most influence on the
European Commission?
76. Social capital is based on:
A. Political parties B. Promotional groups
A. Eastern Europe’s ‘third way’
C. Protective groups D. Social movements
B. Widespread acceptance of the market economy
C. Start-up funding for voluntary organizations 86. Over the last few decades, the importance of party
D. A culture of trust and cooperation mem bers in selecting party leaders in liberal
democracies.
77. Which one of these countries is a consociational A. shows no clear trend
democracy? B. has stayed the same
A. Belgium B. U.K. C. has decreased
C. USA D. Sweden D. has increased
78. Which one of the following is not an example of one 87. The purpose of primary elections is to allow a party’s
of Huntington’s civilizations? supporters to:
A. The West B. Islam A. Choose which candidate will represent their party
C. Communism D. Hindu in a general election.
56 Political Science

B. Choose the party leader. 96. According to Weber, the bureaucracy is:
C. Decide on party policy. A. A disciplined hierarchy in which civil servants
D. Ratify changes to the party’s constitution. are subject to the authority of their superior.
88. Michels’ iron law of oligarchy states that: B. A loose network in which work is negotiated
A. Party members always dominate the party leaders through business-style contracts
B. Power within parties rests with a single oligarch C. The personal office of the head of state
C. The pursuit of power is an end in itself D. A device for distributing patronage and the spoils
D. Who says organization, says oligarchy of office.

89. In Western Europe, several green parties have: 97. In a spoils system:
A. Formed a single party majority government A. public servants seek to dilute politicians’
B. Formed a single party minority government proposals.
C. Participated in coalition governments B. public servants are recruited by competitive
D. Repositioned themselves as far right parties examination.
C. elected politicians distribute government jobs to
90. Which of these electoral systems is most commonly
their own supporters.
used in elections to the legislatives of Western European
D. an Ombudsman monitors public appointments.
countries?
A. The alternative vote 98. An Ombudsman is a public official who:
B. Plurality systems A. offers political advice to ministers
C. Majority system B. investigates allegations of maladministration.
D. Party list proportional representation C. makes judgements on administrative law.
91. Which of the following is generally true for D. supervises recruitment of the bureaucracy.
proportional representation? 99. The first Ombudsman was established in:
A. It is used mainly for presidential elections. A. Denmark B. Finland
B. It leads to single-party government C. Norway D. Sweden
C. It is based on single-member consti-tuencies
D. It often leads to government by coalition 100. E-government refers to the use of information and
communication technology in:
92. Which of these countries employs the mixed member A. Administering elections
proportional system? B. Communicating between states
A. Finland B. France C. Providing public services
C. Germany D. United States D. All of the above
93. Duverger argued there was ‘an almost complete 101. Who said, ‘there is only one way to deal with Congress,
correlation’ between: and that is continuously, incessantly and without
A. The plurality method and a two-party system. interruption’?
B. The plurality method and a multiparty system. A. Bill Clinton
C. Proportional representation and a two-party B. Ronald Reagan
system. C. Lyndon Johnson
D. Mixed member proportional representation and a
D. Abraham Lincoln
multiparty system.
102. In Parliamentary government:
94. In the 21st century, turnout in American presidential
A. One person serves as head of both government
elections:
and state.
A. Always increased
B. The cabinet is directly elected by the voters.
B. Usually increased
C. The government cannot be brought down by the
C. Usually decreased
assembly.
D. Always decreased
D. Governing parties emerge from the assembly.
95. A referendum is:
A. A no confidence vote 103. Semi-presidential government employs both:
B. A vote of the assembly on a constitutional issue A. A Prime Minister and a strong elected president.
C. A form of proportional representation B. A Prime Minister and a ceremonial monarch.
D. A vote of the electorate on an issue of public C. Military and civilian heads of government.
policy D. An elected president and a ceremonial Monarch.
Political Science 57
104. Which one of these countries employes semi- 112. Paul Baran’s “The Political Economy of Growth,
presidential government? Monthly Review Press, 1957” work is also very
A. USA B. UK important. Baran argued that the econom ic
C. France D. Germany relationships that existed between Western Europe (and
later Japan and United States) and the rest of the world
105. Substantive representation is when:
A. Representatives give priority to policy issues. were based on:
B. Representatives give priority to serving their party. A. Conflict and exploitation
C. Representatives resemble their constituents in B. Coordination and Monitoring
given characteristics such as gender. C. Social impact assessment
D. Representatives act in the interests of their D. None of these
constituents. 113. Who among the following developed the idea of world
106. Who said, ‘Parliament is not a congress of ambassadors capitalist economy in the “World System Analysis”?
from different and hostile interests ......... but parliament A. Paul Baran
is a deliberative assembly of one nation, that of the B. Andre Gunder Frank
whole’? C. Colin Leys
A. J.S. Mill B. John Locke D. Immanuel Wallerstein
C. James Madison D. Edmund Burke 114. Who among the following developed “New
107. Which one of the following is traditionally seen as a institutionalism” as an approach to the study of
debating legislature? institutions?
A. America’s Congress A. John Meyer B. Colin Leys
B. Britain’s House of Commons C. Petras D. G.W. Domhoff
C. Germany’s Bundesrat 115. Which of the following is not a character of
D. Sweden’s Riksdag Comparative Politics?
108. The incumbency effect refers to: A. Comparative politics is the modern way of
A. The electoral bonus accruing to members standing studying politics
for re-election B. Comparative Politics has a very wide and
B. The tendency for governments to win no- comprehensives cope
confidence votes. C. Comparative Politics depends upon scientific-
C. The growing prevalence of career politicians. empirical methods of study
D. The growing significance of parliamentary D. Comparative politics has a very narrow scope
committees. 116. Which of the following subject is not included in the
109. Which of these functions of legislatures in liberal scope of Comparative Politics?
democracies has generally grown in importance in A. Study of selected structures
recent decades? B. Study of political behaviours
A. Holding emergency debates C. Analysis of the similarities and differences, among
B. Initiating bills political processes.
C. Shaping foreign policy D. Study of political cultures
D. Overseeing (scrutinizing) the executive. 117. Which of the following subject is not included in the
110. ‘Stages of Economic Growth: A Non-Communist scope of Comparative Politics?
Manifesto” was most clearly elaborated in the writings A. Study of all political systems
of: B. Study of environment and infrastructure of politics
A. W.W. Rostow B. Max Weber C. Study of political socialization
C. Talcott Parsons D. Andre Gunder Frank D. Study of selected structures
111. Dependency theory arose in the late fifties and the 118. Decolonization refers:
sixties as an extended critique of which of the A. The transfer of political authority from colonial
following perspective? rulers to indigenous leaders
A. Traditional Perspective B. European countries lacked the wealth
B. Medieval Perspective C. Economic policies were definitely an impetus for
C. Ancient Perspective the start of colonization
D. Modernization Perspective D. European settlers met labour requirements
(2751) Pol. Sci.—8
58 Political Science

119. The Effects of Colonialism are based on which of the 128. State system suffered a setback due to:
following? A. Growing interdependence
A. Slave Trade B. The rise of nuclear weapons
B. Columbian Exchange C. The growing role of public opinion in recent
C. Boost to Mercantilism years
D. All the above D. All the above
120. In which year the General Assembly of United Nations 129. The Welfare State has often been described as a state
adopted its landmark Declaration on the Granting of that takes responsibility in providing:
Independence to Colonial Countries? A. basic economic and social security for its citizens
A. 1960 B. 1987 B. basic economic and cultural integration for its
C. 1970 D. 1980 citizens
121. In which year the General Assembly proclaimed the C. basic economic and civil security for its citizens
“International Decade for the Eradication of D. only basic economic security for its citizens
Colonialism”? 130. What is the amalgamation and rapid unification
A. 1980 B. 1990 between countries identified as?
C. 1960 D. 1970 A. Globalisation B. Liberalisation
122. The debate on Advanced Capitalist State according to C. Socialisation D. Privatisation
Gramsci consists of: 131. Globalisation has improved the living structure of
A. the coercive apparatus comprising of the police, which of the following?
army and judiciary A. All the people
B. the other includes various institutions of civil B. Workers in developing countries
society such as media, church, schools C. People in developed countries
C. power is distributed and shared by many groups D. None of the above
D. Both (A) and (B)
132. Which of the following is the main reason behind the
123. The State in Capitalist Society (1969) is written by investments of MNCs?
whom among the following political scientists? A. To benefit foreign countries
A. Ralph Miliband B. Althusser B. To provide financial support to the country’s
C. Poulantaz D. Marcuse
government
124. Poulantaz (1973) relates the view of State Theory C. For the welfare of underprivileged people
(1973) refers to the major function of capitalism D. To increase the assets and earn profits
namely:
133. In which year Universal adult franchise was introduced
A. stress the ideological and structural mechanisms
in Germany?
B. the reproduction of the capitalist society in its
A. 1919 B. 1915
totality
C. 1920 D. 1930
C. the concepts of ruling class or the power elite
D. None of these 134. The liberal democratic regimes in the developed states
125. Which among the following is not correctly matched? have been categorized as:
A. A.M. Carnoy (1983) - The State and Political A. Totalitarian regimes
Theory B. Democratic regimes
B. R.A. Dahl (1965) - A Preface to Democratic C. Polyarchical regimes
Theory D. None of these
C. S.M. Lipset (1963) - Social Man 135. The liberal democratic or polyarchical regimes are to
D. R. Miliband (1969) - The State in Capitalist be found in which of the following states?
Society A. North America, Western Europe and Australia
126. Edward Said is famous for his theory of: B. Japan and South Africa
A. Orientalism Theory B. Dependency Theory C. Latin America, Poland and South Africa
C. World System theory D. Realistic theory D. Both (A) & (B)
127. The Changing Nature of Post Colonial State referred 136. In which year France introduced universal adult
as: franchise?
A. Consolidation B. Disintegration A. 1924 B. 1987
C. Globalization D. All the above C. 1945 D. 1939
Political Science 59
137. ‘Oligarchy: Participation and Opposition’ is a work A. David Apter B. S.E. Finer
of: C. Edward Shils D. David Apter
A. Robert A. Dahl B. Karl Manheim 145. Alm ond and Powell have also given a triple
C. Burnham D. Vilfredo Pareto classification of political systems based upon structural
138. Lijphart distiniguishes the democratic regimes differences and functions, and cultural secularisation.
between: They are:
A. majority democratic regimes and the pluralist A. Primitive B. Traditional
democratic regimes C. Modern D. All of these
B. majority democratic regimes and the organised 146. Patrimonialism regimes are autocratic or oligarchic
parliamentary lines and exclude which among the following groups?
C. Both (A) and (B) A. lower, middle and upper classes from power
D. None of these B. mobilized modern systems are those based upon
139. Pluralist democratic regimes based on the US model structuralisation
represent: C. primitive bands
A. the separation of power D. None of these
B. the tradition of authoritarianism 147. Authoritarian regimes have been mostly established in
C. the conventions of British Parliament the developing states of:
D. None of these A. Latin America, the Middle East Africa and South
140 ‘Clash of Civilisations’ is a work of who among the East Asia
following political scientists in comparative political B. Developed states of the West like Spain, Portugal
analysis? and Greece,
A. Almond and Powell B. Samuel P. Huntington C. Neither (A) nor (B)
C. S.M. Lipset D. None of these D. Both (A) & (B)

141. The tradition of classification of political regimes goes 148. What does the idea of constitutionalism capture?
back to the Greek city states in the: A. The idea of a limited government
A. 4th Century BC B. 5th Century BC B. Rule of law principle
C. 6th Century BC D. None of these C. Common Welfare
D. None of these
142. Jellenick is a German writer who classified political
regimes into which one of the following categories? 149. Federal government has an agreement and there are
A. monarchical and republican certain essential features that ensure its proper working.
B. democratic and totalitarian They are:
C. primary and representative A. a written constitution
D. unitary and federal B. division of powers
C. independence of judiciary
143. Which among the following is correctly matched D. All the above
according to Classification of Political Regimes?
A. Tutelary democracy — Edward Shils 150. Which among the following is not correctly matched?
B. Aristocracy — Almond & Powell A. A.R., Ball, (1971) — M odern Politics and
C. Plutocracy — Jean Blondel Government
B. Gabriel Almond — Comparative Politics : A
D. Limited Monarchy — Thomas Hobbes
and Powell (1964) Developmental Approach
144. “Political Development in the New States” is a book C. Robert Dahl (1964) — Modern Political Analysis
written by who among the following political scientist? D. Jean Blondel (1970) — Democracy

ANSWERS
1 2 3 4 5 6 7 8 9 10
C D A C C D D A D B
11 12 13 14 15 16 17 18 19 20
D B C B C B A D D C
60 Political Science

21 22 23 24 25 26 27 28 29 30
D C A C C D B A D D
31 32 33 34 35 36 37 38 39 40
A D C D B B B B B C
41 42 43 44 45 46 47 48 49 50
B A C B C D B B D D
51 52 53 54 55 56 57 58 59 60
D B B C C B B D C C
61 62 63 64 65 66 67 68 69 70
C C C D C C C B D C
71 72 73 74 75 76 77 78 79 80
B A B B C D A C A D
81 82 83 84 85 86 87 88 89 90
A A C A C D A B C D
91 92 93 94 95 96 97 98 99 100
D C A B D A C B D C
101 102 103 104 105 106 107 108 109 110
C D A C D D B A D A
111 112 113 114 115 116 117 118 119 120
D A D A A B A A D A
121 122 123 124 125 126 127 128 129 130
B D A B C A D D A A
131 132 133 134 135 136 137 138 139 140
B D A C D C A A B A
141 142 143 144 145 146 147 148 149 150
A A A C D A D A D D

  
Political Science 61

International Relations &


International Organisation
1. Political realism is contrary to the _______ approach A. Kennan B. Lasswell
to international politics. C. Morgenthau D. Watkins
A. Pragmatic B. Legalistic 12. Who gave theoretical orientation to realism?
C. Moralistic D. Both B and C A. Bross B. Burton
2. According to Morgenthau, the laws by which man C. Morgenthau D. Lasswell
moves in the social world are : 13. The two most vital concepts in which the inconsistency
A. Man-made B. Eternal of Morgenthau’s theory is most clearly revealed are:
C. Temporary D. Obscure A. Power and Society
3. Which theory is the result of the behavioural revolution B. Power and Statesmanship
in social sciences? C. Diplomacy and Society
A. The Systems theory B. The Realist theory D. Diplomacy and Statesmanship
C. The Idealist theory D. The Pluralist theory 14. According to Morgenthau, the best means of preserving
4. Who treats equilibrium mainly in the context of peace in a society of sovereign nations is:
balance of power system? A. National interest B. Diplomacy
A. Fox B. Kaplan C. Foreign trade D. Autonomy
C. Liska D. Merriam 15. Who believes that a theory of International politics is
5. Charles McClelland’s concept of system comes from: “but a specific instance of a general theory of politics”?
A. Geography B. Biology A. Tucker B. Morgenthau
C. Economics D. History C. Kennan D. Wasserman
6. Who treats six models of major international system? 16. The new Economic policy of India comprises the
A. Rosenau B. Lasswell various policy measures and changes introduced since:
C. Kaplan D. None of the above A. June 1992 B. July 1991
C. May 1990 D. July 1992
7. According to Kaplan, the most likely transformation
of the balance of power system is into a : 17. What does NAFTA stand for?
A. Bipolar System B. Unipolar System A. North Asian Free Trade Association
C. International System D. None of the above B. North African Free Trade Association
C. North Atlantic Free Trade Association
8. Who produced formulations of the decision-making
D. North American Free Trade Association
theory in 1738?
A. Anthony Downs B. Daniel Bernouli 18. The World Trade Organisation (WTO) is the successor
C. David L. Sills D. Robinson to which one of the following institutional arrangements?
A. World Trade Community
9. Who was the chief exponent of the realist theory?
B. International Trade and Development Association
A. Kennan B. Hans Morgenthau
C. General Agreement on Trade and Tariffs
C. Watkins D. Easton
D. A ssociation for Econom ic and Social
10. Erich Kaufmann wrote in a book that the essence of Reconstruction
the state was:
19. What does TRIMs stand for?
A. Increase of power B. Development of power
A. Trade-Related Investment Measures
C. Display of power D. All of the above
B. Tariff-Related Investment Measures
11. Who said that the study of politics is the study of C. Technology-Related Investment Measures
influence and the influential in his work on politics? D. Training-Related Investment Measures
61
62 Political Science

20. After the New Economic Policy, 1991 who opined 30. Which of the following is not the basic norm of the
that India would recover from her crisis and carry out balance of power system that are clear to each of the
adjustment programmes within a short duration itself, state actors engaged in the process of balancing?
unlike other economies in a similar situation? A. Any actor or coalition that tries to assume
A. America B. World Bank dominance must be constrained.
C. United Nations D. None of the above B. States want to increase their capabilities by
21. As a part of SAP (Structural Adjustment Policies), the acquiring territory, increasing their population or
economy was being ______ i.e., opened up to foreign developing economically.
goods and capital. C. Negotiating is better than fighting.
A. Systematised B. Globalised D. Other states are viewed as potential enemies.
C. Capitalised D. None of the above 31. Match List-I with List-II and select the correct answer
22. Who said that due to New Economic Policy India will from the codes given below:
find itself at the receiving end? List-I List-II
A. Sethi B. Khushro (Scholar) (Statement)
C. Jagmohan D. Halleiner (a) McLean 1. It is not so much the absolute
and McMillan power of a state as its power
23. Which of the following is the largest open market in position in relation to other
the world? states that counts.
A. ECM B. TRIPs (b) Morgenthau 2. Defined power as the capacity
C. NAFTA D. TRIMs to impose one’s will on others
24. Which of the following countries has open economy? by reliance on effective
A. Hong Kong B. Taiwan sanctions in the case of non-
C. Singapore D. All of these compliance.
25. Which act was passed in India to prevent restrictive (c) George 3. Defined power as man’s control
trade practices? Schwarzer- over the minds and actions of
A. GATT B. MRTP berger other men.
C. MFA D. TRIMs (d) Van Dyke 4. Power is the ability to make
people do what they would not
26. Which agreement encompasses a comprehensive otherwise have done.
overhaul of GATT’S dispute settlements, rules and
procedures aimed at ensuring the prompt and efficient Codes:
resolution of disputes? (a) (b) (c) (d)
A. Seattle agreement B. Durban agreement A. 4 3 2 1
C. Montreal agreement D. None of the above B. 4 3 1 2
C. 4 1 2 3
27. Who among the following personalities coined the D. 1 2 3 4
phrase ‘global village’?
A. James Rosenau B. Fucuyama 32. Given below are two statements, one labelled as
C. Ledley Bull D. Marshall McLuhan Assertion (A) and the other labelled as Reason (R).
Select the correct answer from the codes given
28. Who among the following scholars asserted that – below:
“G lobalization does not simply refer to the Assertion (A) : End of Cold War signified a shift from
objectiveness of increasing inter-connectedness. It also ideological politics in International
refers to cultural and subjective matters [namely, the relations.
scope and depth of consciousness of the world as a Reason (R) : Religious fundamentalism is posing
single place”]?
a threat to security.
A. Raland Robertson B. Romain Rolland
Codes:
C. Ernest Gelner D. None of the above
A. Both (A) and (R) are true, and (R) is the correct
29. Who among the following argued that ‘realism is likely explanation of (A).
to remain the single most useful instrument in our B. Both (A) and (R) are true, but (R) is not the
intellectual toolbox’? correct explanation of (A).
A. Holsti B. E.H. Carr C. (A) is true, but (R) is false.
C. Hans Morgenthau D. Stephen Walt D. (A) is false, but (R) is true.
Political Science 63
33. Match List-I with List-II and select the correct answer 38. According to Iris L Calude Jr. which of the following
from the codes given below: is not a device for the management of power?
List-I List-II A. Arms control B. Collective security
(Writer) (Idea) C. Balance of power D. World Government
(a) Immanual Wallerstein 1. Game theory 39. The modern state system with its territorial sovereignty
(b) Von Newmann 2. End of history came into existence first in Europe in the wake of the
(c) Morgenthau 3. Realism A. Treaty of Versailles B. Congress of Vienna
(d) Francis Fukuyama 4. Dependency theory C. Treaty of Westphalia D. Congress of Berlin
Codes: 40. According to Karen A. Mingrt, which of the following
(a) (b) (c) (d) conditions is not essential to make first generation
A. 2 3 1 4 peacekeeping operations most effective?
B. 3 2 4 1 A. A clear and practical mandate or purpose for the
C. 4 1 3 2 operation.
D. 1 4 2 3 B. US. forces must also be present in the peace-
34. Given below are two statements, one labelled as keeping operations.
Assertion (A) and the other labelled as Reason (R). C. Consent of the parties involved as to the mandate
Select the correct answer from the codes given below: and composition of the force.
Assertion (A) : Non-alignment policy was more D. Strong financial and logistical support of the
relevant during cold war period. members of the UN Security Council.
Reason (R) : Non-aligned countries are divided on 41. Match List-I with List-II and select the correct answer
ethnic and religious differences. from the codes given below:
Codes: List-I List-II
A. Both (A) and (R) are true, and (R) is the correct (Term) (Contents)
explanation of (A). (a) Preventive 1. It is like classical peace-keeping.
B. Both (A) and (R) are true, but (R) is not the diplomacy It refers to the deployment of a
correct explanation of (A). UN force in the field with the
C. (A) is true, but (R) is false. consent of all parties.
D. (A) is false, but (R) is true. (b) Peace making 2. It involves developing social,
35. Given below are two statements, one labelled as political and economic
Assertion (A) and the other labelled as Reason (R). infrastructure to prevent further
Select the correct answer from the codes given below: violence and consolidate peace.
Assertion (A) : American foreign policy supported (c) Peace keeping 3. It involves confidence-building
the democratic regimes. measures, fact finding and
preventive deployment of UN
Reason (R) : At times the military systems were
authorised forces.
also supported by America.
(d) Post-conflict 4. It is designed to bring hostile
Codes:
peace building parties to agreement essentially
A. Both (A) and (R) are true, and (R) is the correct
through peaceful means.
explanation of (A).
B. Both (A) and (R) are true, but (R) is not the Codes:
correct explanation of (A). (a) (b) (c) (d)
C. (A) is true, but (R) is false. A. 3 4 1 2
D. (A) is false, but (R) is true. B. 3 4 2 1
C. 1 2 3 4
36. “Uniting for Peace Resolution” is also called as:
D. 4 3 2 1
A. Truman plan B. American plan
C. Achieson plan D. Eisenhower plan 42. Match List-I with List-II and select the correct answer
from the codes given below :
37. ‘Balance of Power’ is an important principle of:
List-I List-II
A. Liberal thought in international politics.
(Book) (Author)
B. Marxist thought in international politics. (a) International Relations (i) Gerhard Wahlers
C. Traditional thought in international politics. Theory: The Three
D. Realist thought in international politics. Traditions
64 Political Science

(b) The North South (ii) Tadjbaksh 48. Match List-I with List-II and select the correct answer
Divide and the Shakbanon and from the codes given below:
International System Anuradha List-I List-II
Chenoy (a) Counter force (i) Types of nuclear strategy that
(c) Human Security (iii) N.B. Adams strategy threatens assets that are
(d) India and the (iv) Martin Wright valuable to an adversary, such
European Union as cities with industrial assets
and large populations.
Codes:
(a) (b) (c) (d) (b) Counter- (ii) International lawyers who argue
proliferation that there is a legal right of
A. (iv) (iii) (ii) (i)
humanitarian intervention in
B. (ii) (iv) (i) (iii)
C. (iii) (i) (iv) (ii) both U N charter law and
customary international law.
D. (i) (iii) (ii) (iv)
(c) Counter- (iii) Term used to describe a variety
43. Which one of the following can be regarded as the restrictionist of efforts to obstruct slow, or
currency of power? roll back nuclear weapons
A. Food program mes and nuclear
B. Nuclear Weapons proliferation.
C. Industrial Capacity (d) Counter (iv) Type of nuclear strategy that
D. Military Preparedness value targets an adversary’s military
44. Who was of opinion that, “the end of cold war strategy and nuclear capabilities.
represented the victory of ideal state”, and liberal Codes:
capitalism? (a) (b) (c) (d)
A. Michael Doyle B. Thomas Dunne A. (iv) (iii) (ii) (i)
C. Francis Fukuyama D. Samuel Huntington B. (iv) (iii) (i) (ii)
45. Which world leader gave a special concept of C. (ii) (i) (iii) (iv)
‘Development’ after the Second World War? D. (iii) (iv) (i) (ii)
A. Franklin Delano Roosevelt 49. Who is given credit to coin the term—Soft Power?
B. Harry Truman A. Joseph Nye
C. Jawaharlal Nehru B. N. Alarcon
D. Winston Churchill C. A.J. Bellamy
46. Match List-I with List-II and select the correct answer D. S. Chesterman
from the codes given below: 50. According to K.J. Holsti, which one of the following
List-I List-II is not a feature of the contemporary state system?
(Authors) (Books) A. The rise in the number and type of states.
(a) B. Buzan (i) Man, the State and War B. The great potential for destruction by those who
(b) I. Hansen (ii) Social Theory of International possess nuclear weapons and modern delivery
Politics systems.
(c) K.N. Waltz (iii) The Evolution of International C. The predominant position of influences that has
Security Studies been achieved by the three essentially non-
(d) A. Wendt (iv) People, States and Fear European states : Russia, China and the United
Codes: States.
(a) (b) (c) (d) D. The large number of states are keen to join
A. (iv) (iii) (i) (ii) political and economic alliances.
B. (iv) (iii) (ii) (i) 51. The Principal purpose of India’s foreign policy vis-a-
C. (i) (ii) (iii) (iv) vis the UN is to pursue three closely related goals.
D. (iv) (i) (ii) (iii)
Which of the following is not included among them?
47. Which country is the example of Latent Nuclear A. A significant role in the shaping of international
Capacity? relations in the 21st century.
A. India B. Japan B. A movement towards a non-violent and humane
C. China D. North Korea international system.
Political Science 65
C. The promotion of conditions for a sustainable 59. Which of the following Articles of the Charter of
and relatively equitable pattern of international United Nations Organisation are related to Regional
development. Organisations?
D. To help UN to develop its own financial resources. (a) Article 51 (b) Article 52
52. The roots of newly emerging forces of globalisation (c) Article 53 (d) Article 54
have been traced in specific economic and political Select the correct answer from the codes given below:
developments in the late 1980s or early 1990s. Which Codes:
of the following can not be included in this category? A. (b), (c) and (d) B. (a), (c) and (d)
A. The end of Cold War. C. (a), (b) and (d) D. (a), (b) and (c)
B. Testing of nuclear devices by India and Pakistan
in 1998. 60. Which of the following country is not a member of
C. Dismantling of state socialism in the USSR. ASEAN?
D. The collapse of the Berlin wall. A. Laos B. Combodia
C. Vietnam D. North Korea
53. Who among the following is associated w ith
Democratic Peace Theory? 61. Originally what was the total strength of security
A. Kenneth Waltz B. Michael Doyle council of United Nations?
C. Jackson and Sorenson D. John Mearsheimer A. 9 B. 10
C. 11 D. 15
54. Who among the following regards International
Politics as an autonomous discipline? 62. Bachract and Baratz have mentioned about the three
(a) C.A.W. Manning (b) Karlin M. Copper classes of International Relations to control the conduct
(c) Martin Wright (d) Hoffson of states. Which of the following are the three classes?
(a) Influence Relationship
Select the correct answer from the codes given below:
(b) Force Relationship
A. (a), (b) and (c) B. (b), (c) and (d)
(c) Legitimate Relationship
C. (c), (d) and (a) D. (a), (b) and (d)
(d) Power Relationship
55. Which of the following pairs are correctly matched.
Select the correct answer from the codes given below:
Choose your answer from the codes given below:
A. (b), (c) and (d) B. (a), (c) and (d)
(a) The control of the Arms – Hedley Bull
Race C. (d), (c) and (b) D. (b), (c) and (a)
(b) Some Economic Aspects of – L. Gromoo and 63. Which one of the following books was written by E.H.
Disarmament V. Strigachow Carr?
(c) The challenge of the – James P. A. The open society and its enemies
Nineteen sixties Warbarg B. Twenty years crisis
(d) Winning without war – Emile Benoit C. Political Realism and Political Idealism
Codes: D. The Realities of American Foreign Policy
A. (a), (b) and (c) B. (b), (c) and (d) 64. Which of the following pairs is not correctly matched?
C. (d), (c) and (a) D. (b), (d) and (a) Author Book
56. Who among the following is not related to Bargaining A. William : The Growth of a
Theory? C. Olson Discipline
A. Kenneth Waltz B. J.F. Nash B. G.F. Kennan : American Diplomacy
C. Thomas Schelling D. Roger Fisher C. Russel H. : The Introductory Course in
57. Which one of the following was not feature of Old Fifield International Relations
Diplomacy? D. Robert Strausz- : International
A. Limited Hupe and Stefan Relations : A General
B. Aristocratic Possony Theory
C. Transparency 65. Who among the following are supporters of Realistic
D. Freedom of Action for Ambassadors Approach in International Politics?
58. In which year Brezhnev had propounded the concept (a) Couloumbis and Wolfe
of Asian Collective Security System? (b) Fredrick Schuman
A. 1967 B. 1968 (c) Nicholas Spykman
C. 1969 D. 1970 (d) E.H. Carr
(2751) Pol. Sci.—9
66 Political Science

Select the correct answer from the codes given 74. Match List-I with List-II and select the correct answer
below: from the codes given below:
A. (b), (c) and (d) B. (a), (c) and (d) List-I List-II
C. (a), (b) and (d) D. (a), (b) and (c) (Treaties/Proposals) (Year)
(a) SALT II (Strategic Arms (i) 1987
66. Who among the following had declared that arms race
Limitation Talks)
in itself is experience of terror?
(b) PTBT (Partial Test Ban Treaty) (ii) 1979
A. Hadley Bull B. Arnold Toynbee
(c) Atom for Peace Proposal (iii) 1963
C. Kenneth Boulding D. Lynn Miller
(d) INF Treaty (Intermediate (iv) 1953
67. Which of the following countries was a part of recently Nuclear Force Treaty)
concluded P5+1 Nuclear Argument?
Codes:
A. Japan B. Australia
(a) (b) (c) (d)
C. India D. Germany
A. (i) (ii) (iii) (iv)
68. Which of the following is not a form of National B. (ii) (iii) (iv) (i)
Power? C. (iv) (ii) (i) (iii)
A. Social Power D. (iii) (ii) (i) (iv)
B. Military Power
75. Which one of the following factors contributed to the
C. Psychological Power
emergence of ‘New Detente’ after Cold War?
D. Economic Power
A. Pressure by the Non-aligned Movement
69. Which of the following scholars of International B. The signing of INF Treaty by Reagan and
Politics did not subscribe to this view that no social Gorbachev.
science can survive without idealistic aspects? C. Emergence of Namibia as a free African Nation
A. Herbert Butterfield D. Deployment of RDF (Rapid Deployment Force)
B. E.H. Carr in Persian Gulf
C. Reinhold Niebuhr
76. The demand for New International Economic Order
D. Arnold Wolters
was first made at
70. Which of the following is also called as Total A. Tehran Summit
Disarmament? B. Durban Summit
A. Qualitative Disarmament C. Kuala Lumpur Summit
B. Quantitative Disarmament D. Algiers Summit
C. General Disarmament
77. Which one of the following is not correct about
D. Comprehensive Disarmament
Association of South East Asian Nations (ASEAN)?
71. Who stated, “International Politics includes analysis A. First India ASEAN Summit was held in November,
of political relations and problems of peace among 2002.
nations”? B. ASEAN declared South East Asian Nuclear
A. Charles Scheichar B. Norman Padelford Weapon free zone in 1995.
C. Harold Sprout D. H.J. Morgenthau C. ASEAN is working to remove trade barriers and
develop economic relationship with China, Japan
72. Game theory owes a heavy debt to the seminal work
and South Korea.
entitled, “The Theory of Games and Economic
D. India became full dialogue partner of ASEAN in
Behaviour”. It has been written by
1991.
A. Mortan Kaplan
B. John Von Neumann and Oskar Morgenstern 78. Who among the following used the phrase– The United
C. Karl Deutsch Nations is ‘sharing in the name of solidarity’ for
D. Michael Hass underlining the necessity of the UN?
A. Winston Churchill B. Dag Hammarskjold
73. Who among the following has remarked that, power in
C. Trygve Lie D. Woodrow Wilson
a political context means “the power of man over the
minds and actions of other men”? 79. Which one of the following is not a feature of
A. George Schwarzenberger ‘Shopkeeper diplomacy’?
B. Herman Heller A. It is based on peaceful approach
C. Hans Morgenthau B. It is a means of diplomacy for maintaining peace
D. Bertrand Russell through compromise
(2751) Pol. Sci.—9-II
Political Science 67
C. It is based on the sound business principles of 88. As a development model, Neo-liberalism in
moderation international relations refers to the
D. It tries to safeguard national interest through war A. rejection of structuralist economics in favour of
only the Washington Consensus.
B. denotes a conception of freedom as an overarching
80. Positive peace implies
social value
A. a ceasefire
C. involves the privatization of public economic
B. stopping the formation of death squads
sectors
C. transformation of relationship
D. All of these
D. social construction of war
89. “An Inquiry into the Principles of the Good Society”
81. Wilson’s idealism was a precursor to
is a book written by who among the following political
A. liberal international relations theory
philosopher?
B. radical international relations theory
A. Walter Lippmann
C. neither liberal nor radical international relations
B. Alexander Rüstow
theory
C. Ludwig von Mises
D. None of these D. Louis Rougier
82. The Twenty Years’ Crisis (1939) is a work of who 90. Neo-liberalism began accelerating in importance with
among the following idealists in international the establishment of the
relations? A. Mont Pelerin Society in 1947
A. E.H. Carr B. John A. Hobson B. Communitarian Society 1960
C. David Davies D. Lester Pearson C. Post Modern Society 1970
83. Idealism is also marked by the prominent role played D. None of these
by international law and international organizations 91. Neoliberal ideas were first implemented in which of
in its conception of the following country?
A. Policy formation A. West Germany B. East Germany
B. Welfare policies C. North korea D. South Korea
C. Social impact Assessment
D. Neo-Conservatism 92. Power and Interdependence: World Politics in
Transition (2008) is written by who among the
84. One of the most well-known tenets of modern idealist following neo-liberals?
thinking is A. Robert Keohane B. Joseph Nye
A. Democratic peace theory C. Milton Friedman D. Both A & B
B. Human rights theory
C. Authoritarian mindset 93. The critical theory was developed within the modern
D. Realistic theory discipline of IR with the help of
A. Antonio Gramsci and his influence over Robert
85. Neo-conservatism has drawn from historical liberalism Cox and the paradigm of production
its intense focus on the promotion of B. Frankfurt school – Jürgen Habermas in particular
A. Universal values C. Habermas over Andrew Linklater and the paradigm
B. International institutions of communication
C. Minority protections D. All of these
D. Both A & C
94. The perspective of critical theory inquires
86. Structural Marxism is an approach to Marxist A. global economic forces and related hierarchies of
philosophy based on structuralism, primarily associated power
with the work of the French philosopher B. in monitoring the chaos and insecurity
A. Louis Althusser C. forcing people to leave their homes in different
B. Nicos Poulantzas parts of the world
C. Thompson D. All of these
D. Eric Hobsbawm
95. The first comprised women’s suffrage movements of
87. Parliamentary Socialism in 1961 is a work of the 19th and early 20th centuries, promoting women’s
A. Miliband B. E.P. Thompson A. Political right B. Civil liberty
C. John Saville D. C. Wright Mills C. Social right D. Economic right
68 Political Science

96. The second wave of feminism the women’s liberation B. to affect the conduct of a government by mass
movement, began in the destruction, or kidnapping
A. 1960s B. 1980s C. to intimidate or coerce a civilian population;
C. 1950s D. 1940s D. Both A, B and C
97. The fourth wave feminism from around 2012 used 107. Which of the following about terrorist activities is not
social media to correct?
A. combat sexual harassment A. Red Army Faction from 1970 to 1998
B. violence against women B. Revolutionary Cells from 1973 to 1983
C. rape culture C. National Socialist Underground from 2000 to 2006
D. All of these D. The murder of Walter Lübcke
98. Which of the following act is not correctly matched? 108. Political warfare means
A. Custody of Infants Act -1839 A. use of political means to compel an opponent to
B. Married Women’s Property Act -1870 do one’s will
C. Representation of the People Act-1918 B. coercive nature leads to destroying an opponent’s
D. All-white Senate Judiciary Committee-1920 political will
99. Postmodernism is a broad movement that developed C. political warfare is to alter an opponent’s actions
in the in favour of one state’s interests
A. mid- to late 20th century D. All of these
B. mid- to late 19th century 109. After World War II, the threat of Soviet expansion
C. mid- to late 18th century
brought two new aims for American political warfare:
D. mid- to late 17th century
what are these?
100. Who among the following are associated with the A. To restore Western Europe through military,
philosophy of Post-modernism? economic, and political support
A. Jean-François Lyotard, B. To weaken the Soviet hold on Eastern Europe
B. Jacques Derrida through propaganda
C. Fredric Jameson C. To embrace a political warfare philosophy as both
D. All of these a defensive necessity
101. Postmodernism is often associated with schools of D. Both A & B
thought such as 110. President Harry S. Truman established a government
A. deconstruction, B. post-structuralism political warfare capability in
C. institutional critique D. All of these A. The National Security Act of 1947
102. The term postmodern was first used in B. The National Security Act of 1948
A. 1870 B. 1860 C. The National Security Act of 1950
C. 1850 D. 1840 D. The National Security Act of 1960
103. In 1996, Walter Truett Anderson described post- 111. Jihadist extremism in the United States (or Islamist
modernism as belonging to one of typological world extremism in the United States) refers to
views which he identified as: A. Islamic extremism B. Eurasian Policy
A. Postmodern-ironist B. Scientific-rational C. Freedom of religion D. All of these
C. Social-traditional D. All of these 112. A non State actor is an entity that falls into one of the
104. Postmodernism, or, the Cultural Logic of Late following groups:
Capitalism (1991) is associated with the work of A. non-governmental organizations (NGOs)
A. Fredric Jameson B. Douglas Kellner B. business associations
C. Jean Baudrillard D. Jean-François Lyotard C. philanthropic foundations
105. US policy toward international terrorism contains a D. All of these
significant of 113. Transnational organized crime groups and terrorist
A. Military component B. Political component movements are obviously
C. Neither A & B D. Both A & B A. non-state actors
106. Domestic terrorism is defined as activities that B. state actors
A. involve acts dangerous to hum an life by C. neither non-state nor state actors
assassination D. None of these
Political Science 69
114. Under this paradigm, the traditional Westphalian 124. Who is the last member of UN?
nation-state experiences an erosion of power and A. South Sudan became the 193rd member of the
sovereignty, and tick out which of the following is United Nations
coming under the jurisdiction of the cause? B. Russia became the 193rd member of the United
A. Non-state actors are part of the cause Nations
B. state actors are part of the cause C. Afghanistan became the 193rd member of the
C. Fourth World societies United Nations
D. Both A & B D. Poland became the 193rd member of the United
115. The term Non State Actors is widely used in Nations
development cooperation, particularly under which of 125. Which among the following as the most powerful
the following agreement? organ of the United Nations?
A. the Cotonou Agreement A. United Nations Security Council
B. Centre on Housing Rights and Evictions B. United Nations General Assembly
C. Human Rights Council C. United Nations Economic and Social Council
D. International Campaign to Ban Landmines D. United Nations International Court of Justice
116. The traditional security paradigm reached a peak 126. As of 2019, who among the following countries are
during the permanent non-member observer states in the General
A. Cold War B. Look West
Assembly of the United Nations?
C. Look East D. None of these
A. The Holy See and Palestine
117. In the traditional security, the greatest danger to a B. Canada and Japan
country is from C. France and Great Britain
A. Military threats B. Political threats D. None of these
C. Social threats D. Cultural threats
127. The difference between the ICC and the International
118. The non-traditional security consists of dangers such Court of Justice are mainly on:
as: A. The International Court of Justice (ICJ) is a civil
A. terrorism B. human rights court that hears disputes between countries
C. global poverty D. All of these B. The International Court of Justice (ICJ) is the
119. External threats of The “Traditional Notion of principal judicial organ of the United Nations
Security” consist of which of the following components: (UN).
A. military threats B. threat of war C. The ICC is a criminal court that prosecutes
C. balance of power D. Both A, B & C individuals
D. All A, B & C
120. How many countries are in the United Nations?
A. 194 B. 198 128. The Bretton Woods Agreement and System created a
C. 193 D. 197 collective international currency exchange regime that
121. Each of the Member States of the United Nations has lasted from
how many seat in the General Assembly? A. the mid-1940s to the early 1970s
A. Two seats B. Four Seats B. the mid-1950s to the early 1970s
C. One Seat D. Five seats C. the mid-1930s to the early 1940s
D. the mid-1940s to the early 1950s
122. Which of the following members ate and date of
admission is not correctly matched? 129. The Bretton Woods system was the first system used
A. United Republic of Tanzania-14 December 1961 to control
B. United Kingdom of Great Britain-24 October 1945 A. the value of money between different countries
C. United States of America-24 October 1950 B. the political system between different countries
D. Uruguay-18 December 1945 C. the social system between different countries
D. None of these
123. The main purposes of the UN are documented under
the UN Charter as 130. Bretton Woods failed due to
A. To keep peace throughout the world A. the US decision to suspend gold convertibility
B. To develop friendly relations among nations B. collapse was the inflationary monetary policy
C. To help nations work together C. consolidation of the world political system
D. All of these D. Botha A & B
70 Political Science

131. North-South relations begin with 140. Which among the following policy making body of
A. decolonialization after World War II G20 enriched participation of key international
B. colonization of the country during 1st world War organizations?
C. anti-colonial struggles among the developed A. World Bank, International Monetary Fund,
countries B. International Labour Organization, World Trade
D. None of these Organization
C. United Nations, etc.
132. Globalization has largely displaced the North–South D. All A, B & C
divide as the theoretical underpinning of the
development efforts of international institutions such 141. Which of the following reasons lead to climate change?
as A. People burn fossil fuels and convert land from
A. the IMF B. World Bank forests to agriculture
B. Burning fossil fuels produces carbon dioxide
C. WTO etc D. All A, B & C
C. Human activity produces indirect effect
133. The North–South divide is popularly called as D. All of these
A. Global North and Global South
142. In which year under the Paris Agreement, nations
B. Global East and Global West
collectively agreed to keep warming “well under 2.0°C
C. Neither A nor B
(3.6°F)” through mitigation efforts.?
D. Both A & B A. 2011 B. 2010
134. The North is mostly correlated with the Western world C. 2015 D. 2021
called as 143. Which among the following international environ-
A. First World plus much of the Second World mental agreements is not correctly matched?
B. Called third world A. Convention on Long-Range Transboundary Air
C. Called Eastern world Pollution (LRTAP), Geneva, 1979.
D. None of these B. Paris Agreement, 2010
135. The Sustainable Development Goals (SDGs,) set in C. Kyoto Protocol, 1997
2015 by the United Nations General Assembly and D. Framework Convention on Climate Change
intended to be achieved by the year 2030, are part of (UNFCCC)
a UN Resolution called 144. Internal migration refers to
A. The 2030 Agenda B. The 2025 Agenda A. moving within a state
C. The 2024 Agenda D. The 2022 Agenda B. moving to a different state, country
C. moving into a new country
136. Who among the following country hosted the most
D. moving back to where you came from
recent 12th BRICS summit on 17 November 2020
virtually due to the COVID-19 pandemic? 145. The major forms of migration relates to
A. Russia B. Poalnd (a) Invasion
C. South Africa D. India (b) Conquest
(c) Colonization and
137. In which year there policy decisions in the induction (d) Emigration/immigration
of South Africa in BRIC?
A. 2014 B. 2010 Select the correct code given below:
A. (a), (b) & (c) B. (b) & (a)
C. 2017 D. 2020
C. (d) & (a) D. (a), (b), (c) & (d)
138. The New Development Bank (NDB), formally referred
146. The term identity politics may have been used in
to as the
political discourse since at least the
A. BRICS Development Bank
A. 1970s B. 1960s
B. World Bank
C. 1950s D. 1940s
C. International Monetary Fund
D. Structural adjustment Program 147. Who among the following Historian discussed identity
politics extensively in his 1991 book “The Disuniting
139. How many countries comprises as G20 of America”?
A. 19 countries and the European Union A. Brendan O’Neill
B. 20 countries excluding European union B. Owen Jones
C. 20 countries and the African belt C. Arthur Schlesinger Jr.
D. None of these D. Judith Butler
Political Science 71
148. Which among the following is not correctly matched? A. the racial or ethno-cultural consciousness of Arab
A. ethnic racial identity politics dominated American people
politics in the 19th century B. the personal is political
B. racial identity has been the central theme in C. identities of gender, class, and race
Southern politics since slavery was abolished D. None of these
C. racial identity politics utilizes racial consciousness 150. “The Case for Identity Politics: Polarization,
or the group’s collective memory
Demographic Change, and Racial Appeals” is written
D. All of these
by who among the following political scientists?
149. Arab identity politics concerns the identity-based A. Mike Gonzales B. Christopher T. Stout
politics derived from C. Ross Douthat D. Ezra Klein

ANSWERS
1 2 3 4 5 6 7 8 9 10
D B A B B C A B B D
11 12 13 14 15 16 17 18 19 20
B C D B B B D C A B
21 22 23 24 25 26 27 28 29 30
B C C D B C D A D D
31 32 33 34 35 36 37 38 39 40
A B C B B C D A C B
41 42 43 44 45 46 47 48 49 50
A A B C B A B A A D
51 52 53 54 55 56 57 58 59 60
D B B D A A C C A D
61 62 63 64 65 66 67 68 69 70
C B B D A A D A D D
71 72 73 74 75 76 77 78 79 80
D B C B B D D B D C
81 82 83 84 85 86 87 88 89 90
A A A A D A A D A A
91 92 93 94 95 96 97 98 99 100
A D D D A A D D A D
101 102 103 104 105 106 107 108 109 110
D A D A D D B D D A
111 112 113 114 115 116 117 118 119 120
D D D D A A A D D C
121 122 123 124 125 126 127 128 129 130
C C D D A A D A A D
131 132 133 134 135 136 137 138 139 140
A D D A A A B A A D
141 142 143 144 145 146 147 148 149 150
D C B A D A C D A B
  
72 Political Science

Miscellaneous MCQs

1. The end of the Cold War is usually seen as the 9. After how many weeks USA became aware of Missile
beginning of _______________. in Cuba?
A. Contemporary era in world politics. A. Four weeks B. Two weeks
B. Indian Politics. C. Three weeks D. One week
C. Political History 10. A clash seemed imminent in what came to be known
D. Western Political Thought as the_______.
2. The dominance of two superpowers, the of_________ A. Cuban Missile Crises
and _________ the was central to the Cold War. B. Cuban Crises
A. Soviet Union C. Missile Crises
B. United State of America D. Cuba Missile Crises
C. Both A and B 11. The Cuban Missile Crises was a high point of what
D. None of the above came to be known as the _____ .
3. New Internal Economic Order (NIEO) as a means of A. Hot War B. Cold War
attaining ________ and ______. C. World War 1 D. World War 2
A. Freedom and Equality 12. __________ referred to the competition, the tensions
B. Economic development and a series of confrontations between the United
C. Political Independence States and Soviet Union, backed by their respective
D. Both B and C allies.
4. In which year the leaders of the Union of Soviet A. Hot War B. Cold War
Socialist Republics were worried that the United States C. World War 2 D. Cuban Missile Crises
of America would invade communist ruled Cuba. 13. The Cold War included ________.
A. April 1961 B. April 1916 A. Peace, competition and understanding.
C. April 1996 D. April 1960 B. Rivalries, military alliance and the balance of
5. Who was the president of a small island ______ nation power.
of the United States? C. Unity, competition and rivalries
A. John F Kennedy D. Peace, competition and unity.
B. Fidel Castro 14. The Cold War was accompanied by a ______ conflict
C. Nikita Khrushchev as well.
D. George Bush A. Monetary Conflicts B. Ideological Conflicts
6. _______ as an ally of the Soviet Union and received C. Personal Conflicts D. All of the above.
both diplomatic and financial aid from it. 15. Who was the leader of Western Alliance?
A. USA B. Cuba A. US B. Russia
C. Russia D. Britain C. USSR D. Allied powers
7. Who decided to convert Cuba into a Russian base? 16. The Western Alliance represent ideology of:
A. American President Kennedy A. Socialism
B. Fidel Castro B. Liberalism, Democracy and Capitalism
C. Nikita Khrushchev C. Communalism
D. Stalin D. Marxism
8. In which year nuclear missiles was placed in Cuba? 17. Who was the leader of Eastern Alliance?
A. 1962 B. 1926 A. Soviet Union B. USA
C. 1965 D. 1960 C. Russia D. Britain
72
Political Science 73
18. The Eastern Alliance represent the ideology of: 26. The yield of Little Boy and Fat man were _________
A. Capitalism respectively.
B. Marxism A. 15 and 26 kilotons B. 15 and 29 kilotons
C. Socialism and Communalism C. 15 and 21 kilotons D. 14 and 21 kilotons
D. Socialism and Marxism
27. By the early 1950s the US and the USSR were already
19. Allied forces led by the __________. making thermonuclear weapons hat had a yield
A. Germany, Italy and Japan between _______.
B. US, Soviet Union, Britain and France A. 10 and 15 kilotons
C. US, Soviet Union, Italy and Japan B. 10 and 15 thousand kilotons
D. USA, USSR C. 15 and 20 kilotons
20. The World War Two spread up to which regions? D. 100 thousand kilotons.
A. Asia, India, China and Burma 28. When two rival powers are in possession of nuclear
B. South east Asia, China, Burma and parts of India’s weapons capable of inflicting death and destruction
Northeast uncapable to each other, a full-fledged war is:
C. Northeast Asia, China, Burma and part of India’s A. unlikely B. likely
northeast. C. must D. possible
D. Northeast Asia, China, Burma and parts of India’s
29. What happens in Logic of deterrence situation?
north.
A. Both sides have the capacity to retaliate against
21. Why did the critics said it was unnecessary to drop an attack
bombs? B. Cause so much destruction that neither can afford
A. Japan was innocent to initiate war
B. Japan was about to surrender C. Neither side would want to risk war
C. Japan was week D. Both A and B
D. Japan was stronger
30. In spite being an in tense form of rivalry between great
22. According to critics, what was the reasons for American powers—remained a “cold and hot or shooting war
Bombing Japan? due to:
A. To stop the Soviet Union from making military A. Logic of War B. Logic of peace
and political gains in Asia C. Logic of deterrence D. All of the above
B. To show Moscow that the United States was
supreme 31. The deterrence relationship prevents war but not
C. To end the war quickly ________.
D. Both A and B A. Fighting
B. Ideological conflict
23. According to the US supporters have USA Dropping C. Rivalry between powers
of the atomic bombs was necessary to: D. None of the above
A. To stop the Soviet Union from making military
and political gains in Asia 32. In the deterrence relationship both blocks led by the
B. To stop further loss of American and Allied superpowers were respected to behave as ________
lives. actors.
C. To end war quickly and to stop A. Rational and responsible
D. Both B and C B. Responsible and vigilant
C. Careful honest
24. Atomic Bomb of US on Hiroshima was code named.
D. None of the above
A. The bomb was code-named “Little Boy”
B. Fat man 33. When two superpowers and the blocks led them are in
C. Little baby a ______ relationship, fighting wars will be massively
D. Little girl destructive?
A. Friendly B. Deterrence
25. Atomic Bomb on Nagasaki was code named as:
C. Peaceful D. Neutral
A. Fat man
B. Fat boy 34. Cold War managed to ensure ________ .
C. Fat women A. Peace B. Human lives
D. Fatty man C. Human survival D. None of the above
(2751) Pol. Sci.—10
74 Political Science

35. The Western alliance was formalized into an 44. Which of the following is not a member of NATO?
organization, the ______, which came into existence A. The US B. England
________. C. France D. None of these
A. NATO, In April 1949
45. Which of the following was not a member of SEATO?
B. SEATO, In April 1948
A. USSR B. New Zealand
C. CENTO, In April 1994
C. Pakistan D. Australia
D. TANTO, In April 1947
46. Which of the followings statement about the NIEO is
36. NATO is an association of ________ states.
false?
A. 23 B. 30
A. Give the LDCs control over their natural resources
C. 18 D. 25
B. Obtain access to Western markets
37. The Eastern alliance, known as the __________ was C. Reduce the cost of technology from the Western
led by the Soviet Union. countries.
A. Warsaw pact B. SEATO D. Provide the developed countries with a greater
C. CENTO D. NATO role in international economic institutions.
38. The Eastern alliance was created in _______ and its 47. In 1945 the allied forces included the US, _____
principal function was to counter ____ forces in USSR and ______.
Europe. A. US, Britain, USSR and France
A. 1954, CENTO B. 1955, NATO B. US, Japan, USSR and France
C. 1955, SEATO D. None of the above C. US, France, USSR and Italy
39. In East and Southeast Asia and in West Asia, the D. US, Italy, USSR and Japan
United States built an alliance system called______ 48. The event that took place in 1961 was
and ______. A. The construction of the Berlin wall.
A. CENTO and NATO B. NATO and SEATO B. Soviet intervention in Afghanistan.
C. SEATO and CENTO D. None of the above C. Vietnamese intervention in Cambodia.
40. The Soviet Union and Communist China responded D. The Unification of Germany
by heaving close relations with regional countries Directions (Qs. No. 49-53): Read the passage carefully and
such as: answer the question that follow:
A. India, Pakistan and Nepal
B. North Vietnam, North Korea and Iraq In April 1961, the leaders of the Union of Soviet
C. Iraq, Pakistan and Bhutan Socialist Republics (USSR) were worried that the United
D. North Vietnam, North Korea and Pakistan States of America (USA) would invade communist-ruled
Cuba and overthrow Fidel Castro, the president of the
41. Name the movement which gave newly independent small island nation off the coast of the United States. Cuba
countries a way staying out of the alliance. was an ally of the Soviet Union and received both
A. NAM B. NATO diplomatic and financial aid from it. Nikita Khrushchev,
C. CENTO D. SEATO
the leader of the Soviet Union, decided to convert Cuba
42. Which among the following statements about the Cold into a Russian base. In 1962, he placed nuclear missiles in
War is wrong? Cuba. The installation of these weapons put the US, for the
A. It was a competition between the US and Soviet first time, under fire from close range and nearly doubled
Union and their respective allies the number of bases or cities in the American mainland
B. It was an ideological war between the superpowers which could be threatened by the USSR.
C. It triggered of an arms race
49. Who were worried that the United States of America
D. The US and USSR were engaged in direct wars.
would invade communist ruled Cuba?
43. Which among the following statements does not reflect A. The leaders of Cuba
the objectives of NAM? B. The leaders of USSR
A. Enabling newly decolonized countries to pursue C. The leaders of Cuba
independent policies. D. None of the above
B. Not joining any military alliances.
C. Following a policy of neutrality on global issues. 50. Who was Fidel Castro?
D. Focus on elim ination of global economic A. President of USSR B. President of USA
inequalities. C. President of Cuba D. President of Britain
Political Science 75
51. Cuba was an ally of the Soviet Union and received 58. A _______ seems imminent in what came to be known
both: as the Cuban Missile Crises.
A. Technical and Financial aid A. War B. Clash
B. Diplomatic and Financial aid C. Nuclear War D. Social Life
C. Political and Technical aid Directions (Qs. No. 59-63): Read the passage carefully and
D. None of the above answer the question that follow:
52. Nikita Khrushchev, the leader of the Soviet Union, The end of the Second World War is a landmark in
decided to convert Cuba into a: contemporary world politics. In 1945, the Allied Forces,
A. USA base B. Cuba base led by the US, Soviet Union, Britain and France defeated
C. Russian base D. Britain base the Axis Powers led by Germany, Italy and Japan, ending
53. In which year nuclear missile was placed in Cuba? the Second World War (1939-1945). The war had involved
A. 1962 B. 1926 almost all the major powers of the world and spread out to
C. 1965 D. 1960 regions outside Europe including Southeast Asia, China,
Burma (now Myanmar) and parts of India’s northeast. The
Directions (Qs. No. 54-58): Read the passage carefully and
war devastated the world in terms of loss of human lives
answer the question that follow:
and civilian property. The First World War had earlier
Three weeks after the Soviet Union had placed the shaken the world between 1914 and 1918.
nuclear weapons in Cuba, the Americans became aware of
59. Which incident is a landmark in contemporary world
it. The US President, John F. Kennedy, and his advisers
politics?
were reluctant to do anything that might lead to full-scale
A. The end of the First World War.
nuclear war between the two countries, but they were
B. The end of the Second World War.
determined to get Khrushchev to remove the missiles and
C. The end of Cold of War.
nuclear weapons from Cuba. Kennedy ordered American
D. All the above.
warships to intercept any Soviet ships heading to Cuba as
a way of warning the USSR of his seriousness. A clash 60. In 1945, _______ forces, led by the US, Soviet Union,
seemed imminent in what came to be known as the Cuban Britain and France defeated the Axis Powers led by
Missile Crisis. The prospects of this clash made the whole Germany.
world nervous, for it would have been no ordinary war. A. Axis Power B. Allied Power
Eventually, to the world’s great relief, both sides decided C. Soviet Power D. Union Power
to avoid war. The Soviet ships slowed down and turned 61. The war had involved almost all the major powers of
back. the world and spread out to regions outside ______.
54. After____ weeks the Soviet Union had placed the A. Europe, Asia, China, Pakistan and Burma.
nuclear weapons in Cuba. B. Europe, Southeast Asia, China, Burma and parts
A. One week B. Four weeks of India’s northeast.
C. Three weeks D. Two weeks C. China, India, Burma and Pakistan
D. All the above
55. Who were reluctant to do anything that might lead to
full-scale nuclear war between the two countries? 62. Which war had earlier shaken the world between 1914
A. Fidel Castro and Nikita Khrushchev and 1918?
B. John F. Kennedy and Fidel Castro A. Second World War B. Cold War
C. John F. Kennedy and his Advisers C. First World War D. None of the above
D. Nikita Khrushchev and his advisers 63. The War devastated the World in terms of loss of ____
56. To whom Kennedy ordered to intercept any Soviet and _____.
ships heading to Cuba. A. Freedom and Finance
A. Russian warships B. American warships B. Human lives and civilian property
C. Cuban warships D. None of the above C. Ideology and Political life
D. Economy and Freedom
57. What was the greatest relief to the whole world?
A. Soviet ships heading to Cuba. Directions (Qs. No. 64-68): Read the passage carefully and
B. Cuban Missile crise was high point. answer the question that follow:
C. Both sides decided to avoid war. The end of the Second World War was also the
D. All the above. beginning of the Cold War. The world war ended when the
76 Political Science

United States dropped two atomic bombs on the Japanese these states would be obliged to help the other. The eastern
cities of Hiroshima and Nagasaki in August 1945, causing alliance, known as the Warsaw Pact, was led by the Soviet
Japan to surrender. Critics of the US decision to drop the Union. It was created in 1955 and its principal function
bombs have argued that the US knew that Japan was about was to counter NATO’s forces in Europe.
to surrender and that it was unnecessary to drop the bombs.
69. In which year the NATO came into existence?
They suggest that the US action was intended to stop the
A. In April 1948
Soviet Union from making military and political gains in
B. In April 1949
Asia and elsewhere and to show Moscow that the United
C. In April 1984
States was supreme. US supporters have argued that the
D. In April 1947
dropping of the atomic bombs was necessary to end the war
quickly and to stop further loss of American and Allied 70. The Eastern Alliance was led by whom?
lives. Whatever the motives, the consequence of the end of A. USA B. Britain
the Second World War was the rise of two new powers on C. Soviet Union D. None of the above
the global stage. 71. What was the principal function of Warsaw pact?
64. What was also the reason for beginning of the Cold A. To defend Western Alliance.
War? B. To defend Europe and North America
A. The end of the First World War C. To counter NATO’s forces in Europe
B. The end of the Second World War D. All the above
C. The end of the Cold War 72. NATO was an association of __________ states.
D. All the above A. Eleven B. Twelve
65. When was the Second World War ended? C. Thirteen D. Fourteen
A. Both sides decided to avoid war 73. Western Alliance was led by whom?
B. Mutual understanding by both A. Britain B. USA
C. USA dropped atomic bombs on Hiroshima and C. USSR D. Cuba
Nagasaki
D. All the above Directions (Qs. No. 74-78): Read the passage carefully and
answer the question that follow:
66. Who suggested that the US action was intended to
stop the Soviet Union for making military and political The Cold War threatened to divide the world into two
gains in Asia and elsewhere? alliances. Under these circumstances, many of the newly
A. US critics B. US supporters independent countries, after gaining their independence
C. Both A and B D. None of the above from the colonial powers such as Britain and France, were
worried that they would lose their freedom as soon as they
67. Who have argued that the dropping of the atomic gained formal independence. Cracks and splits within the
bombs was necessary to end the war quickly? alliances were quick to appear. Communist China quarrelled
A. US critics B. US supporters with the USSR towards the late 1950s, and, in 1969, they
C. Both A and B D. None of the above fought a brief war over a territorial dispute. The other
68. According to critics, what was the reasons for American important development was the Non-Aligned Movement
Bombing Japan? (NAM), which gave the newly independent countries away
A. To stop Soviet Union from making military and of staying out of the alliances.
political gains in Asia. 74. What was threatening of the cold war to the whole
B. To stop further loss of America and Allied lives. world?
C. To end war quickly and to stop A. Destruction of the world.
D. Both A and C B. Division of the world.
Directions (Qs. No. 69-73): Read the passage carefully and C. Division into two alliances.
answer the question that follow: D. All the above
The western alliance w as formalised into an 75. Why did many newly independent countries worry
organisation, the North Atlantic Treaty Organisation after gaining independence?
(NATO), which came into existence in April 1949. It was A. Getting more facilities
an association of twelve states which declared that armed B. Loss of freedom
attack on any one of them in Europe or North America C. More independence
would be regarded as an attack on all of them. Each of D. None of the above
Political Science 77
76. What was reason behind the quarrel between China Directions (Qs. No. 83-94): In the question below, there are
and USSR? two statements marked as Assertion (A) and Reason (R).
A. Territorial disputes B. Financial disputes Read the statements and choose the correct option.
C. Military disputes D. All the above Options:
77. What was the prospective of the newly independent A. If both (A) and (R) are true, and (R) is the correct
countries? explanation of (A).
A. Cooperate to alliance B. If both (A) and (R) are true, but (R) is not the
B. Stay away from the alliance correct explanation of (A).
C. Stay with USSR C. If (A) is true but (R) is false.
D. Stay with US D. If (A) is false but (R)is true.

78. In which year China ad USSR fought a brief war over 83. Assertion (A) : The end of the Second World War
a territorial dispute? also the beginning of the Cold War.
Reason (R) : The world war ended when the United
A. 1969 B. 1968
States dropped two atomic bombs on
C. 1967 D. 1996
the Japanese cities Hiroshima and
Directions (Qs. No. 79-82): Read the passage carefully and Nagasaki.
answer the question that follow:
84. Assertion (A) : The Western alliance was formalised
Sometimes, countries outside the two blocs, for into an organisation, NATO which
example, the non-aligned countries, played a role in reducing came into existence in 1949.
Cold War conflicts and averting some grave crises. Reason (R) : The Eastern alliance, was led by the
Jawaharlal Nehru — one of the leaders of the NAM — Soviet Union and it was created in
played a crucial role in mediating between the two Koreas. 1955.
In the Congo crisis, the UN Secretary-General played a key
85. Assertion (A) : The Cuban Missile Crises was a high
mediatory role. By and large, it was the realisation on a
point of what came to be known as
superpower’s part that war by all means should be avoided
the Cold War.
that made them exercise restraint and behave more
Reason (R) : The Cold W ar referred to the
responsibly in international affairs. As the Cold War rolled
competition, the tension and a series
from one arena to another, the logic of restraint was
of confrontations between the United
increasingly evident. However, since the Cold War did not
States and Soviet Union.
eliminate rivalries between the two alliance, mutual
suspicions led them to arm themselves to the teeth and to 86. Assertion (A) : Cuba was an ally of the Soviet Union
constantly prepare for war. Huge stocks of arms were and received both diplomatic and
considered necessary to prevent wars from taking place. financial aid from it.
Reason (R) : Three weeks after the USA had placed
79. Which countries played a crucial role in reducing
the nuclear weapons in Cuba.
Cold War?
A. Allied countries B. Non-Aligned countries 87. Assertion (A) : The Cold War was not simply a matter
C. Western countries D. Foreign countries of power rivalries, of military alliance,
and of the balance of power.
80. Who was an Indian played crucial role in mediating
Reason (R) : These were accompanied by a real
between the two Koreas?
ideological conflict as well, a difference
A. Mahatma Gandhi
over the best and the most appropriate
B. Jawaharlal Nehru
way of organising political, economic,
C. Netaji Subash Chandra Bose and social life all over the world.
D. Motilal Nehru
88. Assertion (A) : Economic development was also vital
81. Who played a crucial role in mediating in the Congo for the independence of the new
Crises? countries.
A. Jawaharlal Nehru B. NAM countries Reason (R) : Without sustained development, a
C. UN Secretary General D. Western countries country could not truly free. It would
82. What were considered necessary to prevent wars for remain dependent on the richer
taking place? countries including the colonial
A. International affairs B. Logic of restraint powers from which political freedom
C. Huge stock of arms D. Non-aligned countries had been achieved.
78 Political Science

89. Assertion (A) : The Cuban Missile Crises was a high 96. The Berlin Wall was more than _____ kilometer long.
point of what came to be known as A. 100 km B. 150 km
the Cold War. C. 175 km D. 200 km
Reason (R) : The war devastated the world in terms 97. The USSR group was also known as the _______
of loss of human lives and civilian bloc.
property. A. Capitalist bloc
90. Assertion (A) : The Second World War ended when B. Socialist bloc
the United States dropped two atomic C. Democratic bloc
bombs on the Japanese cities. D. Developed bloc
Reason (R) : The end of the Second World War is 98. Warsaw pact signed by mainly by the countries of
a landmark in contemporary world _____ Europe.
politics. A. Eastern B. Central
91. Assertion (A) : Since the Cold War did not eliminate C. Southern D. Western
rivalries between the two alliances, 99. When Mikhail Gorbachev became General secretary
mutual suspicious led them to arm of the communist party of Soviet Union?
themselves to the teeth and to A. 1981 B. 1985
constantly prepare for war. C. 1989 D. 1990
Reason (R) : The non-aligned countries were more
than merely mediators during the Cold 100. The Soviet Union invaded Afghanistan in ______.
War. A. 1969 B. 1979
C. 1989 D. 1999
92. Assertion (A) : The Cold War was an outcome of the
emergency of the US and the USSR as 101. _______ was the successor of Lenin.
two superpowers rival to each other. A. Putin B. Gaubachev
Reason (R) : When two rival powers are in posse- C. Stalin D. Trotosky
ssion of nuclear weapons capable of 102. Collectivisation of agriculture was done by _______
inflicting death and destruction in the USSR.
unacceptable to each other, a full- A. Stalin B. Lenin
fledged war is unlikely. C. Yeltsin D. Putin
93. Assertion (A) : The two superpowers were not keen 103. Who among the following is not a Baltic republic?
on expanding their spheres of A. Estonia B. Lithuania
influence in different parts of the C. Latvia D. London
world.
104. What is the meaning of Perestroika?
Reason (R) : The smaller states in the alliances
A. Rejecting B. Recounting
used the link to the superpowers for
C. Restructuring D. Reopening
their own purpose.
105. What is the meaning of Glasnost?
94. Assertion (A) : The consequence of the end of the A. Transparency B. Openness
Second World War was the rise of two C. Lockdown D. Strick
new powers on the global stage.
Reason (R) : With the defeat of Germany and Japan, 106. ________ became the first republic to declare its
the devastation of Europe and in independence from USSR?
many other parts of the world, the A. Lithuania B. Liberia
United State and the Soviet Union C. Georgia D. Azerbaijan
became the greatest powers in the 107. Yeltsin became the President of Russia in the year
world. _____.
95. The Berlin War symbolized the division between the A. 1998 B. 1991
_________world. C. 1994 D. 1995
A. capitalist and communist 108. In 1991 _______ signed as the President of the Soviet
B. rich and poor Union.
C. developed and developing. A. Gorbachev B. Yeltsin
D. East and West C. Kruschev D. Breznev
Political Science 79
109. Czechoslovakia was split into _______ parts after 119. Assertion (A) : The Berlin wall was built in 1961.
1945. Reason (R) : Germany was the friend of allied
A. 5 B. 4 countries.
C. 3 D. 2
Directions (Qs. No. 120-124): Read the following passage
Directions (Qs. No. 110-119): The following questions and answer these questions.
consist of two statements—Assertion (A) and Reason (R). The Berlin Wall symbolised the division between the
Answer these questions selecting the appropriate option capitalist and the communist world. Built in 1961 to
given below: separate East Berlin from West Berlin, this more than 150
A. Both (A) and (R) are true and (R) is the correct kilometre long wall stood for 28 years and was finally
explanation of (A). broken by the people on 9 November 1989. This marked
B. Both (A) and (R) are true, but R is not the correct the unification of the two parts of Germany and the
explanation of (A). beginning of the end of the communist bloc.
C. (A) is true, but (R) is false.
D. (A) is false, but (R) is true. 120. Berlin wall symbolises the division between _______
and __________.
110. Assertion (A) : USSR came into being after 1917 A. Capitalism and Secularism
Revolution. B. Capitalism and Democracy
Reason (R) : It was inspired by socialism and C. Capitalism and Communism
opposed capitalism. D. Communism and Socialism
111. Assertion (A) : After Second World War, the world 121. Berlin wall was built in _________ year.
was divided into Eastern Block and A. 2020 B. 2021
Western Block. C. l961 D. 1999
Reason (R) : Western countries joined Eastern
Block. 122. The Berlin Wall Separates ______________.
A. East Berlin from North Berlin.
112. Assertion (A) : Vladimir Lenin died in 1924. B. East Berlin from North West Berlin
Reason (R) : Stalin came to power in 1924. C. East Berlin from West Berlin
113. Assertion (A) : Joseph Stalin period saw the rapid D. East Berlin from South Berlin
industrialisation of USSR. 123. The Berlin Wall stood for ___________ years.
Reason (R) : His period was known as the Great A. 10 B. 21
Terror of 1930’s. C. 20 D. 28
114. Assertion (A) : Cold War came to an end with the 124. The Berlin Wall was broken on ____________.
collapse of Soviet Union. A. 9th November 1989
Reason (R) : USA became only the powerful B. 10th November 1989
country in the world. C. 11th November 1989
115. Assertion (A) : Many western European countries D. 12th November 1989
joined NATO. Directions (Qs. No. 125-130): Read the following passage
Reason (R) : It was a military alliance formed by and answer these questions.
USA.
After the Second World War, the east European
116. Assertion (A) : Shock therapy was the transition of countries that the Soviet army had liberated from the fascist
communism to capitalism. forces came under the control of the USSR. The political
Reason (R) : The collapse of communism resulted and the economic systems of all these countries were
in shock therapy. modelled after the USSR. This group of countries was
117. Assertion (A) : Due to shock therapy the value of called the Second World or the ‘socialist bloc’. The Warsaw
Russian Ruble declined. Pact, a military alliance, held them together. The USSR was
Reason (R) : Ruble is not the currency of USSR. the leader of the bloc.
118. Assertion (A) : India’s Relations with Russia are an 125. The second world war ended in __________ year.
important aspect of India’s foreign A. 1942
policy. B. 1943
Reason (R) : India maintained good Relations with C. 1944
all the post-communist countries. D. 1945
80 Political Science

126. USSR stands for ___________. 134. The communist party was not accountable to the
A. Union of Soviet people, Ordinary people were exploited by rampant
B. Soviet Republic corruption.They were given no representation in the
C. Union of Soviet Socialist Republic state machinery:
D. Union of Republic I. USSR started the five year plan
127. After second world war, the East European countries II. People were subjected to lot of difficulties
II. India adopted five year plan copying from USSR
came under the control of:
IV. USA and USSR remained to be powerful after
A. USA B. USSR
C. UK D. France disintegration.

128. The military alliance of Warsaw was created by Read the following options mention which sentence is
wrong?
_________ .
A. I only B. I and II
A. USA B. USSR
C. Britain D. China C. I, II and III D. IV only

129. The western Block was controlled by __________. 135. Russia had good relationship with India because:
A. India purchases its arms
A. China B. USA
B. India imports oil from India
C. USSR D. India
C. India had very good cultural relations with it
130. Arrange the following in the chronological order of D. All of the above
occurrence:
136. Consider the following statements. Find the wrong
I. Disintegration of USSR
statement
II. Berlin wall collapse
III. February Revolution A. Lenin was the leader of Bolshevik party
B. Stalin was not the immediate successor of Lenin
IV. Gorbachev became the General Secretary of USSR
C. Gorbachev was blamed for the disintegration of
A. I, II, III and IV B. II, I, III and IV
C. III, IV, II and I D. IV, III, II and I USSR
D. Boris Yelstin was the first elected president of
131. Which among the following statements that describe Russia
the nature of Soviet economy is wrong?
A. Socialism was the dominant ideology 137. The Berlin wall was toppled on ______ .
A. 9th November 1989
B. State ownership/control existed over the factors
of production B. 8th November 1989
C. 7th November 1989
C. People enjoyed economic freedom
D. Every aspect of the economy was planned and D. 6th November 1989
controlled by the state. 138. Soviet political system was based on _____ ideology.
A. Mixed economy
132. Which among the following is not an outcome of the
B. Socialism and communism
disintegration of USSR?
C. Capitalism
I. End of the ideological war between USA and
D. Private sector
USSR
II. Birth of CIS 139. USSR stands for _______ .
III. Change in the balance of power in the world A. Union of Socialist Russia
IV. Crisis in the Middle East B. Union of Soviet
C. Union of Soviet Socialist Republic
Select the correct answer using the four given above:
A. IV only B. II, I and III D. United States Russia
C. III, II and IV D. II and IV Directions (Qs. No. 140-144): Read the following passage
133. Which among the following statements is false? and answer the questions.
A. Russian revolution was inspired by the ideals of The UN was founded as a successor to the League of
socialism. Nations. It was established in 1945 immediately after the
B. Lenin was the leader of Russian revolution. Second World War. The organization was set up through
C. The transition from socialism to capitalism is the signing of the United Nations Charter by 51 states. It
known as shock therapy tried to achieve what the League could not between the
D. India was responsible for the disintegration of two world wars. The UN’s objective is to prevent
USSR. international conflict and to facilitate cooperation among
Political Science 81
states. It was founded with the hope that it would act to institutions and regulations that act at the
stop the conflicts between states escalating into war and, if international level.
war broke out, to limit the extent of hostilities. Furthermore, D. International Money Fund is a international
since conflicts often arose from the lack of social and organization that it oversees those financial
economic development, the UN was intended to bring institutions and regulations that act at the
countries together to improve the prospects of social and international level.
economic development all over the world.
146. Write about the membership and voting procedure of
140. _______ was the founded as a successor to league of the International Monetary Fund.
Nations. A. The IMF has 189 member countries, but they do
A. UNESCO B. The UN not enjoy an equal say. The US alone has 16.54
C. The USA D. The USSR per cent voting rights.
B. The IMF has 189 member countries, but they are
141. The UN was established in the year:
A. 1942 B. 1946 enjoying equal rights.
C. The US alone has 16.54 per cent voting rights
C. 1945 D. 1940
because it is the permanent member country in
142. The UN’s objective is: the Security Council.
A. to prevent international conflict and to facilitate D. None of these
cooperation among states
147. Why has the US alone held 16.54 per cent voting
B. to start war
rights?
C. to create war like situation
D. to encourage conflicts A. The US alone has 16.54 per cent voting rights
because it is the poorest country. The US providing
143. How many states signed to set up the organization? less amount for it’s funding.
A. 50 B. 45 B. The US alone has 16.54 per cent voting rights
C. 51 D. 52 because it is the richest member country of the
144. How do often conflicts arise? organization. The US provides maximum for its
A. lack of social and economic development funding.
B. lack of culture C. The US alone has 16.54 per cent voting rights
C. lack of knowledge because it is the sole superpower in the world.
D. lack of faith D. The US alone has 16.54 per cent voting rights
because it is the permanent member country in
Directions (Qs. No. 145-149): Read the following passage
the Security Council.
and answer the questions.
148. Which are the following G-8 member countries?
IMF— The International M onetary Fund is an
A. US, Japan, India, Russia, Nepal, Germany.
International organization that oversees these financial
B. US, Germany, Japan, France, UK, Italy, Canada,
institutions and regulations that act at the International
Russia, China and Saudi Arabia.
level. The IMF has 189 member countries (as on 12 April
C. US, Pakistan, Bhutan, Nepal, Singapore, Canada,
2016) but they do not enjoy an equal say. The top ten
China and Saudi Arabia.
countries have more than 52 per cent of the votes. They are
D. US, Japan, Netherlands, Bhutan, Afghanistan, Italy,
the G-8 members (The US, Japan, Germany, France, the UK,
Iraq.
Italy, Canada and Russia), China and Saudi Arabia. The US
alone has 16.54 per cent voting rights. 149. What is the voting per cent of the top ten countries?
A. 65% B. 75%
145. What is IMF?
C. 85% D. 52%.
A. International Monetary Fund is an international
organization that it oversees those financial 150. The UN Security Council have ______ Permanent
institutions and regulations that act at the members and ______ non-permanent members.
international level A. Ten permanent and fifteen non-permanent member
B. Indian Making Form is an international countries
organization that it oversees those financial B. Ten permanent and ten non-permanent member
institutions and regulations that act at the countries
international level. C. Ten permanent and five non-permanent member
C. International marking Fees is an international countries
organization that it oversees those financial D. Five permanent and ten non-permanent countries
(2751) Pol. Sci.—11
82 Political Science

151. The present Secretary General of the UN is: 163. The International Court of Justice popularly known as
A. Ban Ki-moon B. Kofi A. Annan the
C. Antonio Guterres D. U Thant A. Supreme Court B. World Court
C. Criminal Court D. Subordinate Courts
152. The prime objective of UN is:
A. To maintain peace and security 164. The largest international human rights organization in
B. It is the major financial contributor of the world _______ .
C. To encourage conflicts A. Russia B. Germany
D. None of these C. Canada D. US
153. The total number of members represented in General 165. What is the full form of NGO?
assembly is: A. Non-gazette office
A. 191 B. 195 B. Non-Governmental Organization
C. 193 D. 197 C. Nation governing organization
D. None of these
154. The World Trade Organization (WTO) formed on 1st
January: 166. The more weightage to India’s proposal for permanent
A. 1994 B. 1995 membership in the Security Council is:
C. 1996 D. 1997 A. Nuclear capability
B. Located in Asia
155. Check which is not an agency of UN?
C. India’s membership in the UN
A. International Committee of Red Cross
D. India’s growing economic power and stable
B. UNDP
political system
C. IMF
D. ILO 167. How many judges are there in the International Court
of Justice?
156. India became a member of the UN in:
A. Fifteen judges B. Sixteen judges
A. 1945 B. 1947
C. Seventeen judges D. Eighteen judges
C. 1950 D. 1962
168. Which one of the following is not a permanent member
157. Which of the following organizations is meant for
of the UN Security Council?
Children?
A. Russia B. India
A. UNESCO B. UNICEF
C. China D. France
C. UNHCR D. UNHRC
169. The highest functionary of the UN is called ______.
158. The office of the UN General Assembly:
A. General Assembly B. Secretariat
A. New York B. Washington DC
C. Security Council D. None of these
C. San Francisco D. Zurich
170. How many countries are members of the UN?
159. Currently in 2016 UN peacekeeping operations are:
A. 188 B. 190
A. 15 B. 14
C. 192 D. 193
C. 17 D. 16
171. What is Amnesty International?
160. Which of the following has veto power in the Security
A. it is an NGO (Non Governmental Organization) at
Council?
international forum
A. India B. Pakistan
B. it is an organization
C. Brazil D. America C. it is a government organization
161. The United Nations was not created to take humanity D. None of these
to the heaven, but to save it from the hell. Who made 172. Who has a right to vote in the UN?
this statement?
A. Each member nation of UN
A. Pt. Jawaharlal Nehru B. Kofi Annan B. Only five permanent member countries
C. Ban Ki-moon D. Dag Hammarskjold C. Only non-permanent member countries
162. Expand IAEA? D. None of these
A. International Atomic Energy Act 173. The UN agency concerned with the safety and peaceful
B. International Atomic Energy Accord use of nuclear technology:
C. International Atomic Energy Agency A. The UN Committee on Disarmament
D. International Atomic Eastern Agency B. International Atomic Energy Agency
(2751) Pol. Sci.—11-II
Political Science 83
C. UN International Safeguard Committee 183. The term of office of the UN Secretary-General is:
D. None of these A. Three years B. Four years
174. Which of the following oversees those financial C. Five years D. Six years
institutions and regulations that act at the international 184. Which of the following is Human Rights Organisation?
level? A. The French Community
A. WTO B. IAEA B. The Arab League
C. IMF D. WHO C. The organization of African Unity
175. Which of the following is suggestion for becoming a D. Amnesty International
new member of the Security Council? 185. Which of the following organizations is meant for
A. A major economic power Children?
B. A major military power A. UNESCO B. UNICEF
C. A substantial contributor to the UN budget C. UNHCR D. UNHRC
D. All of the above
186. Which of the following is a genuine contender for
176. Name any two member states of the European Union becoming a permanent member of Security Council?
who are permanent members of the UN Security A. China B. France
Council? C. India D. Pakistan
A. India and Brazil
187. Arrange the following in the chronological order:
B. Britain and France
C. Singapore and Malaysia (a) Establishment of Human Rights Council
D. Andaman and Lakshadweep (b) Yalta Conference
(c) Atlantic Charter
177. Where are the head quarters of WTO?
A. New York (d) India joins the UN
B. Washington DC A. (b), (d), (a), (c) B. (c), (b), (d), (a)
C. Geneva (Switzerland) C. (a), (b), (c), (d) D. (b), (c), (d), (a)
D. New Zeeland 188. Arrange the following in the chronological order of
178. The number of principal organs of UNO is ______. their establishment:
A. 3 B. 4 (a) The WTO
C. 5 D. 6 (b) Amnesty International
179. Which organization of United Nations has ceased to (c) League of Nations
be operational? (d) World Bank
A. Trusteeship Council A. (c), (d), (a), (b)
B. Secretariat B. (d), (c), (b), (a)
C. International Court of Justice C. (d), (c), (b), (a)
D. Economic and Social Council D. (a), (b), (c), (d)

180. Which of the following countries is not a member of 189. Assertion (A) : The World Bank was created during
the G-8 group? the Second World War in 1944.
A. Germany B. France Reason (R) : The Soviet Union has collapsed.
C. Italy D. Spain In the context of the above two statements which one
181. The head quarters of International Court of Justice are of the following is correct?
at: A. Both (A) and (R) are true and (R) is the correct
A. Geneva B. Rome explanation of (A)
C. The Hague D. Vienna B. Both (A) and (R) are true, but (R) is not the
correct explanation of (A)
182. What is the purpose of ‘United Nations Conference on C. (A) is true, but (R) is false
UNCTAD? D. (A) is false, but (R) is true
A. Promotes International Trade with a view to acce-
lerate economic growth of developing countries 190. Assertion (A) : US and other western countries want
B. Set rules for World Trade improvements in the UN’s budgetary
C. Promotes International Monetary cooperation and procedures and its administration.
expansion of International Trade Reason (R) : The UN consists of many different
D. None of the above structures and agencies.
84 Political Science

In the context of the above two statement which one 194. Assertion (A) : The biggest discussion has been on
of the following is correct? the functioning of the Security
A. Both (A) and (R) are individually true, but (R) is Council.
not the correct explanation of (A) Reason (R) : The organization was setup through
B. Both (A) and (R) are individually true and (R) is the signing of the United Nations
the correct explanation of (A) Charter by 51 states.
C. (A) is true, but (R) is false
In the context of the above two statements which one
D. (A) is false, but (R) is true
of the following is correct?
191. Assertion (A) : In the Security Council, there are five A. Both (A) and (R) are true and (R) is the correct
permanent members and ten non- explanation of (A)
permanent members. B. Both (A) and (R) are true, but (R) is not the
Reason (R) : Charter gave the permanent members correct explanation of (A)
a privileged position to bring about C. (A) is true, but (R) is false
stability in the world.
D. (A) is false, but (R) is true
In the context of the above two statements which one
195. Assertion (A) : UN Created for peaceful building
of the following is correct?
Commission.
A. Both (A) and (R) are true and (R) is the correct
Reason (R) : To prevent World Wars.
explanation of (A)
B. Both (A) and (R) are true, but (R) is not the In the context of the above two statements which one
correct explanation of (A) of the following is correct?
C. (A) is true, but (R) is false A. Both (A) and (R) are true and (R) is the correct
D. (A) is false, but (R) is true explanation of (A)
192. Assertion (A) : We should keep in mind that the B. Both (A) and (R) are true, but (R) is not the
membership of the UN Security correct explanation of (A)
Council was expanded from 11 to 15 C. (A) is true, but (R) is false
in 1965. D. (A) is false, but (R) is true
Reason (R) : UN General Assembly members now 196. Assertion (A) : World Trade Organisation is an
are developing countries. international organisation.
In the context of the above two statements which one Reason (R) : It sets the rules for global trade.
of the following is correct? In the context of the above two statements which one
A. Both (A) and (R) are individually true and (R) is of the following is correct?
the correct explanation of (A) A. Both (A) and (R) are true and (R) is the correct
B. Both (A) and (R) are true, but (R) is not the explanation of (A)
correct explanation of (A) B. Both (A) and (R) are true, but (R) is not the
C. (A) is true, but (R) is false
correct explanation of (A)
D. (A) is false, but (R) is true
C. (A) is true, but (R) is false
193. Assertion (A) : The International Atomic Energy D. (A) is false, but (R) is true
Agency (IAEA) was established in
197. Assertion (A) : Only the permanent members of the
1957.
Security Council posses the veto
Reason (R) : IAEA teams did not inspect nuclear
power.
facilities all over the world to ensure
Reason (R) : One veto can stall a Security Council
that civilian reactors are not being
used for military purposes. resolution.

In the context of the above two statements which one In the context of the above two statements which one
of the following is correct? of the following is correct?
A. Both (A) and (R) are true and (R) is the correct A. Both (A) and (R) are true and (R) is the correct
explanation of (A) explanation of (A)
B. Both (A) and (R) are true, but (R) is not the B. Both (A) and (R) are true, but (R) is not the
correct explanation of (A) correct explanation of (A)
C. (A) is true, but (R) is false C. (A) is true, but (R) is false
D. (A) is false, but (R) is true D. (A) is false, but (R) is true
Political Science 85
198. Assertion (A) : Humanitarian policies are imple- 203. Assertion (A) : India was divided into 14 states and
mented by the main organs and 6 union territories by the state
specialized agencies spread across the Reorganisation Act 1956.
globe. Reason (R) : The creation of linguistic provinces
Reason (R) : The members of the General Assembly has helped a lot in instilling the
are automatically the members of all fellings of regionalism in the Indian
other principal organs and specialized public.
agencies of the UN.
204. Assertion (A) : Nehru, believing in secularism,
In the context of the above two statements which one supported the declaration of India as
of the following is correct? a secular state. A secularist state
A. Both (A) and (R) are true and (R) is the correct
means that the state will not have any
explanation of (A)
religion of its own and will treat all
B. Both (A) and (R) are false
religions equally.
C. (A) is true, but (R) is false
D. (A) is false, but (R) is true Reason (R) : Nehru believed that a strong nation
can be built by making India a secular
Directions (Qs. No. 199-208): The following questions
state.
consists of two statements—Assertion (A) and Reason (R).
Answer these questions. Selecting the appropriate option 205. Assertion (A) : The first deputy prime minister and
given below. hom e minister of India, Sardar
A. Both (A) and (R) are true and (R) is the correct Vallabhbhai Patel, emerged as a Iron
explanation of (A) man of India.
B. Both (A) and (R) are true and (R) is not the correct Reason (R) : Sardar Vallabhbhai Patel faced key
explanation of (A) challenges of integration of
C. (A) is true, but (R) is false Hyderabad, Junagarh, Kashmir.
D. (A) is false, but (R) is true
206. Assertion (A) : Partition of India was the outcome of
199. Assertion (A) : The partition of the country proved the “two-nation theory”.
everyone’s worst tears. There were
Reason (R) : Punjab and Bengal were the two
serious questions about the future of
provinces divided on the basis of
India.
religion.
Reason (R) : British government decided to give
independence to India 207. Assertion(A) : Princely states covered one third of
the land area of the British Indian
200. Assertion (A) : There were no single belt of uniform
Empire.
Muslim majority areas in British India.
There were two areas of concentration Reason (R) : Some of the princely states clearly
one in the west and in the east. wanted to become part of the Indian
Reason (R) : Not all Muslim majority areas wanted Union.
to join with Pakistan. 208. Assertion (A) : Nehru, the architect of India’s nation-
201. Assertion (A) : Writers, poets, and film makers in India building had clearly specified that
and Pakistan have often used the India was not a mere geographical or
phrase division of hearts to describe historical entity. According to him
partition. India had a unique capability to
Reason (R) : Muslim league put forward the Two synthesise alien culture and this had
nation theory that British did not been the secret of India’s unity in
consist of one but two people. diversity.
202. Assertion (A) : The ruler of Travancore decided to Reason (R) : In the 21st century, the whole world
remain independent and Nizam of has woken up to the facet of this
Hyderabad made a similar announce- diversity. Selfish and petty politicians
ments the next day. have sowed seeds of disintegration
Reason (R) : With the end of British rule in India, by using ethnic, lingual and cultural
paramountcy of the British crown over factionalism for narrow political
princely rulers also came to an end. gains.
86 Political Science

Directions (Qs. No. 209-213): Read the following passage 214. With which democracy is aligned?
and answer the questions. A. In ideas and life method
Patel faced key challenges of integration from three B. Politics and government
states, viz., Hyderabad, Junagarh and Kashmir. It was under C. Social and economic life
his leadership that Indian forces compelled Hyderabad and D. None of the above
Junagarh to merge with India. Keeping well-versed with 215. What is meaning of choosing democracy?
Pakistan’s intentions from jinna’s divisive “Two Nation A. To recognise unity and accepting it
theory”, Sardar Patel’s opinion on Kashmir was different B. To recognise diversity and not accepting it
from other leaders. Like Hyderabad, he also wanted C. To recognise diversity and accepting it
Kashmir’s integration with India through military D. All of the above
operations. But due to political decisions of some prominent
leaders, Sardar could not Succeed in integrating Kashmir 216. For linguistic states reorganisation, which principle
fully with India which later turned into a major historical was permitted?
blunder for the country. A. Unity
B. Diversity
However, Sardar will always remain as an astounding C. Culture
leader who combined himself the features of a true D. Civilisation
‘Nationalist’, ‘Catalyst’ and ‘Realist’— popularly
characterized as NCR in Indian political history. 217. In diversity there may be mutual ______
A. Protest B. Support
209. In which form Sardar Patel is known as?
C. War D. None of the above
A. VCR B. NCR
C. MCR D. NRC 218. What is the meaning of will of the people?
A. democracy B. autocracy
210. Which state of India could not be unified due to some
C. monarchy D. None of the above
major leaders’ short-sighted political decision?
A. Manipur 219. Which one of the following is a princely states of
B. Junagarh India initially resisted to join the Indian Union?
C. Hyderabad A. Baroda B. Hyderabad
D. Kashmir C. Mysore D. Gwalior
211. In which states, Sardar Patel had to face challenges to 220. When Gujarat was carved out of Bombay?
unite with Indian Union? A. 1950 B. 1960
A. Hyderabad, Manipur, Goa C. 1970 D. 1980
B. Hyderabad, Junagarh, Goa 221. Amrita Pritam was a prominent poet from:
C. Goa, Puducherry and Junagarh A. Punjab B. Gujarat
D. Hyderabad, Junagarh, Kashmir C. Mumbai D. Punjab
212. Sardar Patel was a ____. 222. The cities that were divided into “communal zones”
A. Nationalist B. Changer during the partition violence were.
C. Realistic D. All the above A. Lahore, Amritsar and Calcutta
213. In the matter of which state of India committed blunder B. Kashmir, Lucknow and Allahabad
mistake? C. Madras, Hyderabad and Mysore
A. Kashmir B. Hyderabad D. Delhi, Mumbai and Gwalior
C. Manipur D. Goa
223. The states created in 1960 were:
Directions (Qs. No. 214-218): Read the following passage A. Maharashtra and Gujarat
and answer the questions. B. Odisha and West Bengal
Most important is that with linguistic states permission C. Rajasthan and Gujarat
was given to ‘doctrine of diversity’. When we say that India D. Punjab and Haryana
has opted democracy. There its broader meaning in Indian 224. The interim government formed under the cabinet
democracy. Democracy was chosen for recognising mission plan was headed by:
diversities and accepting them. Besides we also have to A. Dr. Rajendra Prasad
accept that in diversities, there might be conflicts in them. B. Jawaharlal Nehru
In other words, democracy in India is related to ideas, life C. Sardar Vallabhbhai Patel
methods and their multiplicity. D. Rajagopalachari
Political Science 87
225. How many princely states existed at the time of C. Mohammed Ali Jinnah : Wanted to remain with
Independence of India? Indian Union
A. 560 B. 562 D. All are True
C. 563 D. 565
232. How many kinds of challenges that Independent India
226. When Meghalaya was carved out of Assam? faced?
A. 1970 B. 1971 A. 1 B. 2
C. 1972 D. 1973 C. 3 D. 5
227. Who is called “Iron man of India”? 233. Identify the wrongly matched :
A. Jawaharlal Nehru B. Vallabhbhai Patel A. Dr. B.R. Ambedkar – Chairman of drafting
C. Rajendra Prasad D. Muhammad Ali Jinnah committee
228. Which among the following statements about the B. Moulana Abdul – First Education
partition is incorrect? Kalam Azad Minister
A. Partition of India was the outcome of the ‘Two C. Sardar Vallabhbhai – First Finance Minister
Nation Theory.’ Patel
B. Punjab and Bengal were the two provinces divided D. Dr. Babu Rajendra – First President of India
on the basis of religion. Prasad
C. East Pakistan and West Pakistan were not 234. The state which has achieved nearly total literacy in
contiguous. India is:
D. The scheme of partition included a plan for the A. Tamil Nadu B. Andhra Pradesh
transfer of population across the border. C. Kerala D. Manipur
229. Match the principles with the instances: 235. Who said, “Planning is a way of organizing and
(a) Mapping of boundaries (i) Pakistan and utilizing resources to maximum advantage in terms of
of religious grounds Bangladesh defined social ends”?
(b) Mapping of boundaries (ii) India and
A. K.T. Shah
on grounds of a Pakistan
B. Planning commission of India
different language
C. Dr. Rajendra Prasad
(c) Demarcating boundaries (iii) Jharkhand and
D. None of these
within a country by Chhattisgarh
geographical zones 236. The idea of Five Year Plan was adopted in India from:
(d) Demarcating within a (iv) Himachal Pradesh A. United States of America
country on administrative and Uttarakhand B. Japan
and political grounds C. United Soviet Socialist Republics
Codes: D. Australia.
(a) (b) (c) (d) Directions (Qs. No. 237-239): Read the following passage
A. (ii) (i) (iv) (iii) and answer the questions.
B. (i) (iv) (ii) (iii)
Based in Anand, a town in Gujarat, Amul is a dairy
C. (iii) (i) (ii) (iv)
cooperative movement joined by about 2 and half million
D. (iv) (iii) (ii) (i)
milk producers in Gujarat. The Amul pattern became a
230. Arrange the following the chronological order of uniquely appropriate model for rural development and
occurrence. poverty alleviation, spurring what has come to be known
(a) Birth of Pakistan as the White Revolution. In 1970 the rural development
(b) Join of Hyderabad in Indian Union programme called Operation Flood was started. Operation
(c) Carving out of Arunachal Pradesh from Assam Flood organised cooperatives of milk producers into a
(d) Appointment of State Re-organisation commission nationwide milk grid, with the purpose of increasing milk
A. (a), (b), (d), (c) B. (d), (a), (c), (b) production, bringing the producer and consumer closer by
C. (d), (b), (a), (c) D. (a), (b), (d), (c) eliminating middlemen, and assuring the producers a regular
231. Which of the following statement is not false? income throughout the year. Operation Flood was, however,
A. Mahatma Gandhi wanted to become the Prime not just a dairy programme. I’ll saw dairying as a path to
Minister of India development, for generating employment and income for
B. The year 1947 was a year of unprecedented rural households and alleviating poverty. The number of
violence and the year of displacement members of the cooperative has continued to increase with
88 Political Science

the numbers of women members and Women’s Dairy 244. Which one of the following statements about Bombay
Cooperative Societies also increasing significantly. plan is incorrect?
237. In which pace was Amul Diary Co-operative movement- A. It was a blue print for Indias economic future
blasted? B. It supported state ownership of industry
A. Anand, Gujarat C. It was made by some leading industrialists
B. Porbandar, Gujarat D. It supported strongly the idea of planning
C. Gandhinagar, Gujarat 245. During Nehru era, parties and groups in the country
D. Ahmedabad, Gujarat believed that India should become friendly with block
238. In which year was Operation Flood started? led by USA.
A. 1971 B. 1972 I. Because that bloc claimed to be pro democracy.
C. 1970 D. 1973 II. Dr Ambedkar also supported this idea. Some
political parties opposed communism
239. Operation flood is related to ____________
III. Some political parties opposed communism
A. Green Revolution B. White Revolution
IV. India joined Warsaw pact.
C. Russian Revolution D. French Revolution
Select the correct answer choosing the four given
Directions (Qs. No. 240-241): The following questions
above:
consist of two statements—Assertion (A) and Reason (R).
A. II, III B. I, II, IV
Answer these questions selecting the appropriate option
C. I, III, IV D. I, II only
given below:
A. Both (A) and (R) are true and (R) is the correct 246. What considerations should be kept in mind to set up
explanation of (A) an industry?
B. Both (A) and (R) are true, but R is not the correct A. Profit making
explanation of (A) B. Role of environmentalists
C. (A) is true, but (R) is false C. Not to ask local people where it will be set up
D. (A) is false, but (R) is true D. Approval of USA
240. Assertion (A) : The Green Revolution benefited all 247. Consider these statements about White Revolution.
the sections of Indian society. Find the Wrong statement.
Reason (R) : The rich peasants and the large A. It is a revolution in the field of milk production
landholders were the m ajor B. It is also known as operation flood
beneficiaries of the process. C. Amul diary is located in Gujarat
241. Assertion (A) : Land reforms did not take place D. It is not helping the local villagers
effectively in most parts of the 248. Consider these statements about POSCO Plant. Find
country. the wrong statement:
Reason (R) : Political Power remained in the hands A. Odisha has one of the largest resources of untapped
of land-owning classes. iron ore in the country
242. Arrange the following in chronological order of their B. POSCO plant may displace local tribals
occurrence. C. Odisha villagers protested against this
I. The first five year plan D. Odisha is located to the north west of India
II. The great economic depression of Europe 249. Green Revolution benefited only few states.
III. Partition of India
IV. The second five year plan Identify correct option:
A. I, II, III, IV B. IV, II, III, I A. Punjab, Haryana, Western Uttar Pradesh
C. I, IV, III, II D. II, III, I, IV B. Andhra Pradesh, Madhya Pradesh, Uttar Pradesh
C. Odisha, West Bengal, Madhya Pradesh
243. The three major objectives of first five year plan were:
D. Kerala, Karnataka, Tamilnadu
I. Agriculture
II. Dams 250. Article 51 of the Indian Constitution lays down some
III. Irrigation provisions regarding:
IV. Education A. Powers of Prime Minister
Select the correct answer using the four given above: B. Fundamental Rights
A. I, II, III B. II, III, IV C. Promotion of International peace and security
C. III, II, IV D. IV, III, I D. Fundamental Duties
Political Science 89
251. When was Britain attacked Egypt over the Suez Canal 263. The Tibetan spiritual leader, the Dalai Lama, sought
issue? and obtained political asylum in India in:
A. In 1965 B. In 1956 A. 1956 B. 1959
C. In 1966 D. In 1955 C. 1960 D. 1958
252. When was USSR invaded Hungary? 264. After Nehru, the first Prime Minister to visit China
A. In 1976 B. In 1955 was:
C. In 1956 D. In 1965 A. Lal Bahadur Shastri B. Indira Gandhi
253. The Afro-Asian conference held in the Indonesian city C. Rajiv Gandhi D. V.P. Singh
of Bandung in: 265. When was the Communist Party of India spilt into
A. In 1955 B. In 1956 two, (CPIM & CPI):
C. In 1965 D. In 1954 A. In 1965 B. In 1964
254. Which Conference marked the zenith of India’s C. In 1963 D. In 1966
engagement with newly independent Asian and Africa 266. The Indus Waters Treaty was signed between by:
nations? A. Lal Bahadur Shastri & Imran Khan
A. Malaysian Conference B. Egyptian Conference B. Nehru & Ayub Khan
C. Singapore Conference D. Bandung Conference C. Indira Gandhi & Musharaf
255. The First Summit of the NAM was held in Belgrade D. Vajpayee & Mushrid Khan
in: 267. Expand NEFA:
A. September 1916 B. September 1960 A. North Eastern Far Agency
C. September 1963 D. September 1961
B. North Eastern Frontier
256. Who was the Architect of Non-alignment Movement? C. North Eastern Fullest Agency
A. Indira Gandhi B. Atal Behari Vajpayee D. North Eastern Frontier Agency
C. Jawaharlal Nehru D. Mahatma Gandhi
268. The Treaty assured India of Soviet support if the
257. After the Chinese revolution in 1949, which country country faced any attack:
was the first to recognize the Communist Government? A. Treaty of Peace B. Water Treaty
A. India B. Pakistan C. Panchsheel Treaty D. Tashkent Agreement
C. Bangladesh D. USSR
269. Which agreement was signed between Indira Gandhi
258. In the Nehru cabinet who was apprehensive of the and Zulfikar Ali Bhutto on 3 July 1972?
future attack from China? A. Water Treaty B. Treaty of Peace
A. Jawaharlal Nehru B. Vallabhbhai Patel C. Tashkent Agreement D. Shimla Agreement
C. Jagjivan Ram D. C. Rajagopalachari
270. Which among the following is not a principle of India’s
259. The Panchsheel agreement was signed between India Nuclear Doctrine today?
and: A. Not first use
A. Pakistan B. China B. Credible Minimum deterrent
C. Bangladesh D. USA C. Civilian Control
260. When was the Panchsheel agreement signed between D. First Use
India and China?
271. When did India and Israel establish full diplomatic
A. On 29 April 1954 B. On 28 April 1955
relations in?
C. On 30 April 1056 D. On 29 June 1954
A. In January 29, 1990 B. In January 29, 1992
261. In _________, China took over control of Tibet. C. In January 29, 1991 D. In January 29, 1989
A. 1951 B. 1950
272. The period when the nuclear test was conducted was
C. 1952 D. 1949
a difficult period in:
262. Who accompanied the Tibetan spiritual leader Dalai A. Domestic Politics B. Foreign Politics
Lama during the official Chinese visit to India in C. Military dominance D. Financial world.
1956?
273. India has developed self-sufficiency in __________
A. Hua Guofeng
B. Chinese Premier Zhou Enlai technology.
C. Li Peng A. Atomic B. Nuclear
D. Zhao Ziyang C. Electricity D. Scientific
(2751) Pol. Sci.—12
90 Political Science

274. Five principles of peaceful co-existence are called Directions (Qs. No. 282-286): Read the following passage
_________. carefully and answer the questions that follows:
A. Punchsheel India has opposed the international treaties aimed at
B. SAARC non-proliferation since they were selectively applicable to
C. India’s foreign policy the non-nuclear powers and legitimised the monopoly of
D. Treaty Peace
the five nuclear weapons powers. Thus, India opposed the
275. Sri Lanka is member of __________. indefinite extension of the NPT in 1995 and also refused
A. UNESCO B. WHO to sign the Comprehensive Test Ban Treaty (CTBT). India
C. G7 D. SAARC conducted a series of nuclear tests in M ay 1998,
276. Both domestic and international environment influence demonstrating its capacity to use nuclear energy for military
the ______ policy of nation. purposes. Pakistan soon followed, thereby increasing the
A. Nuclear B. Foreign vulnerability of the region to a nuclear exchange. The
C. Peaceful D. Deterrence international community was extremely critical of the
nuclear tests in the subcontinent and sanctions were imposed
Directions (Qs. No. 277-281): Read the following passage on both India and Pakistan, which were subsequently
and answer the questions that followed. waived. India’s nuclear doctrine of credible minimum
Under leadership of Jawaharlal Nehru, India convened nuclear deterrence professes “no first use” and reiterates
the Asian Relations Conference in March 1947, five months India’s commitment to global, verifiable and non-
ahead of attaining its independence. India made earnest discriminatory nuclear disarmament leading to a nuclear
efforts for the early realisation of freedom of Indonesia weapons free world.
from the Dutch colonial regime by convening an
Foreign policy is always dictated by ideas of national
international conference in 1949 to support its freedom
interest. In the period after 1990, Russia, though it
struggle. India was a staunch supporter of the decolonisation
continues to be an important friend of India, has lost its
process and firmly opposed racism, especially apartheid in
global pre-eminence. Therefore, India’s foreign policy has
South Africa. The Afro-Asian conference held in the
shifted to a more pro-US strategy.
Indonesian city of Bandung in 1955, commonly known as
the Bandung Conference, marked the zenith of India’s 282. Which of the following nuclear treaties were rejected
engagement with the newly independent Asian and African by India?
nations. The Bandung Conference later led to the A. NPT, CTBT
establishment of the NAM. The First Summit of the NAM B. Kyoto Protocol
was held in Belgrade in September 1961. Nehru was a co- C. Panchsheel Agreement
founder of the NAM. D. None of the above
277. Under whose leadership, India convened the Asian 283. When did India conduct series of nuclear tests?
Relations Conference in March 1947? A. June 1998 B. May 1998
A. Jawaharlal Nehru C. April 1998 D. March 1998
B. Indira Gandhi
284. Why India’s foreign policy shifted to become more
C. Lal Bahadur Shastri
D. Vallabhbhai Patel pro-US?
A. Because Russia betrayed India
278. In which city the Afro-Asian conference held? B. Because Russia attacked India
A. Belgrade B. Indonesia C. Because US derived more profit to India
C. Bandung D. Africa D. Because Russia lost its global pre-eminence
279. Where was the first Summit of NAM held? 285. Which one of the following is India’s stand for the use
A. Indonesia B. Bandung of nuclear weapon?
C. Belgrade D. Dutch A. No use of nuclear weapon at all
280. Who was the co-founder of NAM? B. No first use of nuclear weapon
A. Indira Gandhi B. Jawaharlal Nehru C. Use nuclear weapon in case of war
C. Vallabhbhai Patel D. Lal Bahadur Shastri D. None of the above
281. Which country made earnest efforts for the early 286. In which year India opposed the indefinite extension
realization of freedom of Indonesia? of the NPT.
A. Pakistan B. India A. In 1996 B. In 1995
C. Dutch D. Belgrade C. In 1994 D. In 1993
Political Science 91
Directions (Qs. No. 287-293): The following questions Reason (R) : The term ‘Panchsheel’ is related to
consists of two statements—Assertion (A) and Reason (R). the five moral principles of Lord
Answer these questions by Selecting the appropriate option Buddha. Lord Buddha made the five
given below. principles obligatory for the life of
A. Both (A) and (R) are true and (R) is the correct an individual. In international life, the
explanation of (A) term Panchsheel was used in 1954
B. Both (A) and (R) are true and (R) is not the correct when India recognised the sovereignty
explanation of (A) of China over Tibet.
C. (A) is true, but (R) is false 293. Assertion (A) : Six-point proposal of Sheikh Mujib –
D. (A) is false, but(R) is true ur Rehman for greater autonomy to
287. Assertion (A) : The first UN General Assembly East Pakistan.
Resolution of 1946 called for Reason (R) : India and Bangladesh sign the
elimination of atomic weapons and Farakka Treaty for sharing of the
weapons of mass destruction. Ganga waters in 1996.
Reason (R) : In 1948 India called for elimination of Directions (Qs. No. 294-298): Read the passage and answer
all nuclear weapons and use of nuclear the questions.
energy only for peaceful purposes.
From this perspective of democracy, representation can
288. Assertion (A) : India opposes the NPT (1968) because help or hurt democracy, even if representation cannot be
the treaty is unequal. democratic since it cannot replicate the demos. But the
Reason (R) : In 1974 India had carried out phenomenon of representation reflects an emotional roller
underground nuclear tests for peaceful - coaster - great hopes during an election campaign, great
purposes. disappointment from the work of governments in office.
This problem stems in part from the fact that representative
289. Assertion (A) : In 1964 China conducted its nuclear
government and, the demos potentially operate at cross -
test. In 1962 China had attacked India.
purposes. The office of a representative results from
Reason (R) : The decision to go ahead and manu-
elections, which elevate a set of political rulers above the
facture the atomic bomb given by
ruled in order to make the laws. In office, they make
Prime Minister Indira Gandhi in 1974.
com prom ises to get legislative business done.
290. Assertion (A) : Non-alignment allowed India to gain Representatives may feel proud of what they accomplished,
assistance both from USA and USSR. because they made the decisions that led to the compromise.
Reason (R) : The cold war has affected the relation- But citizens can’t take pride in legislative work, and they
ship between India and Pakistan. mostly suffer the consequences required by the compromise.
That doesn’t mean representatives always will be scorned,
291. Assertion (A) : The Indo-Pak war of 1965 was the
as mentioned above, their office-enables them to undertake
culmination of a series of disputes
political initiatives not viable during an election campaign
between India and Pakistan. Within or to protect minorities whose needs may have been
the first year of independence both neglected. That said, the necessary activity of legislative
the countries witnessed tension over compromise takes place above the heads and behind the
the issue of Kashmir and refugee backs of the people. As such, it contradicts the ‘Athenian
exchange due to partition. and Aristotelian understanding of democracies as political
Reason (R) : In 1965 Soviet Union started deve- orders that depend on the deliberative and decisive power
loping close relations with Pakistan. of citizens (i.e., the demos) acting with authoritative power.
Despite the efforts of Shastri, Indian Citizens of the Athenian democracy were said to rule and
Prime Minister, Soviet Union was not be ruled in turn, but in every case their power required the
convinced. Using this opportunity, on political presence and deliberative action of citizens.
5th August, 1965 Pakistan stationed
troops along the LOC. 294. Which of the following is reflected in the phenomenon
of representation?
292. Assertion (A) : Since 1947, India has been following A. Great hopes during an election campaign
an independent foreign policy of non- B. Great appreciation for the work of government in
alignment and this policy is in perfect office
accord with the policy of friendly C. Great promises from the representative democracy
relations with all the countries of the D. Great challenges from liberalization, privatisation
world. and globalisation
92 Political Science

295. What is the obvious outcome of an election? traffic movement does not mean restraining the rights, on
A. Career of a leader the other hand, putting a restriction on publication or
B. Ministerial birth of a politician restraining the freedom of speech will infringe the rights
C. Office of a representative even if it enhances general welfare. Thus, even though in
D. Future prospects of a budding activist both cases rights are involved, curtailment of one is justified,
296. Athenian and Aristotelian understanding of whereas the other is not.
democracies means: 299. The rights claimed against the state are considered
A. Political orders depending on the deliberative as:
powers of citizens A. Fundamental rights B. Human rights
B. Political system rooted in grassroot politics C. Natural rights D. Legal rights
C. System of government prevalent in Greek city
300. The idea of right against the government becomes
states
very useful when the society is divided into majority
D. Western conceptualization of democracy
and minority on the basis of:
297. What takes place above the heads and behind the A. Caste B. Class
backs of the people? C. Race D. Religion
A. Executive arrogance
301. The strong rights need to be:
B. Judicial activism
A. inferred B. permitted
C. Parliamentary sovereignty
C. borrowed D. ordained
D. Legislative compromise
302. Which of the following is a weak right?
298. Why could have representatives felt proud of what
A. Right to property
they accomplished?
B. Right to equality
A. They made the decisions for their followers
C. Right to liberty
B. They m ade the decisions that led to the
D. Right to religious freedom
compromise
C. They made a level jump in their career progression 303. The rights that can be curtailed to achieve common
D. They made their adversaries compromise with their welfare may be called as:
core values A. Strong rights B. Fundamental rights
C. Human rights D. Weak right
Directions (Qs. No. 299-303): Read the passage and answer
the questions. 304. ‘All aspects of luck, including natural ability, should
Ronald Dworkin’s thesis is that some of the rights be irrelevant to distributive justice’— This statement
people have fundamental, but many are not. They are refers to the concept of:
fundamental because they are ‘the rights against the state’. A. Meritocracy B. Plutocracy
Dworkin says that the notion of rights as trumps “marks the C. Luck egalitarianism D. Aristocracy
distinctive conept of an individual right against the state 305. Which among the following is NOT a salient feature
which is the heart ... of constitutional theory in the United of ‘individualist anarchism’?
States.” In the special supplement, Dworkin argued that the A. Private property B. Ultra liberalism
concept of right against the government becomes most C. Civil disobedience D. Organicism
useful particularly when the society is divided on racial
306. The theory that ‘the Earth is best understood as a
lines into majority and minority. The right to freedom of
living entity that acts to maintain its own existence’
speech and religious freedom belong to this category.
is known as:
Therefore, they are strong rights. He asserts that these rights
A. Tragedy of commons B. Sustainability
should be permitted and they should not be interfered or
C. Social ecology D. Gaia hypothesis
banned. These freedoms are to be allowed even if the
welfare of the collectivity is infringed. 307. Which among the following countries have no written
On the contrary, he also presupposes a large area of constitution?
rights where the state can make legislations curtailing those A. Australia B. New Zealand
rights to achieve common welfare. They can be called C. Spain D. France
weak rights. Right to liberty for example becomes a weak 308. Which among the following forms of government is
right against right to equality, a strong right. This at the ideal for Machiavelli?
outset may look contradictory, but it is not. For example, A. Republic B. Aristocracy
restraining the movement on the roads for safety and smooth C. Democracy D. Hereditary Monarchy
Political Science 93
309. Which of the following thinkers did not believe in 318. Which article of the Indian Constitution is associated
women equality? with judicial activism?
A. Plato and Mill A. 14 B. 18
B. Aristotle and Hegel C. 21 D. 25
C. Mill and Mary Wollstonecraft 319. Who among the following was strongly opposed to
D. Plato and Mary wollstonecraft the idea of Panchayati Raj system in India?
310. In “The Considerations on Representative A. B.R. Ambedkar
Government”, J.S. Mill did not elaborate upon: B. Jaya Prakash Narayan
A. Ideal government based on proportional C. Ram Manohar Lohia
representation D. Mahatma Gandhi
B. Protection of minorities 320. Which of the following statement is false about the
C. Institutions of self-government Indian federalism?
D. Political Economy A. it is a good example of ‘holding together’
311. Aristotle’s “ideal state is always Plato’s second best:” federation
Who said this? B. it has a tendency of strong Central Government
A. G.H. Sabine B. W. Ebenstein C. it reflects the spirit of ‘coming together’ federation
C. J.A. Dunning D. C.L Wayper D. it has been referred to as ‘quasi federal’ system

312. Which one of the following is not the function of 321. Who has given originally the idea of ‘Hindu Rashtra’?
State, according to Kautilya? A. V.D. Savarkar B. K.B. Hedgewar
A. Acquisition of that which is not under its control C. B.S. Moonje D. M.S. Golwalkar
and possession 322. Who among the following characterized the Indian
B. Preservation of what has been acquired polity as being stable not in the sense of a stationary
C. Accentuation of what has been preserved unresponsive state but in terms of a ‘regulated
D. Empire building only movement’?
313. ‘Agganasutta’ is related with: A. Myron Weiner B. W.H. Morris Jones
A. Hindu Philosophy B. Buddhist Philosophy C. James Manor D. Atul Kohli
C. Jain Philosophy D. Vedanta Philosophy 323. Who among the following was not a member of the
314. Which one of the following party firstly joined by ‘Syndicate’ group of the Congress party?
E.V. Ramasami Periyar? A. M.N. Nijalingappa
A. Bharatiya Janata Party B. Morarji Desai
B. Indian National Congress C. K. Kamaraj
C. Justice Party D. Y.V. Chavan
D. Communist Party of India 324. A formal organization, according to Chester Bernard,
comes into existence when:
315. The concept of ‘credible minimum deterrent’ relates to
A. There are persons who are willing to contribute
which of the follwing types of weapons?
action
A. Chemical B. Biological
B. There are persons with different goals and purpose
C. Nuclear D. Space-based
C. There are prsons willing to communicate through
316. Who gave the idea of ‘the New Northern Concert of informal channels only
Power’ (NNCP) to analyse the relevance of non-aligned D. Executive is unwilling to handle the economy of
movement in post-cold war era? incentives within an organisation
A. M.S. Rajan 325. The concept of ‘traditional’ authority involves:
B. S.D. Muni A. Use of authority by virtue of their inherited status
C. A.P. Rana by persons
D. Muchkund Dubey B. The exercise of power subject to rules and
317. In which of the following subjects, Rajya Sabha has regulations
got more power than Lok sabha? C. The use of power based on the basis of magical
A. Ratification of President’s rule in a state qualities by a person
B. Creation of a new All India Service D. The use of power/authority as per inherited status
C. Creation of Legislation Council in a state as well as extraordinary contribution to the history
D. Alteration in the boundary of a state of advanced business organisation
94 Political Science

326. Which of the following factors did not contribute to 335. According to Greek system of classification of regimes,
the evolution of the scientific management theory? the degenerative form of rule by many is known as:
A. lack of effective standards of work A. Tyranny
B. lack of clear understanding of worker-management B. Aristocracy
responsibilities C. Anarchic democracy (mob rule)
C. rule-of-thumb based decisions by the management D. Oligarchy
D. proper job design and economic incentives to 336. Who among the following is often portrayed as the
workers ‘father’ of cultural nationalism?
327. Which of the following statem ents about the A. Guisseppe Mazzini
‘Consumer Protection Act 2019’ is NOT true? B. Charles Maurras
A. It has widened the definition of consumer C. Frantz Fanon
B. It provides for E-filing of complaints D. Johann Gottfried Herder
C. It establishes Central Consumer Protection 337. Which one of the following is NOT a cause of
Authority Democratic Shift in the context of Democratisation?
D. It ignores mediation as an alternate disputes A. Crisis of import-substitution
resolution mechanism B. Loss of popularity and legitimacy by military
328. ‘Consumer Protection Bill 2019’ was passed by the regimes
Indian Parliament on: C. Social change
A. 13th July 2019 B. 25th July 2019 D. Media
C. 6th August 2019 D. 9th August 2019 338. Who among the following reformulated H ans
329. ‘Hegemony’ refers to: Morgenthau’s ‘six principles of political realism’ from
A. economic exploitation a feminist perspective?
B. structural inequality A. Judith Butler B. J.Ann Tickner
C. the capacity of bourgeoisie to displace rival views C. Helen M. Kinsella D. Betty Friden
and become, in effect, the common sense of the 339. ‘The combination of liberal democracy and the market
age
had drawn a finishing line in the history of political
D. military domination
and social development’. Who made this statement?
330. Who among the following rejected ‘scientific A. Samuel P. Huntington
determinism’ and advocated political and intellectual B. Benjamin Barber
struggle? C. Francis Fukuyama
A. Antonio Gramsci B. Karl Marx D. Alexander Wendt
C. V.I. Lenin D. Friedrich Engels 340. Which of the following is the executive-bureaucratic
331. Which of the following indicates the nature of arm of the European Union?
planning in India in Contemporary times? A. The European Council
A. Annual Planning B. The European Commission
B. Centralised Planning C. The Council
C. Perspective Planning D. The European Parliament
D. Five year Planning 341. Which of the following is not a principle of jus ad
332. Who among the following was/is the first chairman of bellum (just recourse to war)?
NITI Aayog in India? A. just cause B. legitimate authority
A. Amitabh Kant B. Arvind Pangariya C. proportionality D. humanity
C. Narendra Modi D. Manmohan Singh 342. Who among the following is NOT a key figure of
333. Who among the following has presented the ‘Conservatism’?
incremental model of public policy? A. T.H. Green B. Edmund Burke
A. Henry Fayol B. Charles E. Lindblom C. Michael Oakshott D. Friedrich Von Hayek
C. Fred W. Riggs D. David Easton 343. In which one of the follow ing countries,
334. Which one of the following is not a theory or model ‘consociationalism’ as a power-sharing mechanism is
of representation? not practiced?
A. Trusteeship B. Delegation A. Germany B. Netherlands
C. The Mandate D. Consensus C. Switzerland D. Belgium
Political Science 95
344. Which Article of the Indian Constitution provides (b) Unlocking Human capital
power to the President to promulgate ordinances? (c) Crisis Management
A. Article 123 B. Article 221 (d) Ethics in Governance
C. Article 222 D. Article 223
Choose the correct option:
345. Which of the following pairs are correctly matched? A. (a), (b), (c), (d) B. (d), (c), (b), (a)
(a) John Rawls – Egalitarian C. (a), (d), (b), (c) D. (c), (a), (d), (b)
(b) Plato – Idealist 348. Who is not a supporter of Libertarianism?
(c) Mary Wollstonecraft – Communist
A. Robert Nozick B. Milton Friedman
(d) Gramsci – Feminist
C. Robert Dahl D. F.A. Hayek
Choose the correct answer from the options given
349. Who defined leadership as “the ability of a manager
below: to induce subordinates to work with zeal and
A. (a) and (b) B. (b) and (c)
confidence”?
C. (c) and (d) D. (c) and (a)
A. Koontz and O’Donnell
346. Who had formed the “Society for right to die with B. Chester Bernard
dignity”? C. M.P. Follet
A. Jai Prakash Narayan B. Minoo Masani D. Peter Drucker
C. Krishna Ayyar D. Medha Patkar
350. When did International Labour Organization become
347. Arrange the following four reports of Second a specialized agency of the U nited N ations
Adm inistrative Reforms Comm ission in their Organization?
sequential order: A. 1945 B. 1946
(a) Right to Information C. 1947 D. 1948

ANSWERS
1 2 3 4 5 6 7 8 9 10
B C D A B B C A C A
11 12 13 14 15 16 17 18 19 20
B B B B A B A C B B
21 22 23 24 25 26 27 28 29 30
B B D A A C B A D C
31 32 33 34 35 36 37 38 39 40
C A B C A B A B C B
41 42 43 44 45 46 47 48 49 50
A D C D A D A A B C
51 52 53 54 55 56 57 58 59 60
B C A C C B D B B B
61 62 63 64 65 66 67 68 69 70
B C B B C A B D B C
71 72 73 74 75 76 77 78 79 80
C B C C B A B A B B
81 82 83 84 85 86 87 88 89 90
C C A B A C A A B A
91 92 93 94 95 96 97 98 99 100
B A D A A B B B B B
101 102 103 104 105 106 107 108 109 110
C A D C B A B A D A
111 112 113 114 115 116 117 118 119 120
C A B A A A C A C C
96 Political Science

121 122 123 124 125 126 127 128 129 130
C C D A D C B B B C
131 132 133 134 135 136 137 138 139 140
C A D D D B A B C B
141 142 143 144 145 146 147 148 149 150
C A C A A A B B D D
151 152 153 154 155 156 157 158 159 160
D A C B A A B A D D
161 162 163 164 165 166 167 168 169 170
D C B D B D A B B D
171 172 173 174 175 176 177 178 179 180
A A B C D B C D A D
181 182 183 184 185 186 187 188 189 190
C A C D B C B A B A
191 192 193 194 195 196 197 198 199 200
A A C B A A A B B A
201 202 203 204 205 206 207 208 209 210
B A A A A B B B B D
211 212 213 214 215 216 217 218 219 220
D D A A C C B A B B
221 222 223 224 225 226 227 228 229 230
A A A B D D B D A A
231 232 233 234 235 236 237 238 239 240
B C C C B C A C B C
241 242 243 244 245 246 247 248 249 250
A D A A A B D D A C
251 252 253 254 255 256 257 258 259 260
B C A D D C A B B A
261 262 263 264 265 266 267 268 269 270
B B B B B B B A D D
271 272 273 274 275 276 277 278 279 280
B D B A D B C C C D
281 282 283 284 285 286 287 288 289 290
B A B D B B B B A C
291 292 293 294 295 296 297 298 299 300
A B B A C A D B A C
301 302 303 304 305 306 307 308 309 310
B C D C D D B A B D
311 312 313 314 315 316 317 318 319 320
A D B B C C B C A C
321 322 323 324 325 326 327 328 329 330
A B D A A D D C C A
331 332 333 334 335 336 337 338 339 340
C C B D C D D B C B
341 342 343 344 345 346 347 348 349 350
D A A A A B A C A B
   2204
EXAM BITES

This Pdf Is
Downloaded From
www.exambites.in

Visit www.exambites.in for


More Premium Stuffs,Latest
Books,Test Papers,Lectures etc.
jeeneetadda
jeeneetadda_official
jeeneetadda

VISIT NOW !!

You might also like